Oswaal NTA CUET (UG) 10 Mock Test Sample Question Papers General Test For 2024 Exam 9789357288729, 9357288724

Description of the Product: • Valuable Exam Insight With Latest Solved Papers 2023 & its Explanations • 100% Exam Re

118 78 19MB

English Pages 155 Year 2023

Report DMCA / Copyright

DOWNLOAD PDF FILE

Table of contents :
Cover page
Contents
Oswaal Books Expert Tips to Crack CUET (UG) in the First Attempt
Examination Structure for CUET (UG) & Syllabus
CUET (UG) Question Paper-2023 - 21st May 2023 Shift-1
Mock Test Papers
Mock Test Paper - 1
Mock Test Paper - 2
Mock Test Paper - 3
Mock Test Paper - 4
Mock Test Paper - 5
Mock Test Paper - 6
Mock Test Paper - 7
Mock Test Paper - 8
Mock Test Paper - 9
Mock Test Paper - 10
Solutions
Mock Test Paper - 1
Mock Test Paper - 2
Mock Test Paper - 3
Mock Test Paper - 4
Mock Test Paper - 5
Mock Test Paper - 6
Mock Test Paper - 7
Mock Test Paper - 8
Mock Test Paper - 9
Mock Test Paper - 10
Recommend Papers

Oswaal NTA CUET (UG) 10 Mock Test Sample Question Papers General Test For 2024 Exam
 9789357288729, 9357288724

  • 0 0 0
  • Like this paper and download? You can publish your own PDF file online for free in a few minutes! Sign Up
File loading please wait...
Citation preview

For 2024 Exam

BEST SELLER

GENERAL TEST Section III (Compulsory) Strictly as per the Latest Examination Pattern issued by NTA

The ONLY book you need to Ace CUET (UG)

1

2

3

4

5

Valuable Exam Insights

100% Exam Readiness

Extensive Practice

Concept Clarity

Final Boost

With Latest Solved Papers 2023

With 10 Solved Sample Question Papers

With 650+ Latest Typologies Questions

Learn Key Concepts through Detailed Explanations

With Tips & Tricks to ace CUET (UG) in 1st Attempt

(1)

YEAR 2024

3rd EDITION

ISBN SYLLABUS COVERED

“9789357288729”

CUET (UG) General Test PUBLISHED BY

C OPYRIG HT

RESERVED BY THE PUBLISHERS

All rights reserved. No part of this book may be reproduced, stored in a retrieval system, or transmitted, in any form or by any means, without written permission from the publishers. The author and publisher will gladly receive information enabling them to rectify any error or omission in subsequent editions.

OSWAAL BOOKS & LEARNING PVT. LTD. 1/11, Sahitya Kunj, M.G. Road, Agra - 282002, (UP) India

1010, Cambourne Business Centre Cambridge, Cambridgeshire CB 236DP, United kingdom

0562-2857671

[email protected]

www.OswaalBooks.com

DI SC L A IMER

This book is published by Oswaal Books and Learning Pvt Ltd (“Publisher”) and is intended solely for educational use, to enable students to practice for examinations/tests and reference. The contents of this book primarily comprise a collection of questions that have been sourced from previous examination papers. Any practice questions and/or notes included by the Publisher are formulated by placing reliance on previous question papers and are in keeping with the format/pattern/ guidelines applicable to such papers. The Publisher expressly disclaims any liability for the use of, or references to, any terms or terminology in the book, which may not be considered appropriate or may be considered offensive, in light of societal changes. Further, the contents of this book, including references to any persons, corporations, brands, political parties, incidents, historical events and/or terminology within the book, if any, are not intended to be offensive, and/or to hurt, insult or defame any person (whether living or dead), entity, gender, caste, religion, race, etc. and any interpretation to this effect is unintended and purely incidental. While we try to keep our publications as updated and accurate as possible, human error may creep in. We expressly disclaim liability for errors and/or omissions in the content, if any, and further disclaim any liability for any loss or damages in connection with the use of the book and reference to its contents”.

Kindle ( 2Edition )

Preface The National Testing Agency took over the responsibility for conducting CUET in 2021. The Common University Entrance Test (CUET) had been introduced as the assessment test for admission to UG programs across all Central Universities. This has been done to ensure that students across the country, whether from urban areas or rural/ remote areas have access to the same opportunities/environment for assessment. As an all-India test conducted by the National Testing Agency (NTA), it provides a single-window opportunity for admissions, and streamlines the admissions process, and standardizes the evaluation process. A full Computer Based Test (CBT), the question paper is divided into four sections: Sections IA & IB contain language-specific questions, Section II comprises domain questions, and Section III contains questions on general topics. All questions are MCQ based. The curriculum for CUET is based on the National Council of Educational Research and Training (NCERT) syllabus for class 12 only. CUET (UG) scores are mandatory required while admitting students to undergraduate courses in 44 central universities. A merit list will be prepared by participating Universities/organizations. Universities may conduct their individual counselling on the basis of the scorecard of CUET (UG) provided by NTA. Almost 1.92 million candidates registered for CUET (UG) in 2023. Candidates have been quite anxious about appearing for CUET (UG), however, with the right preparation strategy and resources, you can secure a good rank in CUET (UG). This book is a step in that direction, helping candidates refine their examination strategy, and bridging the gap between their present preparation level and a rank-securing preparation level.

A merit list will be prepared by participating Universities/organizations. Universities may conduct their individual counselling on the basis of the scorecard of CUET (UG) provided by NTA.

A few benefits of studying from Oswaal Mock Test Papers • • • • •

Valuable Exam Insight With Latest Solved Papers 2023 & its Explanations 100% Exam Readiness With 10 Mock Test Papers Fully Solved Extensive Practice With 650+ Latest Typologies Questions Concept Clarity Learn Key Concepts through Detailed Explanations Final Boost With Tips & Tricks to ace CUET (UG) in 1st Attempt

Our Heartfelt Gratitude! Finally, we would like to thank our authors, editors, and reviewers. Special thanks to our students who send us suggestions and constantly help improve our books. We promise to always strive towards ‘Making Learning Simple’ for all of you. Wish you all Happy Learning!

(3)

-Team Oswaal Books

Contents l Oswaal Books Expert Tips to Crack CUET (UG) in the First Attempt l Examination Structure for CUET (UG) & Syllabus

l CUET (UG) Question Paper-2023 - 21st May 2023 Shift-1

5-5

6-8

1 - 13

Mock Test Papers l Mock Test Paper - 1

14 - 18 19 - 23 24 - 28 29 - 33 34 - 38 39 - 43 44 - 48 49 - 53 54 - 58 59 - 63

l Mock Test Paper - 2 l Mock Test Paper - 3 l Mock Test Paper - 4 l Mock Test Paper - 5 l Mock Test Paper - 6 l Mock Test Paper - 7 l Mock Test Paper - 8 l Mock Test Paper - 9

l Mock Test Paper - 10

Solutions l Mock Test Paper - 1

64 - 71 72 - 79 80 - 88 89 - 96 97 - 104 105 - 112 113 - 120 121 - 128 129 - 136 137 - 143

l Mock Test Paper - 2 l Mock Test Paper - 3 l Mock Test Paper - 4 l Mock Test Paper - 5 l Mock Test Paper - 6 l Mock Test Paper - 7 l Mock Test Paper - 8

l Mock Test Paper - 9

l Mock Test Paper - 10 



(4)

Oswaal Expert Tips toinCrack Oswaal Books ExpertBooks Tips to Crack CUET (UG) the First CUET (UG) in the First Attempt

empt

Excited about your UG but unsure if you will get admission to your preferred university? In a major announcement by the chairman of the University Grants Commission, the N onal Te ng Agency will be conduc g the Common Univers es Entrance Test (CUET (UG) ) for undergraduate programs in Central Univers es for the upcoming academic session. However, the UGC Chairperson also stated that CUET (UG) will not just be limited to admissions to Central Univers es. Many prominent private univers es have indicated tha hey would also like to adopt a common entrance exam for undergraduate admissions and take admissions on the basis of CUET (UG) scores. This makes CUET (UG) a very important examina n in itself and hence it becomes mandatory to be aware o he s & tricks that could help you ace the exam on the first empt.

The first step is to understand ern of the ex on. The ons, CUET includes three on 1 includes que on based on languages, sec n 2 includes 27 domain-based subjects and sec n 3 includes General Test. The syllabus of the upcoming Common University Entrance Test, CUET is completely based on the syllabus of class 12 th . No ques n will be asked from class 11th syllabus.

While preparing for the exam, i s important to iden y the importan opics and prac e important ques ons from those topics. Pra ce important que ons through Oswaal Ques n Bank and Sample n Papers, Li ng Ques topics also helps in iden ying the weak areas that need special e. The aspirants can effort and start preparing to focus on the areas tha hey consider to be tough, followed by the ones that are their strengths.

Make a habit of preparing notes f ro m t h e b e g i n n i n g o f t h e prepar on. It will not only help in c but making the study syst also make the revision o he syllabus easy even when you e to revise. might have limited

Collec g and preparing from the appropriate study material cannot be ignored as irrelevant. The books chosen by the aspirants to study from should be on the lines of the current syllabus and the ones that could help you with revision before the ex on.

Make sure to revise as much as possible. The revision will help the aspirants in keeping the concepts fresh in their minds un the day of the ons. They may refer to a few good final ex pr ce ques ons and concise revision notes to achieve their desired results.

Devote a sufficient amount o me to all the ons of the ex ons. This requires a wellmade plan and an honest adherence to the said plan. Priori e the most importan opics or the topics that the aspirants are not familiar with to be able to master them in e.

(5)

1712

With this said, an important ques n tha s gaining ground amongst students who will be appearing for this exam is if they should take coaching to get themselves ready for the exams. The answer is a simple no, the exam will simply not require any coaching as it is completely based on the Class 12th syllabus which will be quite fresh in students' minds as they will be just out of school. All they need is a good revision and prac e of ques ons from Oswaal Ques n Bank and Sample Ques n Papers for CUET (UG) prepar ons.

Examination Structure for CUET (UG) CUET (UG) – is consisted of the following 4 Sections:    

Section IA Section IB Section II Section III

– – – –

13 Languages 20 Languages 27 Domain specific Subjects General Test

Choosing options from each Section is not mandatory. Choices should match the requirements of the desired University. Broad features of CUET (UG) - are as follows: Section

Subjects/ Tests

Section IA Languages

There are 13* different languages. Any of these languages may be chosen.

Section IB Languages Section II Domain

Section III General Test

*

Questions to be Attempted 40 questions to be attempted out of 50 in each language

Question Type

Duration 45 Minutes for each language

There are 20** Languages. Any other language apart from those offered in Section I A may be chosen.

Language to be tested through Reading Comprehension (based on different types of passages–Factual, Literary and Narrative, [Literary Aptitude and Vocabulary]

There are 27*** Domains specific sub- 40 Questions to be jects being offered under this Section. attempted out of 50 A candidate may choose a maximum of Six (06) Domains as desired by the applicable University/Universities.

• Input text can be used for MCQ Based Questions • MCQs based on NCERT Class XII syllabus only

45 Minutes for each Domain Specific Subjects.

For any such undergraduate programme/ programmes being offered by Universities where a General Test is being used for admission.

• Input text can be used for MCQ Based Questions • General Knowledge, Current Affairs, General Mental Ability, Numerical Ability, Quantitative Reasoning (Simple application of basic mathematical concepts arithmetic/ algebra geometry/ ensuration/s tat taught till Grade 8), Logical and Analytical Reasoning

60 Minutes

50 Questions to be attempted out of 60

Languages (13): Tamil, Telugu, Kannada, Malayalam, Marathi, Gujarati, Odiya, Bengali, Assamese, Punjabi, English, Hindi and Urdu

** Languages (20): French, Spanish, German, Nepali, Persian, Italian, Arabic, Sindhi, Kashmiri, Konkani, Bodo, Dogri, Maithili, Manipuri, Santhali, Tibetan, Japanese, Russian, Chinese. *** Domain Specific Subjects (27): 1.Accountancy/ Book Keeping 2.Biology/ Biological Studies/ Biotechnology/Biochemistry 3.Business Studies 4.Chemistry 5.Computer Science/ Informatics Practices 6.Economics/ Business Economics 7.Engineering Graphics 8.Entrepreneurship 9.Geography/Geology 10.History 11.Home Science 12.Knowledge Tradition and Practices of India 13.Legal Studies 14.Environmental Science 15.Mathematics 16.Physical Education/ NCC /Yoga 17.Physics 18.Political Science 19.Psychology 20.Sociology 21.Teaching Aptitude 22.Agriculture 23. Mass Media/ Mass Communication 24.Anthropology 25.Fine Arts/ Visual Arts (Sculpture/ Painting)/Commercial Arts, 26. Performing Arts – (i) Dance (Kathak/ Bharatnatyam/ Oddisi/ Kathakali/Kuchipudi/ Manipuri (ii) Drama- Theatre (iii) Music General (Hindustani/ Carnatic/ Rabindra Sangeet/ Percussion/ Non-Percussion), 27. Sanskrit • A Candidate can choose a maximum of any 3 languages from Section IA and Section IB taken together.However, the third language chosen needs to be in lieu of 6th domain specific Subject chosen by the candidate - as applicable (so the maximum number of tests to be taken remains 9 only i.e. 2 Languages+6 Domain Specific Subjects+1 General Test OR 3 Languages+5 Domain Specific Subjects+1 General Test: flexibility being provided to help a candidate apply for many Universities depending on their eligibility conditions). • Section II offers 27 Subjects, out of which a candidate may choose a maximum of 6 Subjects. • Section III comprises General Test.

(6)

Contd... • For choosing Languages from Section IA and IB and domain specific Subjects from Section II and General Test under Section III, the Candidate must refer to the requirements of his/her intended University. Mode of the Test

Computer Based Test-CBT

Test Pattern

Objective type with Multiple Choice Questions

Medium

13 languages (Tamil, Telugu, Kannada, Malayalam, Marathi, Gujarati, Odiya, Bengali, Assamese, Punjabi, English, Hindi and Urdu) Section IA & IB: Language to be tested through Reading Comprehension (based on different types of passages–Factual, Literary and Narrative [Literary Aptitude & Vocabulary]

Syllabus

Section II : As per NCERT model syllabus as applicable to Class XII only Section III : General Knowledge, Current Affairs, General Mental Ability, Numerical Ability, Quantitative Reasoning (Simple application of basic mathematical concepts arithmetic/algebra geometry/mensuration/stat taught till Grade 8), Logical and Analytical Reasoning

Level of questions for CUET (UG) : All questions in various testing areas will be benchmarked at the level of Class XII only. Students having studied Class XII Board syllabus would be able to do well in CUET (UG). Number of attempts: If any University permits students of previous years of class XII to take admission in the current year also, such students would also be eligible to appear in CUET (UG). Choice of Languages and Subjects: Generally the languages/subjects chosen should be the ones that a student has opted in his latest Class XII Board examination. However, if any University permits any flexibility in this regards, the same can be exercised under CUET (UG) also. Candidates must carefully refer to the eligibility requirements of various Central Universities in this regard. Moreover, if the subject to be studied in the Undergraduate course is not available in the list of 27 Domain Specific Subject being offered, the Candidate may choose the Subject closest to his/her choice for e.g. For Biochemistry the candidate may choose Biology.

GENERAL AWARENESS

SYLLABUS

1. Indian History → Prehistoric Period, The Harappan Civilisation, The Vedic Period, Jainism and Buddhism, Magadha Empire, The Mauryan Empire, The Sangam Period, The Gupta Period, Age of Harshavarthana, Rashtrakutas, Gangas, Pallavs, The Cholas 2. World and Indian Geography → 2.1. ( World Geography): Universe, Solar System, Planets, Sun, Earth, Structure of Earth’s Interior, Plate Tectonic Theory, Volcanism, Rocks, Landforms, Atmosphere, Jet Stream, Oceans, Continents, Important Facts → 2.2. ( Indian Geography): India at a Glance, Physical Features, Islands, Drainage System of India, Climate, Agriculture, Soils of India, Mineral Resources, Transport 3. Indian Polity and Constitution → Constitution, Preamble, Sources of the Constitution, Parts of the Constitution, Amendability of the Preamble, Schedules of the Constitution, Union and its Territories, Citizenship, Rights, Directive Principles of State Policy, Union Government, State Government, Local Government, Elections, Planning Commission, NDC, Finance Commission, Amendments of the Constitution 4. Indian Economy → Economy, Economic Growth, Indian Economy, National Income of India, Planning of India, India’s New Economic Policy, Flagship Programmes of Government of India, Agriculture, Indian Financial System, Indian Currency System, Macro Economic, Problems in India. 5. General Science → 5.1. ( Physics): Units, Motion, Friction, Gravitation, General Properties of Matter, Heat and Thermodynamics, Thermal Expansion, Waves, Light, Electricity and Magnetism → 5.2. ( Chemistry): Physical and Chemical Changes, Matter; Atom, Molecule and of Element, Corrosion, Renewable and Non-renewable Natural Resources, Coal, acids, Bases and Salts, Some important Compounds in Everyday life, Polymers → 5.3. ( Biology): Living World, The Cell, Nucleic Acids, Human Systems, Respiratory System, Vitamins, Major Enzymes of Digestion, Blood (Lymphatic System), Central Nervous System, Some Human Diseases Caused by Viruses and Bacteria / Fungi, Animal/Human Diseases Caused by Fungi, Important Vaccines Discoverer, Ebola Virus, Ecology, Pollution, Biotechnology, Some Important Branches of Biology, Some Important Discoveries, Some Important Antibiotics, Environment and Ecology 6. Computer Science and Technology → Computer, Components of Computer, Memory, Hardware, Software, Networking, Security Threats, Internet, Some commonly used terms, Super Computer. 7. General Knowledge → Art & Culture, Important Inventions & Discoveries Science & Tech, Awards & Honours, Books & Authors, Decodes & Dates, Important Organizations, Sports, Miscellaneous.

(7)

Contd... QUANTITATIVE APTITUDE

Number System → Introduction to Numbers; Various types of Numbers; Even numbers; Odd numbers, Prime numbers; Rational numbers; Irrational number; Tests of divisibility; Standard results; Division algorithm; Arithmetic Progression; Geometric Progression; Unit place value in the Product of Numbers; Problems on Numbers & H.C.F and L.C.M. 2. Operations → BODMAS Rule; Squares; Cubes; Square Roots; Cube Roots; Fractions; Types of fractions; Decimals; Surds and Indices; Properties of Surds and Indices. 3. Algebraic Expressions → Constants; Variables; Algebraic expressions; Equations; Linear equation; Quadratic equations; Factors; Solutions. → 3.1. Average and problems on Ages → Average; Properties of Averages; Problems on Ages → 3.2. Percentage, Profit-Loss and Discount → Percentage meaning; Fraction representation, Consumption & Expenditure, Reducing & Exceeding prices, Voters in an Election; Percentage increase or increasing and decreasing problems ; Profit, Loss; Buy-sell problems ; Discounts, Successive discount problems; Tax Miscellaneous Questions. → 3.3. Ratio and Proportion, Alligation & partnership → Ratio; Proportion; Properties of Ratio and Proportion; Alligation; Partnership; Active Partner; Sleeping Partner; Types of Partners; Shares and Partners; Coins and Rupees; Income and Expenditure → 3.4. Simple Interest & Compound Interest → Interest; Principal; Rate of Interest; Time; Compound Interest; Difference between Compound and Simple Interest; Miscellaneous questions on Simple Interest and Compound Interest. → 3.5. Time & Work and Pipes & Cisterns → Time and Work; Work and Wages; Efficiency of worker; worker leaves or joins; Man−day work done; Pipes and Cisterns; Inlet; Outlet; Quantity and Time; Miscellaneous Question. → 3.6. Speed, Time and Distance → Speed; Time; Conversion of Units; Average Speed; Speed of Stream; Upstream; Downstream; Problems on Trains in same direction; Problem on train cross each other in opposite direction; Relative Speed. 4. Mensuration → Area and Perimeter of triangles, Rhombus, Quadrilaterals, Circles; Volume of 3D figures; Surface area; Total surface area; Cylinder, Sphere, Hemisphere, Cube, Cuboid, Cone, Frustum of a cone; Standard Results; Equilateral triangle; Parallelogram; Rhombus, Trapezium; Regular Hexagon etc. 5. Geometry → Lines; Angles; Bisectors; Polygons; Coordinate geometry; Types of triangles; Congruency; Similar triangles; Circles; Chord; Tangents; Secant; Cyclic quadrilateral; Circumcircle; Incircle. 6. DATA Interpretation → Data Interpretation; Tables; Bar Graphs; Line Graphs; Pie Charts; Mix Graph. 1.

LOGICAL REASONING

1. Analogy → Alphabet Analogy; Number Analogy; Word Analogy; Analogy based on prime number and mixed operation based Analogy. → 1.1. Alphabet Series → Finding the next or missing alphabet in a series. → 1.2. Number Series: N2 series, N2 + 1 series, N2 – 1 series, Cube series, Prime number series, Mixed Operation series etc. 2. Direction and Distance : Concept of direction, Different types of directions, Orientation, Turns, Concept of Shadow, Finding distance, Finding shortest distance, Different types of questions- Based on turns and rotations, Based on shadow, Based on finding shortest distance, Based on coded form, Based on seating arrangements, Based on clock timing indication. Coding and Decoding : Letter coding, Alphabetical coding, Direct letter coding, Number coding, Deciphering message coding/Message coding, Substitution coding, Mixed coding and Alpha-Numeric coding. 3. Ranking : Age-Based Comparison, Height Based Comparison, Integrated Height and Age-Based comparison, Linear seating arrangement comparison. No. of elements between two-element, Position interchanges in a queue, Puzzle based comparison. 4. Blood Relation : Dialogue/Conversation Based, Puzzles, Coding-Decoding, Linear seating arrangement base blood relations, Circular seating arrangement based blood relations, Mixed blood relation. 5. Mathematical Operations : Interchanging operators, Symbols, Balancing the equation and Trick based mathematical operations. Venn Diagram : Relation Based Venn Diagram, Analysis Based Venn Diagram. 6. Seating Arrangement → Facing towards the center, facing outwards/opposite/outside the center, facing inside and outside the center Rectangular or square arrangement. Triangular arrangement (facing the centre, facing opposite to the centre). 7. Syllogism → Positive conclusion, Negative conclusion, Either-OR/Complementary pair, No conclusion. 8. Statement and Conclusion → One statement multiple conclusions, Two or more than two statements, Multiple conclusions. Statement and Assumptions, Statement and Course of Action, Cause and Effect, Statement and Arguments, Inference. → 8.1. F  igure Completion → Based on movement, Based on Rotation 45°/90°/135°/180°, Symmetry of image/ Water and Mirror Concept, Miscellaneous Concept. → 8.2. F  igure Formation → Assembling main figure from split figures, Fragmentation of the main figure into pieces. Making up a figure from given components, Making up a three dimensional figure by paper folding. → 8.3. F  igure Counting → Based on triangles, Based on squares, Based on Rhombus, Based on rectangles, Based on the circle, Based on combined figures, Counting of straight lines. 9. Non-Verbal Series → Figure series, Figure series based on rotation, Based on addition of symbols, Based on deletion of symbols, Based on replacement, Based on replacement and arrangements, Similar Figure present in questions figures, Simultaneous operations of rotation, addition and replacement. 

(8)

CUET (UG) Question Paper - 2023 National Testing Agency 21st MAY 2023 – SHIFT 1

Section - III (General Test) Examination Duration: 60 Minutes

Maximum Marks - 250

General Instructions:

Marking scheme of the test: (a) There are 60 questions asked in the section- III. But there are 50 questions to be attempted in the section - III. (b) Correct answer or the most appropriate answer will be given five marks (+5). (c) Any incorrect option marked will be given minus one mark (–1). (d) Unanswered/Marked for review will be given no mark (0).

1. If the word ‘LEADER’ is coded as 20-13-9-12-13-26. How would you write “LIGHT” (A) 20-16-15-17-22 (B) 20-17-15-16-28 (C) 20-15-16-18-23 (D) 20-16-17-15-27 Ans. Option (B) is the correct. Explanation: Logic: Table of Alphabetical series (point to remember) Alphabets

A

B

C

D

E

F

G

H

I

J

K

L

M

Positional value

1

2

3

4

5

6

7

8

9

10

11

12

13

Alphabets

Z

Y

X

W

V

U

T

S

R

Q

P

O

N

Positional value

26

25

24

23

22

21

20

19

18

17

16

15

14

L ⇒ 12 + 8 = 20 E ⇒ 5 + 8 = 13 A⇒1+8=9 D ⇒ 4 + 8 = 12 E ⇒ 5 + 8 = 13 R ⇒ 18 + 8 = 26 Therefore, LIGHT can be coded as L ⇒ 12 + 8 = 20 I ⇒ 9 + 8 = 17 G ⇒ 7 + 8 = 15 H ⇒ 8 + 8 = 16 T ⇒ 20 + 8 = 28 So “LIGHT” will be coded as = 20 - 17 - 15 - 16 - 28. 2. In a class boys stand in a single line. One of the boys is seventeenth in order from both the ends. How many boys are in the class? (A) 34

(B) 33

(C) 32

(D) 27

Ans. Option (B) is correct. Explanation: Given: One of the boys is seventeenth in order from both ends. Hence, Total boys in the class = 16 + 1 + 16 = 33

3. If the 2nd half of the letters of the word INTERMEDIATE are reversed and placed before 1st half of the letters, which letter will be 2nd to the right of 10th letter from the right? (A) A (B) D (D) I (C) E Ans. Option (B) is correct. Explanation: Word: INTERMEDIATE Reverse 2nd half and placed before 1st half ETAIDEINTERM 10 - 2 = 8th from right, which is D.

OSWAAL CUET (UG ) 10 Mock Test Papers GENERAL TEST

2

4. Arrange the following in meaningful sequence: (A) Key (B) Door (C) Lock (D) Room Choose the most appropriate answer from the options given below: (A) A, C, B, D (B) D, C, A, B (D) C, B, D, A (C) B, A, D, C Ans. Option (A) is correct. Explanation: (A) Key

(C) Lock

(B) Door

(D) Room

5. Which of the following is a metal (A) Carbon (B) Mercury (C) Sulphur (D) Iodine Ans. Option (B) is the correct. Explanation: Mercury is a metal. It is a chemical element represented by the symbol Hg with atomic number 80 and an atomic mass 200.59. It is a d-block element which is present in period 6 and group 12 of the periodic table. It is the only metal liquid at room temperature, which is why it is used in thermometers to detect body temperature. Carbon is a non-metal which is represented by the symbol C. It has an atomic number of 6 and an atomic mass of 12. It is a p-block element belonging to period 2 and group 14 of the periodic table. Sulphur is a non-metal which is represented by the symbol S. It has an atomic number of 16 and an atomic mass of 32. It is a p-block element belonging to period 3 and group 16 of the periodic table. It is yellow-coloured and crystalline in nature. Iodine is a non-metal which is represented by the symbol I. It is a halogen with an atomic number 53 and atomic mass 126.9. It is a p-block element and belongs to the periodic table’s group 17 and period 5. 6. Match List I with List II LIST I Movement A. Quit India Movement

LIST II Purpose I. To achieve self Government in India of demand for large political representation

B. Civil II. Compelled British’s to Disobedience leave India Movement C. Non Cooperation Movement

III. Refusal to obey certain laws orders or Commands of the Government

D. Home rule league Movement

IV. Indians resigning their title boycotting foreign goods and Government institutions refused to pay taxes

Choose the most appropriate answer from the options given below: (A) A-I, B-III, C-II, D-IV (B) A-II, B-III, C-I, D-IV (C) A-III, B-II, C-IV, D-I (D) A-II, B-III, C-IV, D-I Ans. Option (D) is the correct. Explanation: Quit India Movement was started by Mahatma Gandhi in which he demanded the end of British rule in India. This was launched at the Bombay session of All India Congress Committee on 8 August 1942. Gandhiji’s famous slogan of ‘Do or Die’ was the driving force of this movement. The phrase ‘Quit India’ was coined by Yusuf Meherally. However, this movement was suppressed by British forces. The Civil Disobedience Movement was launched under the leadership of Mahatma Gandhi in 1930 as part of the larger Indian National Congress’s campaign for freedom. He urged Indians to peacefully protest by refusing to cooperate with British authorities. Indians boycotted British institutions, refused to pay taxes, and participated in acts of non-cooperation. There were mass protests, demonstrations, and acts of civil disobedience taking place throughout the country. The Salt March or Dandi March was one of the important events of Civil Disobedience Movement. Non-Cooperation Movement was a movement in which Indians united against British colonial rule with the suspension of cooperation with British-run institutions such as schools, colleges, courts, and legislative councils. Indians boycotted British goods and promoted the use of Indianmade products. However, this movement was suspended in 1922 due to a violence incidence at Chauri-Chaura, Uttar Pradesh. Home rule league movement was started by Annie Besan and it aimed to achieve selfgovernment in India within the British Raj. She formed the All India Home Rule League in September 1916. She was the first woman president of the Indian Congress. 7. Choose the one which is different from the rest three. (A) 431 (B) 162 (C) 831 (D) 232 Ans. Option (C) is correct. Explanation: Logic: 431  4  3  1  12 162  1  6  2  12 831  8  3  1  24  12 232  2  3  2  12 Hence, we can see that option (C) is not following the same pattern as others. 8. X got 98 marks in his exam which is 56 % of the total marks. What is the maximum marks of the exam? (A) 150 (B) 175 (C) 200 (D) 225

3

CUET(UG) Solved Paper 2023 (21st May Shift -1)

Ans. Option (B) is correct. Explanation: Let the total marks be x. According to question, 56% of x = 98 56 x = 98 100 98  100 x  175 56 Hence,the maximum marks of the exam is 175. 9. Which of the following speeds is the least? (A) 50 meter / second (B) 50 meter/minute (C) 70 km / hour (D) 5 km / minute Ans. Option (B) is correct. Explanation: Logic: Solving by options, (A) 50 m/sec

50  m   60  s  70 km 70  1000 700  m  (C)   hour 3600 36  s  5 km 5  1000 500  m  (D)   min 60 6  s  (B)

50 m/minute =

Hence, option (B) is correct answer. 10. When seen through a mirror, a clock shows 3:30. What is the correct time? (A) 2: 30 (B) 8: 30 (C) 5: 30 (D) 4: 30 Ans. Option (B) is correct. Explanation: The correct time is: (11 : 60 - 03 : 30) = 8 : 30 11. A is 3 years younger than C but one year older than D. D is one year older than B but 4 years younger than C. C is 15 years old. What is the age of B in years (A) 13 (B) 12 (C) 11 (D) 10 Ans. Option (D) is correct. Explanation: Let the present age of C = x Present age of D = y Age of A = x - 3 Age of C = 15 years A is 3 years younger than C Hence, A’ s age = 15 - 3 = 12 years Age of D = C - 4 = 15 - 4 = 11 years Age of D = Age of B + 1 Age of B = 11 - 1 = 10 years

12. What is the smallest square number which is divisible by 4,6 and 32? (A) 100 (B) 196 (C) 96 (D) 576 Ans. Option (D) is correct. Explanation: LCM of 4, 6, 32 =2×2×3×8 = 96 Now find the multiple of 96. 96 is not a perfect square number. Hence, 576 is correct answer. 13. The only Indian who received noble prize in literature is. (A) Bankim Chandra Chatterjee (B) Toradutt (C) R.K. Narayan (D) Rabindra Nath Tagore Ans. Option (D) is the correct. Explanation: Rabindra Nath Tagore is the only Indian to receive the noble prize in Literature in the year 1913 for his novel Gitanjali. He was the first Indian and Asian to be awarded the prize. Rabindra Nath Tagore was a Bengali writer, poet, composer, philosopher, painter, and social reformer. He is the writer of India’s national anthem-Jana Gana Mana and Bangladesh’s national anthem- Amar Shonar Bangla. Some of his best known works are Ghare-Baire , Gore etc. The Nobel Prize in Literature is a literary honour given by the Swedish Academy and is one of the five Nobel Prizes established by the will of Alfred Nobel in 1895. The Nobel laureate is honoured with a gold medal, reward money and a diploma with citation. The first Nobel Prize in literature was given in 1901 to the French poet and essayist Sully Prudhomme. 14. Who won the ‘Noble Prize’ for in the field of “physiology or medicine”? (A) C.V. Raman (B) Jagdish Chandra Bose (C) Homi Jehangir Bhabha (D) Har Gobind Khorana Ans. Option (D) is the correct. Explanation: Har Gobind Khorana won the Noble Prize in the field of physiology or medicine in the year 1968. He shared the prize along with Marshall W Nirenberg and Robert W. Holley. The three of them were awarded for their interpretation of the genetic code and its function in protein synthesis. Har Gobind Khorana was an Indian American biochemist who has won many national and international awards for his research. The Nobel Prize in Medicine is awarded by the Nobel Assembly at the Karolinska Institute for outstanding discoveries in physiology or medicine every year. It is one of the five Nobel

OSWAAL CUET (UG ) 10 Mock Test Papers GENERAL TEST

4

Prizes that are a part of the will of Alfred Nobel in 1895. The Nobel laureate is honoured with a medal, reward money and a diploma with citation. The first Nobel Prize in literature was given in 1901 to the German physiologist, Emil von Behring, for his work on serum therapy and the development of a vaccine against diphtheria.

Ans. Option (C) is correct. Explanation:

15. The relationship between the values of a country’s imports and its exports is called. (A) Balance of Trade (B) Balance of Payment (C) Balance of currency (D) Bill of exchange Ans. Option (A) is the correct. Explanation: The relationship between the value of country’s imports and exports is called Balance of Trade(BOT). It is simply the difference between country’s imports and exports of goods is also known as net exports or commercial balance. If the import is more than the export, it is known as trade deficit and if the exports are more than imports, it is known as trade surplus. Balance of Payment is basically the summary of country’s economic transactions with the world during an accounting year. This includes all the inflow and outflow of funds that have taken from the country. The BOP statement is indicative of country’s surplus or deficit of funds. The Bill of exchange is defined under the Negotiable Instruments Act 1881. It is a written instrument that directs a person to pay the designated sum of money to the bearer of the instrument. It basically ensures a timely payment. It involves three parties- the drawee is the party that pays the sum, the payee receives that sum, and the drawer is the one that obliges the drawee to pay the payee. 16. Find out which of the answer figures (1), (2), (3) and (4) completed the figure matrix?

A B

A B

A B

B

B

B

A B

A B

?

(A)



(B)

A

A

B

B

(C)



(D)

A

A

B

B

A B

A B

A B

B

B

B

A B

A B

?

Logic: Follow the pattern and the symmetry. Hence, A B 17. Match List I with List II LIST I River

LIST II City

A.

Mahanadi

I.

Ludhiana

B.

Godavari

II.

Cuttack

C.

Sutlej

III

Lucknow

D.

Gomti

IV.

Nasik

Choose the most appropriate answer from the options given below: (A) A-II, B-IV, C-III, D-I (B) A-II, B-IV, C-I, D-III (C) A-IV, B-II, C-I, D-III (D) A-III, B-I, C-II, D-IV Ans. Option (B) is the correct. Explanation: Mahanadi River flows through Odisha and Chattisgarh and ends in Bay of Bengal. It is a Peninsular river in East Central India. The city of Cuttack is situated on the banks of the Mahanadi river. Godavari is the second longest River of India after Ganga. It passes from the states of Maharashtra, Andhra Pradesh, Telangana, Chhattisgarh, Odisha and empties into Bay of Bengal. Nasik is situated on the banks of Godavari river. Sutlej is the longest of the five Rivers that flows through Punjab in India and Pakistan. It is the easternmost tributary of Indus River and is also known as Satadru. Ludhiana stands on the old bank of Sutlej River. Gomti River is a tributary of the River Ganges and originates in Pilibhit, Uttar Pradesh. It flows through Lucknow and supply water to the city. 18. The point (-2, 3) lies in which quadrant? (A) I (C) III Ans. Option (B) is correct.

(B) II (D) IV

5

CUET(UG) Solved Paper 2023 (21st May Shift -1)

21. Which of the following players didn’t receive Medal in Tokyo Olympics 2020?

Explanation: 3

(A) PV Sindhu

(2, 3)

(B) Neeraj Chopra x axis

2

(D) Ravi Kumar Dahiya Ans. Option (C) is correct.

(2,3)3)lies lies quadrant. (-2, inin II II quadrant. 19. If selling price of 80 articles is equal to the cost price of 100 articles, then find the gain percentage. (A) 30 % (B) 25% (C) 40 % (D) 50 % Ans. Option (B) is correct. Explanation: Selling Price of 80 articles = C.P of 100 articles SP 100 5 = = CP 80 4 Gain% 

(C) Abhinav Bindra

SP  CP  100 CP

 SP    1   100 CP   5     1   100 4  54   100 4 = 25% 20. A railway half-ticket costs half the full ticket. However the reservation charge for all the tickets is constant. One full reserved ticket for a journey is ` 525. If the cost of one full and one half reserved ticket for the same journey is ` 850, then what is the reservation charge per ticket? (A) ` 120 (B) ` 150 (C) ` 125 (D) ` 115 Ans. Option (C) is correct. Explanation: Let the reservation charge be x Cost of full ticket = y Price of one full reserved ticket = 525 x + y = 525 y Cost of 1 full and 1 half ticket  x  y  x  2 3y 2x   850 2 3y 2  525  y    850 2 2(1050 - 2y) + 3y = 850 × 2 y = 2100 - 1700 = 400 x = 525 - y x = 525 - 400 = `125

Explanation: Abhinav Bindra didn’t receive medal in Tokyo Olympics 2020. He is a retired shooter and the first Indian to win a gold medal at the Olympics. He had won the gold medal in shooting at the 2008 Summer Olympics. PV Sindhu won the Bronze medal in women’s singles badminton at Tokyo Olympics 2020. She is the first Indian woman and second athlete to win two individual Olympic medals. Ravi Kumar Dahiya won the Silver medal in men’s 57 kg freestyle wrestling at Tokyo Olympics 2020, freestyle wrestling. Neeraj Chopra became India’s second individual Olympic champion, after Abhinav Bindra, with his men’s javelin throw gold at Tokyo 2020. It was India’s first track-and-field medal at any Olympic Games. 22. Which book is Written by Dr. S. Radha Krishnan? (A) The world’s largest democracy (B) India divided (C) Indian Philosophy (D) India Priceless heritage Ans. Option (C) is the correct. Explanation: Dr. Sarvapalli Radhakrishnan wrote the book, ‘Indian Philosophy’. He was the second President (1962 to 1967) of India. The History of the World’s Largest Democracy is a non-fiction book which is written by Indian historian Ramachandra Guha. The famous book ‘India divided’ was written by Dr. Rajendra Prasad. The book “India Priceless heritage” was written by N.A. Palkhivala, was an Indian lawyer and a noted jurist. x x x x 23. If the median of , x , , and (where x > 0 ) is 8, 4 2 5 3 then the value of x will be (A) 24 (B) 32 (C) 8 (D) 16 Ans. Option (A) is the correct. Explanation: Arrange in increasing order x x x x , , , ,x 5 4 3 2 Since number of terms = 5 ⇒ odd Hence, Median 

n1 th observation 2

OSWAAL CUET (UG ) 10 Mock Test Papers GENERAL TEST

6 8

51 th 2

= 8 3= rdobservation  x = 8 × 3 = 24

20  25 100

 20  25 

= 50% Single equivalent % increase if 50%, 30% 50  30  50  30  100 = 95%

x 3

24. The minimum number of colours to required paint all sides of a cube that no two adjacent faces may have the same colour is. (B) 4 (A) 5 (D) 6 (C) 3 Ans. Option (C) is correct.

28. Find out which of the figures out of given option can be formed from the pieces given in fig (X)

Explanation: Opposite face has the same colour and there are six faces in a cube so we need 3 different colours to do the need full. 25. What sum of money will amount to ` 520 in 5 years and to ` 568 in 7 years on simple interest? (B) ` 120 (A) ` 400 (C) ` 510 (D) ` 220 Ans. Option (A) is correct. Explanation: A = P + SI P×R×5 ...(1) 520 = P + 100 PRT 568  P  100 2 PR 568  520  100 48 PR = 2 100 Now,put in equation 1 520 = P + 24 × 5 P = 520 - 120 = 400

Fig X (A)

Ans. Option (B) is correct. Explanation:

Fig X

Logic: Follow the pattern and the symmetry. Hence,

Explanation: Code for India is Prot. Hence, code for All Radio should be different from the codes given in the question. Hence, All India Radio-Kud Prot nid

Explanation: Single equivalent % increase if 20%, 25% ab

ab 100

(B)

(C)

26. Here are some words translated from an artificial language Holo polo means base ball Moto prot means my India Prot shot means India won Which word could be mean “All India Radio” (A) Holo polo prot (B) Kud prot nid (C) Prot polo nid (D) Polo nid prot Ans. Option (C) is correct.

27. Find a single equivalent increase if the number is successively increased by 20 %, 25 % and 30%? (A) 75 % (B) 85 % (C) 95 % (D) 35 % Ans. Option (C) is correct.



is correct answer. 29. Match List I with List II LIST I Physical Quantity A. B. C. D.

Electric charge Force Power Energy

LIST II Units I. II. III. IV.

Newton Coulamb Joule Watt

7

CUET(UG) Solved Paper 2023 (21st May Shift -1)

Choose the most appropriate answer from the options given below: (A) A-I, B-II, C-IV, D-III (B) A-II, B-I, C-IV, D-III (C) A-II, B-IV, C-I, D-III (D) A-III, B-II, C-IV, D-I Ans. Option (B) is correct. Explanation: The SI unit of electric charge is Coulomb and is represented by the symbol ‘C’. Coulomb is named after French physicist Charles-Augustin de Coulomb. 1C is approximately equal to the charge of 6.242 × 1018 protons or electrons. The SI unit of Force is Newton and is represented by N. It is defined in the Second Law of Motion and is named after Issac Newton. 1N is defined as the force required to accelerate 1 kilogram of mass at the rate of one meter per second in the direction of applied force. The SI unit of Power is Watt and is named after James Watt. 1W is the power produced when 1 Joule of work is done for 1 second. The SI unit of Energy is Joule and is named after the English physicist James Prescott Joule. 1 Joule is the amount of work done when a force of 1 newton displaces a mass through a distance of 1 metre in the direction of the force applied. 30. In a row of 40 children, A is 13th from the left end and B is ninth from the right end. How many children are there between A and C if C is forth to the left of B. (A) 13 (B) 14 (C) 15 (D) 16 Ans. Option (B) is the correct. Explanation: Left

12

A

14

13 from left th

C

B 8

The entry-level rank for the Indian navy is SubLieutenant. The Person then gets promoted to Lieutenant and then to Lieutenant Commander. 32. Statement I : Constitution is the frame work for the governance of a country which delegates power and authority to the executive, legislative and judiciary. Statement II : It serves a country in maintaining good relationships with her neighboring countries. In the light of the above statements, choose the most appropriate answer from the options below. (A) Both Statement I and Statement II are correct (B) Both Statement I and Statement II are incorrect (C) Statement I is correct but Statement II is incorrect (D) Statement I is incorrect but Statement II is correct Ans. Option (C) is the correct. Explanation: Constitution provides the framework for governance and delegates the power and authority to the judiciary, executive, and legislative of the country. It outlines the rights and duties of a citizen in order to build a prosperous nation. 33. How many terms are there in the A.P. 3, 7, 11, ... 407? (A) 100 (B) 101 (C) 99 (D) 102 Ans. Option (D) is correct. Explanation: 3, 7, 11…………………407 = an 407 = ,a 3 d  a2  a1  7  3  4 an  a   n  1  d

Right

13 9 from right th

th

407  3   n  1 4

40 Hence,14 is correct answer.

31. Major is related to Leiutenant in the same way as ‘Squadron Leader is Related to? (A) Pilot officer (B) Flying Attendant (C) Group captain (D) Flying officer Ans. Option (D) is the correct. Explanation: Major is related to Lieutenant in the same way as Squadron leader is related to Flying Officer. A lieutenant is an entry-level rank for the commissioned officer in the Indian army. The person then gets promoted to Captain and then to Major. Similarly, the entry-level rank for the Indian air force is Flying officer. The person then gets promoted to Flight Lieutenant and then to Squadron Leader.

407  3   n  1 4 404  n1 4 101 + 1 = n n = 102 34. What is the probability that any non-leap year will have 53 Sundays? (A)

1 53

1 7 Ans. Option (C) is correct. (C)

(B)

2 53

(D)

2 7

Explanation: As a result, there are 52 Sundays in a non-leap year. But one leftover day apart from those 52 weeks can be either a Monday, Tuesday,

OSWAAL CUET (UG ) 10 Mock Test Papers GENERAL TEST

8

Wednesday, Thursday, Friday, Saturday, or a Sunday. Therefore, the probability of getting 53 Sundays in a non-leap year is 1/7. 35. Match List I with List II LIST I A.

Blue Revolution

LIST II I.

Increase in crop yield and Agricultural Products

B.

White Revolution

II.

C.

Yellow Revolution

III. Increase of Fish Production

37. A man walks 2 km towards East and then he turns to South and walks 6 km. Again he turns to East and walks 4 km, after this he turns to North and walk 14 km. How far is he from his starting point? (B) 15 km (A) 10 km (C) 20 km (D) 25 km Ans. Option (A) is correct. Explanation: C

Increase in Oil-Seeds Production

Green IV. Increase in the field of Revolution milk production Choose the most appropriate answer from the options given below: (A) A-IV, B-III, C-II, D-I (B) A-III, B-IV, C-II, D-I (C) A-I, B-III, C-II, D-IV (D) A-III, B-I, C-II, D-IV Ans. Option (B) is the correct.

8 km 4 km

Start A

2 km

D.

Explanation: Blue Revolution is the improvisation, domestication, production and cultivation of fish, aquatic plants and animals in order to get economic benefits from the same. It was also known as Neel Kranti Mission and was launched by Dr. Hiralal Chaudhari and Dr. Arun Krishnsan in 1985. White Revolution focused on increasing the milk production of the country. It is also known as Operation Flood and was launched by Dr Verghese Kurien, who is also known as the father of white revolution. Yellow Revolution was launched to increase the production of edible oil in the country, especially from mustard and sesame seeds in 1986. Sam Pitroda is known as the father of the Yellow Revolution in India. Green Revolution was launched to increase crop yield and agricultural products in India. It led to an increase in the production of food grains especially wheat and rice. MS Swaminathan is known as the father of Green Revolution in India. 36. A bag contains 5 black, 3 white and 2 red balls. Three balls are drawn in suecession. What is the probability that the first ball is red, the second ball is black and the third ball is white? 3 1 (A) (B) 10 24 1 1 (C) (D) 2 10 Ans. Option (A) is correct. Explanation: E: event that first ball is red, second ball is black and 3rd ball is white. 2 5 3 1 P  E     10 9 8 24

End

6

B 6 km

4 km In Δ ABC,

 AC 

2

 8  6 2

2

= 64 + 36 = 100

= AC

= 100 10 km

38. Find the perimeter of a rhombus whose one diagonal is 16 cm long and area is 240 cm2. (A) 68 cm (C) 24 cm Ans. Option (A) is correct.

(B) 30 cm (D) 36 cm

Explanation: Areaof  R hom bus  240 

1  d1  d2 2

1  16  d2 2

d2 = 30 cm In rhombus diagonal bisect each other  2 2 sideof    8   15    64  225   r hom bus  289  17



Perimeter of Rhombus = 4 × 17 = 68 cm 39. From the figure, what is the value of x? A 50 x B (A) 50° (C) 60° Ans. Option (D) is correct.

120 C

D

(B) 120° (D) 70°

9

CUET(UG) Solved Paper 2023 (21st May Shift -1)

Ans. Option (C) is the correct.

Explanation: A 50˚

x

120˚

B C D Exterior angle of a triangle = Sum of opposite interior angles 120  50  x x  120  50  70

40. Consider the Diagram. 500 Candidates appeared in an Examination comprising test in English, Hindi and Maths. The Diagram gives number of students who failed in different tests. What is the % percentage of student who failed at least two subjects? English

Hindi 30

75

10

Explanation: The Major constituent of natural gas is Methane. Natural gas is a fossil fuel energy source. It is composed mainly of methane (CH4), smaller amounts of natural gas liquids (NGLs, which are also hydrocarbon gas liquids), and nonhydrocarbon gases, such as carbon dioxide and water vapor. It is formed from the remains of plants and animals that have been buried for millions of years under the surface of earth. These mix with sand, calcium carbonate, silt and change into natural gas as a result of high temperature and pressure. 42. Statements. I. Some cars are black II. Some Lions are cars Conclusion I. Some blacks are Lions II. No black is Lion (A) Only I follow (B) Only II follow (C) Either I or II follows (D) None follows Ans. Option (C) is correct. Explanation:

5 12

Lion

12

Black

Since there is no definite relation between lion and Black. Hence, either some blacks are lions or no black is lion

50

43. Match List I with List II LIST-I Books

Math (A) 6.8% (B) 7.8 % (C) 1.0 % (D) 0.078 % Ans. Option (B) is correct. Explanation: English

Hindi 30

Car

75

10 5 12

12

LIST-II Authors

A. India wins freedom

I.

B.

EL Mahatma Gandhi

The guide

R K Naravan

C. India from midnight III Abdul Kalam to Millenium Azad D. Conquest of self

IV. Shashi Tharoor

Choose the most appropriate answer from the options given below: (A) A-I, B-III, C-IV, D-II (B) A-III, B-I, C-IV, D-II (C) A-III, B-I, C-II, D-IV

50 Math

Number of students who failed in at least 39 × 100 = 7.8% subjects = 500 41. Major Constituent of natural gas is. (A) Propane (B) Butane (C) Methane (D) Carbon

(D) A-I, B-IV, C-II, D-III Ans. Option (B) is the correct. Books

Authors

India Wins Freedom

Maulana Azad

The Guide

R. K. Narayan.

India from Midnight Shashi Tharoor to Millennium Conquest of self

Mahatama Gandhi

OSWAAL CUET (UG ) 10 Mock Test Papers GENERAL TEST

10

Ans. Option (C) is correct.

44. Match List I with List II LIST-I Deficiency

LIST-II Diseases

A. Insulin

I.

Kwashiorka

B. Protein

II.

Scurvy

C. Thyroxin

III. Diabetes

D. Vitamin C

IV

(C) A-II, B-I, C-IV, D-III (D) A-IV, B-III, C-II, D-I Ans. Option (A) is the correct. Diseases

Doctors

1 5 6 2

Professor

3

4 Married People 1 number indicates doctors who are not married.

Goitre

Choose the correct answer from the options given below: (A) A-III, B-I, C-IV, D-II (B) A-I, B-III, C-IV, D-II

Deficiency

Explanation:

47. The average of 12 numbers is 15 and the average of the first two numbers is 14. What is the average of the remaining numbers? (A) 15 (B) 15.2 (C) 14 (D) 14.2 Ans. Option (B) is correct. Explanation:

Insulin

Diabetes

Protein

Kwashiorkor

Average =

Thyroxin

Goitre

Vitamin C

Scurvy

Sum of first 2 numbers = 14 × 2 = 28 28  remaining number sum 15  12 Remaining number sum = 15 × 12 - 28 = 152 152 152 Average remaining    15.2 12  2 10

45. Find the angle of elevation of the Sun, when the 1 times the length of the shadow of a tree is 3 height of the tree. (A) 30° (B) 45° (C) 60° (D) 90° Ans. Option (C) is correct.

48. Match List-I with List-II

Explanation: Sun Tree

sum of all numbers  Total number of all numbers

A. B.

LIST-I Scientists Har Gobind Khorana C.V Raman

I. II.

x  Tree shadow 1 x 3  1  tan   x /  x  3  ⇒ tan    3 ⇒ tan   tan60 ⇒   60

46. In figure out which Number indicate doctors who are not married (A) 2 (B) 4 (C) 1 (D) 6

C. D.

Jagdish Chandra III. Bose Aryabhata IV

LIST-II Discoveries Discovery of Zero Genetic composition of cell Scattering of light Measurement of plant growth

Choose the most appropriate answer from the options given below: (A) A-II, B-III, C-I, D-IV (B) A-III, B-II, C-IV, D-I (C) A-II, B-III, C-IV, D-I (D) A-I, B-III, C-II, D-IV Ans. Option (C) is the correct. Scientists

Discoveries

Har Gobind Khorana

Genetic composition of cell

CV Raman

Scattering of Light

Jagdish Chandra Bose

Measurement of Plant growth

Aryabhatta

Discovery of Zero

11

CUET(UG) Solved Paper 2023 (21st May Shift -1)

49. Bhoodan-Gram Dan Movement was initiated by. (A) Mahatma Gandhi

Cities

(B) Vinoba Bhave (C) Shri Ram Chandra Reddy (D) Sardar Patel Ans. Option (B) is the correct. Explanation: Bhoodan Gram Dan Movement or Land Gift Movement was initiated by Acharya Vinoba Bhave. It was a voluntary land reform movement initiated at Pochampally village, in the then Andhra Pradesh. It was launched in 1951 and as a part of this movement, wealthy land owners voluntarily gave some percentage of their land to landless people. These landless labourers could use this and for growing crops or settling down on the same. They were not allowed to resell it or use it for any commercial activity. 50. The ratio of ages of 2 boys is 3: 7. After 2 years, the ratio of their ages will become 5: 9. The ratio of their ages after 10 years will be (A) 15 : 16 (B) 5 : 17 (C) 17 : 18 (D) 13 : 17 Ans. Option (D) is correct. Explanation: Let the age of first boy = 3x Let the age of second boy = 7x After 2 years 3x  2 5  7x  2 9 27x + 18 = 35x + 10 8x = 8 x=1 Ratio of their ages after 10 years 

Ans. Option (A) is the correct.

3x  10 3  1  10 13    13 : 17 7 x  10 7  1  10 17

51. Match List I with List II Cities with their nicknames LIST I Cities

LIST II Nickname

A.

Nagpur

I.

Diamond City

B.

Sural

II.

Pink City

C.

Jaisalmer

III. Orange City

D.

Jaipur

IV.

Golden City

Choose the most appropriate answer from the options given below: (A) A-III, B-I, C-IV, D-II (B) A-I, B-III, C-IV, D-II (C) A-III, B-I, C-II, D-IV (D) A-II, B-I, C-III, D-IV

Nickname

Nagpur

Orange City

Surat

Diamond City

Jaisalmer

Golden city

Jaipur

Pink City

52. Which of the following is not a ‘state’? (A) Nagaland (B) Manipur (C) Laddakh (D) Meghalaya Ans. Option (C) is the correct. Explanation: Ladakh is not a State. It is a Union Territory and became one on 31 October 2019 as a part of the Indian Government’s Jammu and Kashmir Reorganisation Act 2019. The erstwhile Jammu and Kashmir was stripped of its special status and reconstituted into two Union Territories – Ladakh without legislature and Jammu and Kashmir with the legislature. The first Lt. Governor of Ladakh Union Territory was RK Mathur. This is for the first time in the history of India that a state is being converted into two Union Territories. India has 28 states and 9 union territories. 31 October is observed as the ‘National Unity Day’ to mark the birth anniversary of the country’s first home minister Sardar Vallabhbhai Patel. 53. Find the angle traced by hour hand of a correct clock between 7 pm o’ clock and 2 am o’ clock. (A) 200° (B) 210° (C) 310° (D) 290° Ans. Option (B) is correct. Explanation: Angle made by hour hand in 1 hour = 30° 7 pm to 2 am = 7 hours. Angle traced = 30° × 7 = 210° 54. Find the angle of elevation of the Sun, when the length of the shadow of a tree is 3 times the height of the tree. (A) 30° (B) 45° (C) 60° (D) 90°

OSWAAL CUET (UG ) 10 Mock Test Papers GENERAL TEST

12

Ans. Option (B) is correct.

Ans. Option (A) is correct. Explanation: Sun Tree x  Tree shadow  3 x tan   x / ⇒ tanθ =



3x



1

3 ⇒ tan   tan30

⇒   30 55. Which of the following is not the satellite launched by India? (A) Explorer (B) APPLE (C) Bhaskar (D) INSAT Ans. Option (A) is the correct. Explanation: Explorer 1 was the first successful satellite launched by the United States on 31 January 1958. It was launched as a response to Soviet Union’s Sputnik 1. It is credited for the discovery of radiation belts around Earth held in place by the planet’s magnetic field. These were later named as Van Allen belts in honor of their discoverer. APPLE stands for Ariane Passenger Payload Experiment. It was India’s first communication satellite launched by ISRO on 19 June 1981. The satellite was carried on a non-metallic bullock cart to avoid reflections off the metal plates, affecting the satellite’s antenna. Bhaskara I was the first experimental remote sensing satellite built by ISRO and named after Indian mathematician Bhaskara. The satellite was launched to collect data on hydrology, oceanography, forestry and telemetry. It was launched from Volgograd Launch Station, Russia. INSAT is Indian National Satellite System. It is a space-based satellite system that provides television, communication, and meteorological services to India and its neighbouring countries. 56. The area of a circle is numerically equal to its circumference. Find the diameter of the circle. (A) 2 unit (B) 4 unit (C) 1 unit (D) 5 unit

Explanation: Area of circle = πr2 Circumference of circle = 2πr According to question πr2 = 2πr r=2 Hence, diameter = 2r = 2 × 2 = 4 units 57. Find the next term in the alpha-numeric series D4T, F9R, H20P,  J43N. (A) L 90 M (B) N 90 N (D) J 90 L (C) L 90 L Ans. Option (C) is correct. Explanation: D4T, F9R, H20P,  J43N. Logic: D  2  F, F  2  H , H  2  J , J  2  L T  2  R , R  2  P, P  2  N , N  2  L 4 × 2 + 1 → 9 , 9 × 2 + 2 → 20 , 20 × 2 + 3 → 43, 43 × 2 + 4 → 90 Hence, L90L is the correct answer. 58. Match List I with List II LIST I LIST II Diet deficiency Disease A. Deficiency of I. Anaemia Vitamin B B. Deficiency of II. Beri-Beri Vitamin A C. Deficiency of Iron III. Goitre D. Deficiency of IV Nisht blindness Iodine Choose the most appropriate answer from the options given below: (A) A-IV, B-II, C-III, D-I (B) A-II, B-IV, C-I, D-III (C) A-I, B-III, C-IV, D-II (D) A-III, B-I, C-II, D-IV Ans. Option (B) is the correct. Explanation: Diet Deficiency

Disease

Deficiency of Vitamin B

Beri Beri

Deficiency of Vitamin A

Night Blindness

Deficiency of Iron

Anaemia

Deficiency of Iodine

Goitre

59. A and B can do a work in 9 days and 12 days respectively. If they work on alternate days starting with A, then in how many days will the work be completed? (A) 36 days (B) 10 days 1 (D) 13 days (C) 10 days 4

CUET(UG) Solved Paper 2023 (21st May Shift -1)

Ans. Option (C) is correct. Explanation: A 9

B 12

days 4

3 (efficiency)

(Total work =LCM(9, 12)) 36 So, if they work on alternate days for two days, they will complete a total of 4 + 3 = 7 units of work. Therefore, if they work on alternate days for 10 days, they will complete 7 × 5 = 35 units of work. The remaining work will be 36 - 35 = 1 unit. Time taken by A to finish the remaining 1 work = 4 1 1 So, total time 10   10 days 4 4

13 60. If today is Saturday then what will be the day on 363rd day? (A) Sunday (B) Monday (C) Thursday (D) Friday Ans. Option (D) is the correct. Explanation: Considering that there are 7 days in a week. Since 363 is not an exact multiple of 7, we need to find the remainder when dividing 363 by 7. 363 ÷ 7 = 51 with a remainder of 6. This means that after 51 weeks, there will be 6 days left. Adding these 6 days to Saturday, we get: Saturday + 6 days = Friday So, the day on the 363rd day will be Friday.



OSWAAL CUET (UG ) 10 Mock Test Papers GENERAL TEST

14

MOCK

1

Test Paper Maximum Marks : 250

Time : 60 Minutes

General Instructions : (i) This paper consists of 60 MCQs, attempt any 50 out of 60 (ii) Correct answer or the most appropriate answer: Five marks (+5) (iii) Any incorrect option marked will be given minus one mark (-1) (iv) Unanswered/Marked for Review will be given no mark (0) (v) If more than one option is found to be correct then Five marks (+5) will be awarded to only those who have marked any of the correct options (vi) If all options are found to be correct then Five marks (+5) will be awarded to all those who have attempted the question. (vii) Calculator / any electronic gadgets are not permitted.

1. Which of the following is not correctly matched? Sikh Guru: Contribution (1) Guru Nanak Dev Ji: Contemporary of Mughal emperor Babur. (2) Guru Arjan Dev Ji: Compiled the Adi Granth (the primary scripture of Sikhism) and completed the construction of the Harmandir Sahib (Golden Temple). (3) Guru Amardas Sahib Ji: Introduced the concept of “Langar” (community kitchen). (4) Guru Ram Das Ji: Founded the city of Amritsar and designed the Harmandir Sahib (Golden Temple). 2. Fertile riverine alluvial soil is best suited for producing _________. (1) Rice (2) Tea (3) Cotton (4) Corn 3. Andaman and Nicobar Islands is a group of ______ islands/islets? (1) 275 (2) 450 (3) 780 (4) 572 4. Match the following International boundaries with their countries. International Countries Boundaries 1. 17th Parallel A. Egypt and Sudan 2. 38th Parallel B. Pakistan and Afghanistan 3. 22nd Parallel C. South Vietnam and North Vietnam 4. Durand Line D. South Korea and North Korea (1) 1 - A, 2 - B, 3 - C, 4 -D (2) 1 - C, 2 - D, 3 - A, 4 - B (3) 1 - C, 2 - D, 3 - B, 4 - A (4) 1 - C, 2 - A, 3 - B, 4 - D

5. In which part of the Indian Constitution are centrestate relations mentioned? (1) Part IV (Article 227 to 234) (2) Part XI (Article 245 to 255) (3) Part X (Article 234 to 240) (4) Part XII (Article 265 to 277)

6. Manipur, Meghalaya and Tripura became states under _____________. (1) North Eastern Retention (Reconstruc­tion) Act, 1971 (2) North Eastern Republic of India Act, 1972 (3) North Eastern Region New State Act, 1972 (4) North-Eastern Areas (Reorganisation) Act, 1971

7. The amount of light entering into eye can be controlled and regulated by _____________. (1) Iris

(2) Cornea (3) Pupil

(4) Retina

8. Which of the following is not correctly matched? (1) Treaty of Madras 1769: The Treaty of Madras was signed between the British and Hyder Ali of Mysore to bring to an end the first Mysore War. (2) Treaty of Wadgaon 1779: Signed between the British and the Marathas bringing the second phase of the First Anglo-Maratha War to an end. (3) Treaty of Salbai 1782: Signed between the British and the Marathas bringing the First Anglo-Maratha War to an end. (4) Treaty of Seringapatam 1792: Signed between the British and the Marathas bringing the first phase of the First Anglo-Maratha War to an end.

MOCK TEST Paper-1

15

9. Which of the following pair is correctly matched? (1) First Five-Year Plan (1951-1956): It is based on Mahalanobis Model. (2) Second Five-Year Plan (1956-1961): It is based on Harrod-Domar Model. (3) Fourth Five-Year Plan (1969-1974): It is based on Ashok Rudra Alon Model. (4) Fifth Five-Year Plan (1974-1979): It is based on Gadgil Model.

10. Which of the following is/are correctly matched? International organization

Headquarters

1. UNICEF



New York

2. World Bank



Washington DC

Geneva 3. World Trade Organization Choose the correct option. (1) 1 only

(2) 2 only

(3) 3 only

(4) 1, 2 and 3

11. The Aligarh Movement was started by _________ . (1) Dr. Maghroor Ahmad Ajazi (2) Muhammad Ali Jinnah (3) Maulana Manzoor Ahsan (4) Syed Ahmed Khan

12. Which of the following is not a computer language? I. C++

17. Which one of the following is NOT a natural fiber? (1) Terylene (2) Jute

(4) Wool

18. Which of the following is good insulator? (1) Mercury

(2) Cadmium

(3) Germanium

(4) Glass

19. In which nation did the Indian team achieve its inaugural bronze medal in the ‘Asian Surfing Championship’? (1) Malaysia

(2) Russia

(3) Greece

(4) Maldives

20. The Biraja Temple, the Rajarani Temple and the Samaleswari Temple are all located in ____________ (1) Odisha

(2) Assam

(3) Tamil Nadu

(4) Kerala

21. If the 8-digit number 789x531y is divisible by 72. Then the value of (5x – 3y) is: (1) 0

(2) –1

(3) 2

(4) 1

22. The angle of elevation at the top of an unfinished tower at a point distant 78 m from its base is 30°. How much higher does the tower be raised (in m) so that the angle of elevation of the top of the finished tower at the same point will be 60°? (1) 78 3 (2) 80 (3) 52 3 (4) 26 3 23. The arithmetic mean of the following numbers 1, 2, 2, 3, 3, 3, 4, 4, 4, 4, 5, 5, 5, 5, 5, 6, 6, 6, 6, 6, 6 and 7, 7, 7, 7, 7, 7, 7, is (1) 4

II. Java

(3) Flax

(2) 5

(3) 14

(4) 20

24. A dealer sells a machine having marked price as

III. Linux (1) Only I

(2) Only III

(3) II and III

(4) I and II

13. Which of the following individuals or organizations was not honoured with the Nobel Peace Prize in 2022? (1) Ales Bialiatski (2) Russian human rights organisation Memorial (3) Maria Ressa (4) Ukrainian human rights organisation Center for Civil Liberties

14. Surendranath Banerjee and Ananda Mohan Bose founded ___________ in Bengal in 1876. (1) Bengal Association

` 3840 at a discount of 20%. What is the selling price (in `) of the machine? (1) 3072

(2) 3500

(3) 4608

(4) 3240

25. Two pipes of length 1.5 m and 1.2 m are to be cut into equal pieces without leaving any extra length of pipes. The greatest length of the pipe pieces of same size which can be cut from these two lengths will be (1) 0.13 metre

(2) 0.4 metre

(3) 0.3 metre

(4) 0.41 metre

26. If the simple interest on Re. 1 for 1 month is 1 paisa, then the rate per cent per annum will be (1) 10%

(2) 8%

(3) 12%

(4) 6%

27. Two numbers whose sum is 84 can not be in the

(2) Indian Association

ratio

(3) Tathagat Association

(1) 5 : 7

(4) Oriental Association

15. The rover of the Chandrayaan 3 mission is referred to as: (1) Vikram

(2) Bheem

(3) Pragyaan

(4) Dhruv

16. Which of the following is the study of soil? (1) Mineralogy

(2) Pedology

(3) Glaciology

(4) Cosmology

(2) 13 : 8

28. If m – n = 16 and

m2

+

(3) 1 : 3 n2

(4) 3 : 2

= 400, the value of mn

is = (1) 72

(2) 25

(3) 144

(4) 192

29. A boat moves downstream at the rate of 8 km per hour and upstream at 4 km per hour. The speed of the boat in still waters is: (1) 4.5 km per hour

(2) 5 km per hour

(3) 6 km per hour

(4) 6.4 km per hour

OSWAAL CUET (UG ) 10 Mock Test Papers GENERAL TEST

16

30. A takes three times as long as B and C together

to do a job. B takes four times as long as A and C together to do the work. If all the three, working together can complete the job in 24 days, then the number of days. A alone will take to finish the job is

(1) 100

(2) 96

(3) 95

(4) 90

31. If p, q, r are in Geometric Progression, then which is true among the following?

(1) q 

pr 2

(2) p2 = qr p r (4) = r q

(3) q = pr

32. ABC is an isosceles triangle where AB = AC which is circumscribed about a circle. If P is the point where the circle touches the side BC, then which of the following is true? (1) BP = PC (2) BP > PC (3) BP < PC

(4) BP =

1 PC 2

33. In an isosceles triangle ABC, AB = AC, XY || BC. If ∠A = 30°, then ∠BXY = ?

(1) 75°

(2) 30°

(3) 150°

(4) 105°

39. Two trains leave Varanasi for Lucknow at 11:00 AM

and at 11:30 AM respectively and travel at speeds of 110 km/h and 140 km/h, respectively. How many kilometres from Varanasi will both trains meet?

(1) 256 2 km 3 1 (3) 246 km 3

(2) 255 1 km 3 (4) 238 2 km 3 40. In how many years will a sum of ` 800 at 10% per annum compounded semi-annually become ` 926.10? (1) 2 1 years 2 (3) 2 years

(4) 1 1 years 2 41. Select the option in which the words share the same relationship as that shared by the given pair of words. Dentist : Doctor (1) Biology : Astrology (2) Chemistry : Science (3) Algebra : Geometry (4) Line : Circle

the following data. Refer the following data table and answer the question. 15

Dreaming

15

Light sleep

5

Extremely light sleep

30

Awake

35



(2) 2.5 hours

(3) 4.5 hours

(4) 4 hours

(1) 30th





(1) 7 : 5



36. If x  3  2 2 ,then x + (1) 0

(2) 1

1 x

(4) 1 : 2

is equal to

(3) 2

(4) 2 2

37. The radius of the incircle of an equilateral DABC of 1 1 (2) (3) 1 (4) 3 3 2 38. If diagonals of a rhombus are 16 cm and 30 cm. What is the perimeter (in cm) of the rhombus? (1) 32

(2) 64

(3) 34

(4) 68

(4) 34th

44. If the minute hand of a clock points towards the

45. Select the option in which the number set shares (17, 24, 45) (1) (19, 26, 34) (3) (15, 20, 43)

(2) (18, 23, 46) (4) (12, 19, 40)

46. In a code language ‘BLOCK’ is written as ‘31316412’.



How will ‘SUPREME’ be written in that language? (1) 202217196146 (2) 192116185605 (3) 202317206146 (4) 192217196156



47. Select the Venn diagram that best illustrates the relationship among the following classes: Uncle, Relatives, Rich

side 2 3 units is x cm. The value of x is: (1)

(3) 33rd

the same relationship as that shared by the given number set.

the ratio 4 : 3 and 2 : 3 respectively. In what ratio, the liquids in both the vessels be mixed to obtain a new mixture in vessel C containing half milk and half water? (3) 3 : 11

(2) 32nd

South at 12 : 00 a.m., then in which direction will the minute hand point at 3 : 00 a.m. in the same clock? (1) North (2) South (3) West (4) East

35. The milk and water in two vessels A and B are in

(2) 5 : 2

43. In a class of 42 students, Swati’s rank is 19th from the

bottom. Purshottam is 6 ranks below Swati. What is Purshottam’s rank from the top?

How long was the user in Deep Sleep or was Awake? (1) 3.5 hours

42. In a code language, if ‘I like chocolates’ is written as ‘958’, ‘we bought chocolates’ is written as ‘153’ and ‘we like them’ is written as ‘816’, then how would ‘I bought them’ be written in this language? (1) 859 (2) 251 (3) 936 (4) 951

34. Between 10 pm and 6 am, a fitness band records

Deep sleep

(2) 3 years



(1)

(2)





MOCK TEST Paper-1

17

(3)



(4)





48. Read the given statements and conclusions carefully. Assuming that the information given in the statements is true, even if it appears to be at variance with commonly known facts, decide which of the given conclusions logically follow(s) from the statements.

Statements:

I. All polygons are angles.



II. All angles are diagonals.



III. All cones are cubes.



(1) Only Conclusion II follows



(2) Conclusion I and II both follow



(3) Neither I nor II follow



(4) Only Conclusion I follows



52. Select the correct option that is related to the third term in the same way as the second term is related to the first term.

Television : Telecast :: Radio : ?

(1) Broadcast

(2) Friendship



(3) Compose

(4) Talking



53. Mohini is taller than Nita but not shorter than Sarita. Sarita and Malini are of same height. Mohini is shorter than Hema. Among them, who is the second tallest?



IV. All cubes are decagons.



(1) Mohini



V. No diagonal is a cube.



(2) Nita



(3) Hema

Conclusions:

I. Some diagonals are polygons.





II. All diagonals are decagons.





III. No polygon is a cone.



IV. Some cubes are angles.



(1) Bothe conclusions II and IV follow.



(2) Only conclusion I follows.



(3) Both conclusions I and II follow.



(4) Both conclusions I and III follow.

two children. Manideep is the brother of Niharika. Niharika is the daughter of Kavita. Ananya is the granddaughter of Kavita. Sujit is the father of Ananya. How is Sujit related to Manideep?

49. Which number will follow next in the given series? 3, 16, 29, 42, ? (1) 57

(2) 55

(3) 54

(4) 61

50. Six friends A, B, C, D, E and F are sitting around a round table facing the centre. C is seated between A and B, B is in immediate left of D. If C is facing E, who is facing F? (1) E



(2) A

(3) D

(4) Cannot be determined

54. Ramesh is the father of Manideep. Ramesh has only



(1) Soni

(2) Brother



(3) Brother-in-law

(4) Son-in-law



to make the given equation correct? 8 ÷ 2 + 4 = 10

(1) ÷ and +, 8 and 4

(2) ÷ and +



(3) + and ÷, 8 and 2

(4) + and ÷, 2 and 4



51. In the following question, two statements are

Statements:

(I)  Major companies are nowadays surviving because of the strong supply chain system.



(II)  Supply chain comprises of logistics, retailers and distributors.

Conclusions:

(I) Retailers play most important role because they interact with customers on first hand.



(II)  If an organization has strong logistics, then product can easily be available to the customers in such an era of competition.

56. From a point D Sarita Walked 50 metres to the North, then after turning to right walked 50 metres. She again turned right and walked another 70 metres and finally turned to right and walked 50 metres. In which direction and at what distance is she from the starting point D?

(4) B

given each followed by two conclusions I and II. You have to consider the statements to be true even if they seem to be at variance from commonly known facts. You have to decide which of the given conclusions, if any, follows from the given statements.

55. Which two signs/numbers need to be interchanged



(1) 20 metres to the North



(2) 40 metres to the South



(3) 20 metres to the South



(4) 30 metres to the South



57. Pointing to a lady, Diwakar said, “Her mother’s only grandson is my son.” How is that lady related to Diwakar?



(1) Aunty

(2) Sister



(3) Mother

(4) Wife



58. Question figure

OSWAAL CUET (UG ) 10 Mock Test Papers GENERAL TEST

18 Answer figures (2)

(1)











60. Select the term that will come next in the following

(4)

(3)



and walks another 10 km. He takes another right turn and walks 10 km to reach his school. How far (in km) and in which direction is he from his home? (1) 10, North (2) 10, South (3) 20, North-East (4) 20, South-West





series. YCL, MQZ, ......, OSB



59. Kamal starts walking from his home facing West direction. After walking 10 km he takes a right turn



(1) AEN

(2) BFQ



(3) ADM

(4) BEM

 Answer Key 1.

(3)

2.

(1)

3.

(4)

4.

(3)

5.

(2)

6.

(4)

7.

(1)

8.

(4)

9.

(3)

10.

(4)

11.

(4)

12.

(2)

13.

(3)

14.

(2)

15.

(3)

16.

(2)

17.

(1)

18.

(4)

19.

(3)

20.

(1)

21.

(2)

22.

(3)

23.

(2)

24.

(1)

25.

(3)

26.

(3)

27.

(4)

28.

(1)

29.

(3)

30.

(2)

31.

(3)

32.

(1)

33.

(4)

34.

(4)

35.

(1)

36.

(4)

37.

(3)

38.

(4)

39.

(1)

40.

(4)

41.

(2)

42.

(3)

43.

(1)

44.

(2)

45.

(4)

46.

(1)

47.

(4)

48.

(4)

49.

(2)

50.

(4)

51.

(1)

52.

(1)

53.

(4)

54.

(3)

55.

(4)

56.

(3)

57.

(2)

58.

(2)

59.

(1)

60.

(1)

MOCK TEST Paper-1

19

MOCK

2

Test Paper Maximum Marks : 250

Time : 60 Minutes

General Instructions : (i) This paper consists of 60 MCQs, attempt any 50 out of 60 (ii) Correct answer or the most appropriate answer: Five marks (+5) (iii) Any incorrect option marked will be given minus one mark (-1) (iv) Unanswered/Marked for Review will be given no mark (0) (v) If more than one option is found to be correct then Five marks (+5) will be awarded to only those who have marked any of the correct options (vi) If all options are found to be correct then Five marks (+5) will be awarded to all those who have attempted the question. (vii) Calculator / any electronic gadgets are not permitted.

1. Which of the following is/are correctly matched with their respective states: 1. Somnath Temple: Gujarat 2. Kamakhya Temple: Assam 3. Ramnathaswamy Temple: Tamil Nadu 4. Mahabodhi Temple: Bihar Choose the correct option: (1) 1 and 2 only (2) 2 and 3 only (3) 1, 2 and 4 only (4) 1, 2, 3 and 4 2. Which of the following is/are the official language of the United nation? 1. Arabic 2. Hindi 3. English 4. Hebrew Choose the correct option: (1) 1 and 2 only (2) 1 and 3 only (3) 2 and 3 only (4) 1, 2, 3 and 4 3. Which of the following is/are correctly matched? Battles Year 1. Battle of Chausa (1) 1539 2. Battle of Saraighat (2) 1671 3. Battle of Saragarhi (3) 1897 4. The first battle of Panipat (4) 1526 Choose the correct option: (1) 1, 2 and 3 only (2) 2, 3 and 4 only (3) 1 and 4 only (4) 1, 2, 3 and 4 4. “Regarding TRIMS, which of the following statements is true? (1) Trade-Related Investment Measures (TRIMS) prohibit quantitative restrictions on imports by foreign investors. (2) They apply to investment measures related to trade in both goods and services. (3) Both 1 and 2 (4) Neither 1 nor 2

5. Who among the following was the last Nawab of Awadh? (1) Amjad Ali Khan

(2) Muhammad Mukim

(3) Wajid Ali Shah

(4) Saadat Ali Khan

6. Rajat has hypermetropia. What type of lens will the ophthalmologist recommend to correct his vision? (1) Bifocal

(2) Concave

(3) Progressive

(4) Convex

7. Which of the following is/are correctly matched? List I (Port)

List II (Feature)

1. Mumbai

Biggest port in India

2. Visakhapatnam

Landlocked harbour

3. Chennai

Oldest port on the Eastern Coast

4. Kochi

Queen of the Arabian Sea

Select the correct option: (1) 1 and 2 only

(2) 2 and 3 only

(3) 1, 2 and 4 only

(4) 1, 2, 3 and 4

8. Which among the following is not an input device? (1) Plotter (2) Magnetic Ink Character Recognition (MICR) (3) Optical Mark Recognition (OMR) (4) Barcode Reader

9. The largest invertebrate is a/an (1) Sponge

(2) Mollusc

(3) Arthropod

(4) Echinoderm

10. How does a Unicellular Organism reproduce? (1) Cell division

(2) Cell reproduction

(3) Cell synthesis

(4) Fragmentation

OSWAAL CUET (UG ) 10 Mock Test Papers GENERAL TEST

20

11. Which of the following is/are correctly matched?

19. Who invented vaccination for ‘Small Pox’?

1. Jnanpith Award: Literature

(1) Sir Fredrick Grant Banting

2. Pritzker Architecture Prize Architecture

(2) Sir Alexander Fleming

3. Grammy Award Outstanding achievements in the music industry

(3) Edward Jenner

4. Abel Prize Outstanding scientific work in the field of mathematics

(4) Louis Pasteur

20. Which city in India is world renowned for one of the most traditional embroidery styles, Chikankari?

Choose the correct option: (1) 1 and 4 only

(2) 2 and 3 only

(3) 1 and 3 only

(4) 1, 2, 3 and 4

12. In our country the ‘Van Mahotsav’ Day is observed on: (1) 10th August

(2) 1st July

(3) 2nd October

(4) 1st December

13. The first woman of Indian origin who undertook a space journey

(1) Ahmedabad

(2) Lucknow

(3) Hyderabad

(4) Udaipur

21. How much simple interest will ` 4000 can in

22.

18 months at 12% per annum? (1) ` 216 (2) ` 360 (3) ` 720 (4) ` 960 The remainder when 321 is divided by 5 is (1) 1

(2) 2

(3) 3

(4) 4

23. The marked price of a piano was `15,000. At the time of sale, there were successive discounts of 20%, 10% and 10% respectively on it. The sale price was

(1) Sunita Williams (2) Kalpana Chawla (3) Darshan Ranganath

(1) ` 9,720

(4) Avani Chaturvedi

14. Which Commission was appointed by the central government to examine issues related to CentreState relations? (1) Mandal Commission (2) Sarkaria Commission

(2) ` 9,750 (3) ` 9,760 (4) ` 9,780

24. Three numbers are in the ratio 3 : 4 : 5. The sum of the largest and the smallest equals the sum of the second and 52. The smallest number is (1) 20

(2) 27

(3) 39

(4) 52

25. A boat goes a certain distance at 30 km/hr and

(3) Nanavati Commission

comes back the same distance at 60 km/hr. What is the average speed (in km/hr) for the total journey?

(4) Kothari Commission

(1) 45

15. The three-tier system of local governance does not encompass the? (1) Panchayat Samiti

(2) Village Committee

(3) Gram Panchayat

(4) Zila Parishad

16. To neutralize too acidic soil ______________ is used.

(2) 50

(3) 40

(4) 35

26. 84 Maths books, 90 Physics books and 120 Chemistry books have to be stacked topic-wise. How many books will be there in each stack so that each stack will have the same height too? (1) 12

(2) 18

(3) 6

(4) 21

27. If the mean of the numbers 2, (2p + 2), 7, 13, 17, 4, and (p – 1) is 8, then find their median.

(1) Quick lime

(1) 4

(2) Slaked lime (3) Both quick and slaked lime (4) None Option Is correct

17. Which of the following primarily produces Nitrogen dioxide leading to air pollution? (1) CFL Lamp

(2) Automobile Battery

(3) Polymer

(4) Diesel Engine

18. Which of the following is/are correctly matched?

(2) 6

(3) 7

(4) 13

28. A can do 1 of a work in 10 days. B can do 1 of the

4 3 work in 20 days. In how many days can both A and B together do the work? (1) 30 days (2) 32 days (3) 24 days

(4) 25 days

29. Tickets numbered from 1 to 20 are mixed and a

(2) Hockey: Clapper

ticket is drawn at random. What is the probability that the ticket drawn bears a number which is a multiple of 3?

(3) Baseball: Duck

(1) 3/10

(1) Cricket: Pinch hitter

(4) Lawn Tennis: Deuce Choose the correct option: (1) 1 and 2 only

(2) 2 and 3 only

(3) 2 and 4 only

(4) 1, 2, 3 and 4

(2) 3/20

(3) 2/5

(4) 1/2

30. The sum of the series (1 + 0.6 + 0.06 + 0.006 + 0.0006 + ...) is

2 (1) 1 3

1 (2) 1 3

1 (3) 2 3

2 (4) 2 3

MOCK TEST Paper-2

21

31. In the figure (not drawn to scale) given below, if AD = DC = BC and ∠BCE = 96°, then ∠DBC is: E C

40. A rice trader buys 16 quintals of rice for ` 5632. 20% rice is lost in transportation. At what rate should he sell to earn 25% profit? (1) ` 225.2 per quintal (2) ` 550 per quintal

96°

(3) ` 440 per quintal

(4) ` 563.2 per quintal

41. Richa starts walking from her home and walks A

(1) 32°

D

B

(2) 84°

(3) 64°

(4) 96°

2

32. If (10.15) = 103.0225, then the value of 1.030225 + 10302.25 is (1) 1025·15

(2) 103·515 (3) 102·515 (4) 102·0515

33. Refer the following data table and answer the question.

50 m towards West and then turns left and walks 90 m. Then she again turns left and walks 30 m. After that she again turns left and goes 120 m. She finally turns right walks 20 m and reaches her final destination. How far is she from her home now and in which direction with reference to her home? (1) 40 m, North West

(2) 30 m, South

(3) 40 m, North East

(4) 30 m, North

Weight (kg)

Height (m)

Apeksha

59

1.8

was the day of 18th February 1999?

Apsara

51

1.72

(1) Friday

(2) Monday

Aradhana

62

1.57

(3) Tuesday

(4) Thursday

Archana

74

1.68

Who has the least weight to height ratio? (2) Apsara (1) Apeksha (3) Aradhana

(4) Archana

34. A bicycle wheel makes 5000 revolutions in moving

11 km. Then the radius of the wheel (in cm) is 22 ) (Take π = 7

(1) 70

(2) 35

(3) 17.5

(4) 140

35. What is the ratio of in-radius and circum-radius of an equilateral triangle?

(1) 1 : 2

(2) 1 : 3

(3) 1 : 4

(4) 3 : 2

36. In two blends of mixed tea, the ratios of Darjeeling and Assam tea are 4 : 7 and 2 : 5. The ratio in which these two blends should be mixed to get the ratio of Darjeeling and Assam tea in the new mixture as 6 : 13 is (1) 22 : 35

(2) 26 : 35 (3) 35 : 78 (4) 13 : 22

37. A train without stoppage travels with an average speed of 72 km/h and with stoppage, it travels with an average speed of 60 km/h. For how many minutes does the train stop on an average per hour? (1) 10

(2) 12

(3) 6

(4) 8

38. The compound interest on ` 64,000 for 3 years, compounded annually at 7.5% p.a. is (1) ` 14,400

(2) ` 15,705

(3) ` 15,507

(4) ` 15,075

43. In a certain code language. ‘AROUND’ is coded as ‘52182412144’ and ‘FIX’ is coded as ‘63624’. How will ‘PLASTIC’ be coded in that language? (1) 1612261920183

(2) 1612521920363

(3) 1812521920383

(4) 1612522021363

44. A # B’ means ‘A is the son of B’. ‘A @ B’ means ‘A is the mother of B’. “A & B’ means ‘A is the wife of B’. ‘A% B’ means ‘A is the sister of B’. If ‘M @ R % K # G # N & T’ which of the following statements is NOT correct? (1) N is paternal grandmother of K. (2) T is paternal grandfather of M. (3) M is the mother of K. (4) R is the daughter of G.

45. Select the option in which the numbers are related in the same way as are the numbers in the given set. (109, 114, 139) (1) (313, 318, 343)

(2) (579, 534, 549)

(3) (268, 302, 237)

(4) (419, 424, 439)

46. In a certain code language, ‘FRENCH’ is coded as ‘114’ and ‘LOSS’ is coded as ‘47’. How will ‘COURSE’ be coded in that language? (1) 81

(2) 120

Snakes, Reptiles, Poisonous (2)

(1) (3) 206

(4) 87

relationship among the following classes :

1 1 of 2 + 2 ? P Q (2) 194

(3) 103

47. Select the Venn diagram that best illustrates the

39. If P = 7 + 4 3 and PQ = 1, then what is the value

(1) 196

42. If 18th February 1997 fell on Tuesday, then what

(4) 182





OSWAAL CUET (UG ) 10 Mock Test Papers GENERAL TEST

22

54. Which number will appear on the face opposite (3)



to 1?

(4)



48. Read the given statements and conclusions carefully. Assuming that the information given in the statements is true, even if it appears to be at variance with commonly known facts, decide which of the given conclusions logically follow(s) from the statements. Statements: I. No bank is an office. II. All offices are stalls. Conclusions: I. No bank is a stall. II. No stall is a bank. III. Some stalls are offices. IV. All the stalls are offices. (1) Only conclusion I follows (2) Both conclusions I and II follow (3) Only conclusions III follows (4) Both conclusions I and IV follow

49. Select the option that will correctly replace the question mark (?) in the series. PRL, QPM, RNN, SLO, ? (1) TIO

(2) TKP

(3) TJP

(4) TIP

50. Five students are standing in a line. Suman is standing next to Pranav but not next to Deena. Kumar is standing next to Raju, who is standing on the extreme left, and Deena is not standing next to Kumar. Who is standing in the middle? (1) Deena

(2) Raju

(3) Kumar (4) Suman

51. Select the correct combination of mathematical signs to replace ‘*’ signs and to balance the given equation. 32 * 8 * 14 * 7 * 16 (1) × − + =

(2) ÷ × − =

(3) + = ÷×

(4) − × ÷ =

2

2 4

(1) 6

3

(2) 4

4

1

(3) 5

family. N is the son of B, who is not the mother of N. L is the brother of B. D and B are a married couple. T is the daughter of D, who is the sister of P. How is N related to T? (1) Sister (2) Mother (3) Brother (4) Father 56. In the following question, two statements are given each followed by two conclusions I and II. You have to consider the statements to be true even if they seem to be at variance from commonly known facts. You have to decide which of the given conclusions, if any, follows from the given statements. Statements: (I) Pre-school culture is prevailing in all parts of the country now. (II) Parents have less time to spend with their children so toddlers are admitted to school at a very early age. Conclusions: (I) Number of pre-schools is decreasing in all parts of country. (II) Children are becoming very active and intelligent because of this pre-schooling. (1) Only Conclusion II follows (2) Conclusion I and II both follow (3) Neither I nor II follow (4) Only Conclusion I follows 57. Pointing towards a girl, Chetan said, “She is the daughter of the only child of my grandmother”. How is Chetan related to that girl? (1) Father (2) Son (4) Husband (3) Brother

58. Question figure

52. P and Q start cycling from the same point. P cycles 18 km North, then turns to her right and cycles 3 km. Q cycles 9 km East, then turns North and cycles 10 km North, then turns left and cycles 6 km. Where is Q now with respect to the position of P? (1) 8 km South

(2) 8 km North

(3) 28 km South

(4) 28 km North

Answer figures

(2)

(1)





53. Select the correct option that is related to the third term in the same way as the second term is related to the first term. (3)

Bees : Hum :: Snakes : ? (1) Hiss (2) Hoot

(3) Roar (4) Talk

(4) 3

55. P, L, T, B, N and D are six members of a business

(4)





MOCK TEST Paper-2

23

59. Which answer figure will complete the pattern in the question figure? Question Figure:

(4)

(3)

60. A migrating bird flies 40 km North, then turns East and flies 50 km, then turns North and flies 110 km, and turns to its left and flies 50 km. Where is it now with reference to its starting position? (2) 150 km North (1) 150 km South (3) 70 km North (4) 70 km South

?



Answer Figures:

(1)

(2)





Answer Key 1.

(4)

2.

(2)

3.

(4)

4.

(1)

5.

(3)

6.

(4)

7.

(4)

8.

(1)

9.

(2)

10.

(1)

11.

(2)

12.

(2)

13.

(2)

14.

(2)

15.

(2)

16.

(3)

17.

(4)

18.

(3)

19.

(3)

20.

(2)

21.

(3)

22.

(3)

23.

(1)

24.

(3)

25.

(3)

26.

(3)

27.

(3)

28.

(3)

29.

(1)

30.

(1)

31.

(3)

32.

(3)

33.

(2)

34.

(2)

35.

(1)

36.

(1)

37.

(1)

38.

(3)

39.

(2)

40.

(2)

41.

(4)

42.

(4)

43.

(2)

44.

(2)

45.

(1)

46.

(4)

47.

(1)

48.

(3)

49.

(3)

50.

(4)

51.

(4)

52.

(1)

53.

(1)

54.

(4)

55.

(3)

56.

(3)

57.

(3)

58.

(3)

59.

(2)

60.

(2)

24

OSWAAL CUET (UG ) 10 Mock Test Papers GENERAL TEST

MOCK

Test Paper Maximum Marks : 250

3 Time : 60 Minutes

General Instructions : (i) This paper consists of 60 MCQs, attempt any 50 out of 60 (ii) Correct answer or the most appropriate answer: Five marks (+5) (iii) Any incorrect option marked will be given minus one mark (-1) (iv) Unanswered/Marked for Review will be given no mark (0) (v) If more than one option is found to be correct then Five marks (+5) will be awarded to only those who have marked any of the correct options (vi) If all options are found to be correct then Five marks (+5) will be awarded to all those who have attempted the question. (vii) Calculator / any electronic gadgets are not permitted.

1. Which of the following is/are correctly matched? Part Function 1. Ribosomes Making protein 2. Golgi Apparatus Involved in transporting substances in and out of the cell 3. Mitochondria Produces energy for the cell Choose the correct code: (1) 1 only (2) 2 only (3) 3 only (4) 1, 2 and 3 2. Which of the following is/are correctly matched 1. The plastids: It is the outermost covering of animal cells. 2. Lysosomes: It is also known as a suicide bag of the cell. 3. Nucleus: It is defined as a double-membraned eukaryotic cell organelle that contains genetic material. 4. Cell Membrane: It is the cell organelles found in all plant cells. Choose the correct code: (1) 1 and 4 only (2) 2 and 3 only (3) 2, 3, and 4 only (4) 1, 2, 3 and 4 3. Which of the following pair is correctly matched Organisation: Headquarter (1) National Bank for Agriculture and Rural Development (NABARD): Delhi (2) National Housing Bank (NHB): Mumbai (3) Insurance Regulatory and Development Authority (IRDAI): Hyderabad (4) Small Industries Development Bank of India (SIDBI): Prayagraj

4. Who ruled Mysore under a subsidiary alliance with the British after the defeat of Tipu Sultan at the Battle of Seringapatam? (1) Gangas (2) Kadambas (3) Wodeyars (4) Pandyans 5. Which of the following water current is correctly matched? (1) Kuroshio Current: It is a warm current of the Indian Ocean. (2) Humboldt Current: It is a Cold current of the Atlantic Ocean. (3) Labrador Current: It is a Cold current of the Pacific Ocean. (4) Canaries Current: It is a cold current of the Atlantic Ocean. 6. Who became the Nawab of Bengal after the death of Alivardi Khan in 1756? (1) Mir Jafar (2) Murshid Quli Khan (3) Siraj ud-Daulah (4) Sujan Khan 7. Which of the following is/are correctly matched regarding Extreme points of India: 1. Easternmost: Kibithu in Arunachal Pradesh. 2. Westernmost: Ghuar Mota in the Kutch region of Gujarat. 3. Northernmost: Indira Col 4. Southernmost (Mainland): Kanyakumari is also known as Cape Comorin. Choose the correct code (1) 1 and 2 only (2) 2, 3, and 4 only (3) 1, 2, 3 and 4 (4) 2 and 4 only 8. In Microsoft Excel, the ____________ function returns the smallest value among the values passed as arguments. (1) LEAST (2) LESS (3) MIN (4) LOW

MOCK TEST Paper-3

25

9. Which type of image is always formed by a convex mirror?

1. Article 23: Prohibition of traffic in human beings and forced labour.

I. Real II. Virtual II. Enlarged (1) Only I

(2) Only II

(3) Only I and III

(4) Only II and III

10. Which of the following is/are correctly matched? Battle: Year (1) Battle of Chamkaur: 1704 (2) Battle of Swally: 1612 (3) Battle of Plassey: 1764 (4) Battle of Buxar: 1757 Choose the correct code. (1) 1 and 3 only

(2) 2 and 3 only

(3) 3 and 4 only

(4) 1 and 2 only

11. Which of the following gas contributes the maximum to the phenomenal of global warming? (1) Methane (2) Chlorofluorocarbon (CFC) (3) Nitrogen dioxide

2. Article 25: Freedom of conscience and free profession, practice and propagation of religion. 3. Article 19: Protection of certain rights regarding freedom of speech, etc. 4. Article 15: Prohibition of discrimination on grounds of religion, race, caste, sex or place of birth. Choose the correct code. (1) 1, 3 and 4 only

(2) 2, 3 and 4 only

(3) 1, 2 and 4 only

(4) 1, 2, 3 and 4

20. “Bulls eye” is used in the game of: (1) Boxing

(2) Basketball

(3) Polo

(4) Shooting

21. LCM of 2 , 4 , 5 is 3 9 6

(1)

(4) Carbon dioxide

12. Who discovered electron? (1) E. Ghostein

19. Which of the following is/are correctly matched?

(2) J. J. Thomson

(3) Ernest Rutherford (4) J. Chadwick

13. From which country does Dr Thani Al Zeyoudi,

who has been chosen as the chair of the 13th Ministerial Conference of the WTO, hail?

(1) Greece

(2) Qatar

(3) Finland

(4) UAE

14. Sir C. V. Raman was awarded Nobel Prize for his work on:

8 27

(2) Radioactivity

(3) Cryogenics

(4) Sonometer

15. J.J. Thomson received the Nobel Prize in physics for the discovery of __________ (1) Protons

(2) Electrons

(3) Neutrons

(4) Positrons

16. Rehuke khim or ‘cowrie shawl’ are traditional textiles of (1) Assam

(2) Jharkhand

(3) Odisha

(4) Nagaland

17. Who is the recipient of the Lokmanya Tilak Award 2023?

20 3

(3) 10 3

(4) 20 27

22. A boat can row 35 km, upstream in 7 hours and the same distance downstream in 5 hours. What is the speed (in km/hr.) of the boat in still water? (1) 5

(2) 7

(3) 9

(4) 6

23. If a 10-digit number 1330x558y2 is divisible by 88. Then the value of (x + y) is: (1) 7

(2) 9

(3) 6

(4) 8

24. Find the mean of 12, 15, 7, 8 and ‘x + 13’, if x = 2 (1) 11.2

(1) Light Scattering

(2)

(2) 9.5

(3) 11.4

(4) 12

25. A rice trader buys 16 quintals of rice for ` 5632. 20% rice is lost in transportation. At what rate should he sell to earn 25% profit? (1) ` 225.2 per quintal (2) ` 550 per quintal (3) ` 440 per quintal

(4) ` 563.2 per quintal

26. Terms a, 1, b are in Arithmetic Progression and terms 1, a, b are in Geometric Progression. Find ‘a’ and ‘b’ given a ≠ b. (1) 2, 4

(2) –2, 1

(3) 4, 1

(4) –2, 4

27. ∆ABC is an isosceles right angled triangle having

(1) Arvind Kejriwal

(2) Piyush Goyal

∠C = 90°. If D is any point on AB, then AD2 + BD2 is equal to

(3) Amit Shah

(4) Narendra Modi

(1) CD2

18. The Malimath Committee Report deals with: (1) Judicial delays (2) Textile sector reforms (3) Criminal justice system reforms (4) Stock market reforms

(2) 2CD2

(3) 3CD2

(4) 4CD2

28. A kite flying at a height of 120 m is attached to a string which makes an angle of 60° at horizontal. What is the length (in m) of the string? (1) 90 3

(2) 75 3 (3) 84 3

(4) 80 3

OSWAAL CUET (UG ) 10 Mock Test Papers GENERAL TEST

26

29. Refer the following data table and answer the question.

37. To do a certain work, A and B work on alternate

Division/Standard

Boys

Girls

A/5

40

10

B/5

35

25

C/5

15

10

days, with B beginning the work on the first day. A can finish the work along in 48 days. If the work 1 gets completed in 5 days, then B alone can finish 3 4 times the same work in: (1) 24 days (2) 32 days

A/6

35

35

(3) 27 days

B/6

40

30

C/6

20

20

38.

What is the ratio of boys to girls? (1) 26:37

(2) 39:28

(3) 28:39

(4) 37:26

(2) 3 2

(3) 2 6

A is as efficient as B and C together. Working together A and B can complete a work in 36 days and C alone can complete it in 60 days. A and C work together for 10 days. B alone will complete the remaining work in: (1) 110 days (2) 88 days (3) 84 days

30. If x = 5 + 2 6 , then what is the value of x + (1) 2 3

(4) 30 days

1 x

?

(4) 90 days

39. Three circles of diameter 10 cm each, are bound together by a rubber band, as shown in the figure.

(4) 6 2

31. A train without stoppage travels with an average speed of 80 km/h and with stoppage, it travels with an average speed of 72 km/h. For how many minutes does the train stop on an average per hour?

(1) 8

(2) 6

(3) 7

(4) 9

32. A train without stoppage travels with an average speed of 80 km/h and with stoppage, it travels with an average speed of 64 km/h. For how many minutes does the train stop on an average per hour?

(1) 12

(2) 8

(3) 10

(4) 14

33. There are two containers of equal capacity. The ratio of milk to water in the first container is 3 : 1, in the second container 5 : 2. If they are mixed up, the ratio of milk to water in the mixture will be

(1) 28 : 41

The length of the rubber band, (in cm) if it is stretched as shown is (1) 30 (2) 30 + 10π (4) 60 + 20π

(3) 10π

40. The compound interest on ` 4000 for 4 years at 10% per annum will be (1) ` 1856.40 (3) ` 1856

(2) ` 1600 (4) ` 1756.60

41. Which answer figure will complete the pattern in the question figure? Question Figure:

(2) 41 : 28 (3) 15 : 41 (4) 41 : 15

34. An article is listed at ` 2375. A man purchases it

at two successive discounts of 50% and 25% and spends Rs. 165 on repairing of article. If he sells the article at a profit of 62.5%, what is the selling price (in `) of the article?

(1) 1467.6

Answer Figures:

(2) 1492.6 (3) 1715.39 (4) 1467.6

35. The ratio of number of balls in bags x, y is 2:3. Five

balls are taken from bag y and are dropped in bag x. Number of balls are equal in each bag now. Number of balls in each bag now is

(1) 45

?

(2) 20

(3) 30

(2)

(3)

(4)

(1) `

100 2R

(2) ` 50

(3) `

100 R

(4) `

200 R





(4) 25

36. For what sum will the simple interest at R% per annum for 2 years will be R?

(1)





42. If 1st January 2001 was Monday. Then which day of the week was it on 31 December 2001? (1) Monday (2) Wednesday (3) Friday (4) Saturday

MOCK TEST Paper-3

27

43. In a certain code language ‘REASON’ is coded as ‘5410129’ and ‘HEALTHY’ is coded as ‘716231216’. How will ‘DIVULGE’ be coded in that language? (1) 10733494 (2) 21435438 (3) 13704349 (4) 21434383

44. P is the brother of Q. S is the father of P. R is the brother of S. If T is the mother of R, then how is P related to T? (1) Brother (2) Uncle (3) Grandson (4) Father

45. Select the suitable option that is related to the third

number is the same way as the second number is related to the first number. 45 : 315 :: 37 : ? (1) 144 (2) 259 (3) 169 (4) 221

46. In a certain code language, ‘FORENSIC’ is coded as ‘61218221419183’ and ‘DORM’ is coded as ‘4121813’. How will ‘CARAMEL’ be coded in that language? (2) 3260926130512 (1) 3011801132212 (4) 3261826132212 (3) 3261826142212

47. Select the Venn diagram that best illustrates the relationship between the following classes: Parents, Rich persons, Farmers (1)

around a circular table facing each other for a lunch. A is opposite to D and third to the right of B. G is between A and F. H is to the right of A. E is between C and D. Who is sitting between B and E? (1) D

(2) E

(3) F

(4) C

51. From a point, Lokesh starts walking towards South and after walking 30 metres he turns to his right and walks 20 metres, then he turns right again and walks 30 metres. He finally turns to his left and walk 40 metres. In which direction is he with reference to the starting point? (1) North-West

(2) East

(3) West

(4) South

52. Pointing towards a man, Ritika said, “He is the son of my grandfather’s only son”. How is Ritika related to that man? (1) Aunty

(2) Sister

(3) Mother

(4) Wife

53. Select the correct option that is related to the third term in the same way as the second term is related to the first term. Innocent : Guilty :: Moisten : ? (1) Drench (2) Dried

(2)



50. Eight friends A, B, C, D, E, F, G and H are sitting

(3) Desire (4) Wet

54. In a line of boys, Aman is 12th from the top and (3)



Baman is l8th from the bottom. If there are 6 boys between Aman and Baman, then how many maximum boys are there in the row?

(4)



48. Read the given statements and Conclusions

carefully. Assuming that the information given in the statements is true, even if it appears to be at variance with commonly known facts, decide which of the given Conclusions logically follow(s) from the statements. Statements: I. Some cards are postcards. II. Some cards are envelopes. III. All envelopes are copies. Conclusions: I. Some copies are envelopes. II. Some postcards are copies. III. Some cards are copies. (1) Only conclusions II and III follow. (2) Only conclusions I and III follow. (3) Only conclusions I follow. (4) Only conclusions I and II follow.

49. Select the correct option that will fill in the blank and complete the series. pmtz, rkvx, tixv, vgzt, ......... (1) xebr (2) zdcw (3) xeyv

(4) yerb

(1) 34

(2) 36

(3) 35

(4) 37

55. In the following question, if a mirror is placed on line AB, then which of the answer figures is the right image of the given figure? A A C B

A

A BC

A CB

CB

A B C

a

b

c

d

B

(1) a

(2) b

(3) c

(4) d

56. Select the correct combination of mathematical signs to replace ‘*’ signs and to balance the given equation. 54 * 6 * 5 * 14 (1) +÷ =

(2) = ×+ (3) − = × (4) ÷+ =

57. Question figures

OSWAAL CUET (UG ) 10 Mock Test Papers GENERAL TEST

28 Answer figures

(4)

(3) (2)

(1)









60. In the following question, two statements are (3)

given each followed by two conclusions I and II. You have to consider the statements to be true even if they seem to be at variance from commonly known facts. You have to decide which of the given conclusions, if any, follows from the given statements.

(4)



58. Identify the correct option to complete the following series. 50, 49, 47, 44, 40, 35, ........, 22 (2) 27 (1) 30 (3) 29 (4) 25

Statements:

(I) Many rituals, if incorporated in daily routine can make positive changes in one’s life.

59. Which answer figure will complete the pattern in



(II) Bending forward in mosque or prostrating before God are great exercises as these involve backstretch, squat, bend and rise again.

the question figure? Question Figure:

Conclusions:

?



(I) Religion offers great scope for exercise.



(II) If these religious rituals are practiced daily, they can make positive changes in health of an individual. (1) Only Conclusion II follows (2) Conclusion I and II both follow

Answer Figures:

(3) Neither I nor II follow (4) Only conclusion I follows (2)

(1)





 Answer Key 1.

(4)

2.

(2)

3.

(3)

4.

(3)

5.

(4)

6.

(3)

7.

(3)

8.

(3)

9.

(2)

10.

(4)

11.

(4)

12.

(2)

13.

(4)

14.

(1)

15.

(2)

16.

(4)

17.

(4)

18.

(3)

19.

(4)

20.

(4)

21.

(2)

22.

(4)

23.

(2)

24.

(3)

25.

(2)

26.

(4)

27.

(2)

28.

(4)

29.

(4)

30.

(1)

31.

(2)

32.

(1)

33.

(4)

34.

(3)

35.

(4)

36.

(2)

37.

(3)

38.

(1)

39.

(2)

40.

(1)

41.

(4)

42.

(1)

43.

(2)

44.

(3)

45.

(2)

46.

(4)

47.

(2)

48.

(2)

49.

(1)

50.

(1)

51.

(3)

52.

(2)

53.

(2)

54.

(2)

55.

(1)

56.

(4)

57.

(3)

58.

(3)

59.

(4)

60.

(2)

MOCK TEST Paper-1

29

MOCK

4

Test Paper Maximum Marks : 250

Time : 60 Minutes

General Instructions : (i) This paper consists of 60 MCQs, attempt any 50 out of 60 (ii) Correct answer or the most appropriate answer: Five marks (+5) (iii) Any incorrect option marked will be given minus one mark (-1) (iv) Unanswered/Marked for Review will be given no mark (0) (v) If more than one option is found to be correct then Five marks (+5) will be awarded to only those who have marked any of the correct options (vi) If all options are found to be correct then Five marks (+5) will be awarded to all those who have attempted the question. (vii) Calculator / any electronic gadgets are not permitted.

1. Which of the following pair is/are correctly matched? Dynasty

Regional Area

1) Ahom Dynasty

ASSAM

2) Parmar Dynasty

MALWA

3) Gurjar Pratihara Dynasty

UJJAIN

4) Chandel Dynasty

KHAJURAHO

Choose the correct code: (1) 1 and 2 only

(2) 2 and 3 only

(3) 1 and 4 only

(4) 1, 2, 3 and 4

2. Which of the following international organisation headquarter is not located in New York? (1) United Nations Organisation (UNO) (2) United Nations Children's Emergency Fund (UNICEF) (3) United Nations Security Council (UNSC) (4) United Nations Educational Scientific and Cultural Organisation (UNESCO)

3. Representatives of the Maratha Empire and the British East India Company signed the ‘Treaty of Salbai’ in which year? (1) 1782

(2) 1769

(3) 1758

(4) 1771

4. Which of the following pair is not correctly matched? Viceroy/ Governor General 1. Lord Canning 2. Lord Ripon

3. Warren Hastings 4. Lord Dufferin Choose the correct code. (1) 1 only (2) (3) 3 only (4)

He was the 1st Governor General of India. The formation of INC took place during his tenure in 1885. 2 and 3 only 3 and 4 only

5. Which of the following option is not correct? 1. In terms of geographical expanse, the district of Mahe in Puducherry holds the distinction of being the smallest in India. 2. The largest district in India by area is Kutch in Gujarat. 3. The second largest district in India by area is Leh. 4. Madhya Pradesh is the largest Indian state in terms of area. Choose the correct code. (1) 1 and 2 only (2) 2 only (3) 4 only (4) 1 and 4 only

6. Which of the following pair is/are correctly matched?

Major crops

Top-producing countries:

Worked during his rule

1. Rice

China

2. Corn

USA

He served as the first Viceroy of India.

3. Jute

India

4. Wheat

India

The Vernacular Press Act was repealed in 1882.

Choose the correct option: (1) 1, 2 and 3 only

(2) 2 and 4 only

(3) 2, 3 and 4 only

(4) 1, 2, 3 and 4

OSWAAL CUET (UG ) 10 Mock Test Papers GENERAL TEST

30

7. Which of the following pair is/are correctly

matched? 1. Fiscal deficit = Total expenditures – Total receipts excluding borrowings 2. Revenue deficit = Total revenue expenditure – Total revenue receipts 3. Primary deficit = Fiscal deficit – Interest payments Choose the correct option. (1) 1 only (2) 2 only (4) 1, 2 and 3 (3) 2 and 3 only

8. Which of the following pair is INCORRECT? I. Touch Pad – Locator device II. Microphone – Output device III. Monitor – Output device (1) I and II (2) I and III (3) Only II (4) All options are correct.

9. Which angiosperm is vessel less? (1) Hydrilla (3) Maize

(2) Trochodendron (4) Wheat

10. Hind limbs thumping on ground by a rabbit is a behaviour related to (1) Courtship (2) Preparation for a duet (3) Warning signal to members (4) Surrender to a stronger opponent

11. Which of the following option is correct? 1. The provision of Bharat Ratna was introduced in 1954. 2. The first Bharat Ratna award was awarded to politician C.Rajagopalachari, philosopher Sarvepalli Radhakrishnan, and scientist CV Raman in the year 1954. 3. The first two Non-Indians who receive the Bharat Ratna are Khan Abdul Ghaffar Khan in 1987 and Nelson Mandela in 1990. 4. The award had been awarded to a naturalized Indian citizen Mother Teresa in 1980. Choose the correct code. (1) 1, 3 and 4 only (2) 2, 3 and 4 only (3) 1, 2 and 3 only (4) 1, 2, 3 and 4

12. The theme of Cancer Day 2023 is (1) We can, I can (3) I Am and I Will

(2) Close the care gap (4) Not Beyond Us.

13. Laser was invented by (1) A.H. Taylor (3) Lee De Forest

(2) T.H. Maiman (4) Thomas Edison

14. Coup is ______________. (1) Sudden overthrow of a government illegally (2) A system of rules that taken effect when a military authority takes control of normal administration of justice.

(3) Territory under the immediate political control of another state (4) None of these

15. According to Article 361 of the Constitution of India, a criminal proceeding cannot be instituted in a court against the _____________ during his term of office. (1) Vice-president

(2) Prime Minister

(3) Chief Minister

(4) Governor

16. Which of the following is the major component of vinegar? (1) Nitric acid

(2) Lactic acid

(3) Citric acid

(4) Acetic acid

17. Methane is a colorless. Odorless, non-toxic but flammable gas. What is its common name? (1) Blue vitriol

(2) Laughing gas

(3) Heating gas

(4) Marsh gas

18. Match List-I with List-II List-I (Games)

List-II (Trophies)

A. Hockey Aga

I. Khan Cup

B. Football

II. Begum Hazrat Mahal Cup

C. Table Tennis

III. Ramanuja trophy

D. Golf

IV. Walker cup

Choose the correct answer from the options given below: (1) a-III, b-I, c-IV, d-II

(2) a-I, b-II, c-III, d-IV

(3) a-IV, b-I, c-II, d-III

(4) a-III, b-II, c-IV, d-I

19. Where is the satellite launching center of India located? (1) Ahmedabad

(2) Hassan

(3) Sriharikota

(4) Thumba

20. Match list-I with list-II List-I (State)

List-II (Festivals)

A. Andhra Pradesh

I. Flamingo Festival

B. Karnataka

II. Pongal

C. Tamil Nadu

III. Kambala Festival

D. Kerala

IV. Theyyam Festival

Choose the correct answer from the options given below: (1) a-III, b-I, c-IV, d-II

(2) a-I, b-III, c-II, d-IV

(3) a-IV, b-I, c-II, d-III

(4) a-III, b-II, c-IV, d-I

21. 6 1 % of 1600 + 12 1 % of 800 equals 4

(1) 100

2

(2) 200

(3) 300

(4) 400

22. A shop-keeper sold a sewing machine for ` 1,080 at a loss of 10%. At what price should he has to sold it so as to gain 10% on it? (in `) (1) 1,069

(2) 1,200

(3) 1,230

(4) 1,320

23. From a point P on the ground, the angle of elevation of the top of a 10 m high building is 30°.

MOCK TEST Paper-4

31

A flag is hoisted at the top of the building and the angle of elevation of the top of the flagstaff from the point P is 45°. The length of the flagstaff is: (Take 3 = 1.732) (1) 7 m

(2) 7.24 m (3) 7.56 m (4) 7.32 m

24. For the data 11, 15, 13, 12, 10, 8, 11, 7, 15, 11, 13, 7, 11, which of the following is true? (1) Mean = Median = Mode (2) Mean > Median > Mode (3) Mean > Mode = Median

25. Two successive discounts of 10% and 5%, in this

order, are given on a bill of ` 110. Find the net amount of money payable to clear the bill. (answer to the nearest rupee) (2) ` 95

(3) ` 96

question. Marks

Number of students

40 and above

25

30 and above

44

20 and above

61

10 and above

88

0 and above

105

How many students have scored marks 20 or more but less than 40?

(4) Mean < Median < Mode

(1) ` 94

34. Refer the following data table and answer the

(4) ` 97

26. The sum of money that will yield ` 60 as simple

(1) 44

(2) 61

(3) 36

35. If the square of the sum of two numbers is equal to 4 times of their product, then the ratio of these numbers is :

(1) 2 : 1

(2) 1 : 3

(3) 1 : 1

its altitude is equal to:

(1) ` 200

(1) 6 2 cm

(2) 3 2 cm

(3) 6 3 cm

(4) 3 6 cm

(3) ` 175

(4) ` 300

27. In a stream running at 3 km/h, a motorboat goes 12 km upstream and back to the starting point in 60 minutes. Find the speed of the motorboat in still water, (in km/h).

(1) 2( 2 + 17 ) (3) 3( 2 + 17 )



(2) 2( 4 + 15 )

divided into the largest possible equal classes, so that each class of boys numbers the same as each class of girls. What is the number of classes? (2) 19

(3) 44

(4) 17

29. A and B together can do a piece of work in 9 days.

If A does thrice the work of B in a given time, the time A alone will take to finish the work is

(1) 4 days

(2) 6 days (3) 8 days (4) 12 days

30. A canal of a village can be cleaned by 24 villagers in 12 days. The number of days in which 36 villagers can clean the canal is

(1) 18

(2) 8

(3) 72

(4) 16

31. If x  3  5 , then the value of x2 – 16x + 6 is (1) 0

(2) – 2

(3) 2

(4) 4

32. The digit in the unit place in the square root of 66049 is

(1) 3

(2) 7

(3) 8

average weight of L and M be 89 kg and that of M and N be 96.5 kg, then the weight (in kg) of M is_____________. (2) 86

(3) 101

station B in 45 minutes. If the speed of the train is reduced by 5 km/hr, then the same distance is covered in 48 minutes. The distance between station A and B is

(1) 60 km

(2) 64 km (3) 80 km (4) 55 km

39. The compound interest on a certain sum of money for 2 years at 5% is ` 328, then the sum is

(1) ` 3000

(2) ` 3600 (3) ` 3200 (4) ` 3400

40. An alloy contains copper, zinc and nickel in the ratio of 5 : 3 : 2. The quantity of nickel (in kg) that must be added to 100 kg of this alloy to have the new ratio 5 : 3 : 3 is (1) 8 (2) 10 (3) 12 (4) 15

41. Select the Venn diagram that best illustrates the relationship between the following classes: Graduates, Literates, Hardworking (2)

(1)

(4) 2



33. In an isosceles ∆ABC, AD is the median to the

unequal side meeting BC at D. DP is the angle bisector of ∠ADB and PQ is drawn parallel to BC meeting AC at Q.Then the measure of ∠PDQ is:

(1) 130°

(2) 90°

(3) 180°

(4) 95

38. A train covers a distance between station A and

28. In a school, 391 boys and 323 girls have been

(1) 23

37. The average weight of L, M and N is 93 kg. If the

(1) 92

(4) 3( 4 + 17 )

(4) 1 : 2

36. If each side of an equilateral triangle is 12 cm then

interest at the rate of 6% per annum in 5 years is (2) ` 225

(4) 105

(4) 45°

(3)





(4)



OSWAAL CUET (UG ) 10 Mock Test Papers GENERAL TEST

32

42. Read the given statements and conclusions

carefully. Assuming that the information given in the statements is true, even if it appears to be at variance with commonly known facts, decide which of the given Conclusions logically follow(s) from the statements. Statements: I. Some bowls are cups. II. All cups are glasses. III. Some glasses are plates. All plates are utensils. Conclusions: I. All bowls cannot be utensils. II. All glasses cannot be utensils. (1) Only conclusion I follows. (2) Both the conclusions follow. (3) Only conclusion II follows. (4) Neither conclusion I nor II follows.

43. Choose the most appropriate mirror image of the figure.

47. Preeti is the only sister of Virat. Manoj is the son of Dileep, who is the husband of Maaya. Virat is the son of Devansh. Devansh is Manoj’s father-in-law. How is Maaya related to Preeti? (1) Daughter-in-law

(2) Mother-in-law

(3) Mother

(4) Sister in-law

48. Select the suitable option that is related to the third number in the same way as the second number is related to the first number. 22 : 242 :: 18 : ? (1) 275

(2) 169

(3) 162 (4) 221

49. In a certain code language, “who are you” is written as “432”, “they is you” is written as “485” and “they are dangerous” is written as “295”. How is “dangerous” written in that code language? (1) 2

(2) 4

(3) 5

(4) 9

50. Select the letter that will come next in the following series. O, R, U, X, A, ..... (1) D

(2) C

(3) B

(4) E

51. Given below are two statements: one is labelled as Assertion (A) and the other is labelled as Reason (R). Assertion (A): Leakages in household gas cylinders can be detected. Reason (R): LPG has a strong smell. (1)



(2)

In the light of the above statements, choose the most appropriate answer from the options given below: (1) Both (A) and (R) are correct and (R) is the correct explanation of (A)

(3)



(4)

(2) Both (A) and (R) are correct but (R) is not the correct explanation of (A) (3) (A) is correct but (R) is not correct (4) (A) is not correct but (R) is correct

44. Select the option in which the word shares the same relationship as that shared by the given pair of words Turmeric Spices (1) Fig Cashew Nuts (2) Dates Dry Fruits (3) Apricot Pear (4) Nutmeg Chili Flakes 45. In a certain code language, ‘RAJ’ is coded as ‘87’ and ‘GITA’ is coded as ‘148’. How will ‘VARUN’ be coded in that language? (1) 234 (2) 380 (3) 176 (4) 403 46. In a row of cars, blue car is at the bottom end. There are 18 cars between yellow and blue car. There are 8 cars between red and blue car. If there are 6 cars above yellow car, then how many minimum cars are there in the row? (1) 34 (2) 26 (3) 32 (4) 35

52. If ‘P’ means ‘subtracted from’, ‘X’ means ‘multiplied by’, ‘Y‘ means ‘added to’, and ‘Z’ means ‘divided by’, then: 54 Z 3 Y 22 X 5 = ? (1) 135

(2) 128

(3) 110

(4) 144

53. X and Y both start from a same point X walks 17 m West, then turns to his right and walks 13 m. At the same time, Y walks 9 m North, then turns East and walks 7 m, then turns to his left and walks 4 m. Where is Y now with respect to the position of X? (1) 24 m West

(2) 10 m East

(3) 10 m West

(4) 24 m East

54. Select the correct option that will fill in the blank and complete the series. T, R, P, N, L, ...., ....... (1) J, H

(2) K, I

(3) J, G

(4) K, H

55. Which answer figure will complete the pattern in the question figure?

MOCK TEST Paper-4

33

Question Figure:

Conclusions: (I) Pollution is resulting into rise in temperature of the world. (II) Pollution is the only reason for depletion of the nature.

?

(1) Only Conclusion II follows (2) Conclusion I and II both follows (3) Neither I nor II follows

Answer Figures:

(4) Only Conclusion I follows

57. Select the correct option that is related to the third (1)

term in the same way as the second term is related to the first term.

(2)

Lion : Roar :: Elephant :? (1) Bark (2) Bray



(3) Trumpet (4) Grunt

58. Select the correct option that will fill in the blank and complete the series. (3)

(4)

2, 8, 32, 128, 512, ? (1) 2507



(2) 2536

(3) 2067

(4) 2048

59. A paper boy cycles 10 km East, then turns South



and cycles 3 km, then turns West and cycles 6 km, then turns to his right and cycles 3 km. Where is he now with reference to his starting position?

56. In the following question, two statements are given each followed by two conclusions I and II. You have to consider the statements to be true even if they seem to be at variance from commonly known facts. You have to decide which of the given conclusions, if any, follows from the given statements. Statements: (I) Glaciers are receding and the temperature rise of the world is more than last year this time. (II) Pollution level of the Earth is rising a lot resulting in avalanches.

(1) 4 km West

(2) 14 km East

(3) 4 km East

(4) 14 km West

60. Pointing to a lady, Anup said, “She is the only daughter of the lady who is the mother of my mother’s only grandson”. How is the lady which is pointed related to Anup? (1) Daughter

(2) Wife

(3) Sister

(4) Mother

 Answer Key 1.

(4)

2.

(3)

3.

(1)

4.

(3)

5.

(3)

6.

(2)

7.

(4)

8.

(3)

9.

(2)

10.

(3)

11.

(4)

12.

(2)

13.

(2)

14.

(1)

15.

(4)

16.

(4)

17.

(4)

18.

(2)

19.

(3)

20.

(2)

21.

(2)

22.

(4)

23.

(4)

24.

(3)

25.

(1)

26.

(1)

27.

(4)

28.

(4)

29.

(4)

30.

(2)

31.

(3)

32.

(2)

33.

(2)

34.

(3)

35.

(3)

36.

(3)

37.

(1)

38.

(1)

39.

(3)

40.

(2)

41.

(1)

42.

(4)

43.

(1)

44.

(2)

45.

(2)

46.

(2)

47.

(2)

48.

(3)

49.

(4)

50.

(1)

51.

(4)

52.

(2)

53.

(4)

54.

(1)

55.

(1)

56.

(3)

57.

(3)

58.

(4)

59.

(3)

60.

(1)

34

OSWAAL CUET (UG ) 10 Mock Test Papers GENERAL TEST

MOCK

5

Test Paper Maximum Marks : 250

Time : 60 Minutes

General Instructions : (i) This paper consists of 60 MCQs, attempt any 50 out of 60 (ii) Correct answer or the most appropriate answer: Five marks (+5) (iii) Any incorrect option marked will be given minus one mark (-1) (iv) Unanswered/Marked for Review will be given no mark (0) (v) If more than one option is found to be correct then Five marks (+5) will be awarded to only those who have marked any of the correct options (vi) If all options are found to be correct then Five marks (+5) will be awarded to all those who have attempted the question. (vii) Calculator / any electronic gadgets are not permitted.

1. Which of the following statement is/are correct? 1. When the ruler of Avanti, Pradyota suffered from Jaundice bimbisara sent royal physician Jivaka. 2. The famous weapons Mahasilakantaka and Rathamusala were introduced by Ajatshatru. 3. Udayin made Pataliputra the capital of Magadha and also built a fort at the confluence of the Ganges and Son at Patna. 4. The second Buddist council was held at Vaishali during kallashoka reign. Choose the correct code. (1) 1 only (2) 2 and 4 only (3) 3 and 4 only (4) 1, 2, 3 and 4 2. Which of the following is/are correct? 1. X-Rays were discovered by Wilhelm Conrad Roentgen in 1895. 2. JJ Thomson, while investigating cathode rays in the 19th century, discovered the electron. 3. Ernest Rutherford discovered protons in 1917 during his gold foil experiment. Choose the correct code. (1) 1 only (2) 1, 2 and 3 (3) 2 and 3 only (4) 3 only 3. Match list I with list II List I (Wind) List II (Location) 1. Foehn A. The Alps 2. Sirocco B. Sahara to the Mediterranean Sea 3. Chinook C. The Rockies mountains 4. Mistral D. The Alps and France Choose the correct code. (1) 1-A, 2-B, 3-C, 4-D (2) 1-B, 2-C, 3-B, 4-D (3) 1-D, 2-B, 3-C, 4-A (4) 1-C, 2-D, 3-B, 4-A

4. Which of the following is NOT a twin city of India? (1) Delhi and New Delhi (2) Durgapur and Asansol (3) Pune and Pimpri-Chinchwad (4) Durg and Bhilai

5. What will be the focal length when human eye lens becomes thin? (1) Increases

(2) Decreases

(3) Remain same

(4) No option is correct

6. Which of the following is/are correct? 1. The 42nd Amendment to the Indian constitution, enacted in 1976, established Administrative Tribunals in India. 2. The 24th constitutional amendment made it obligatory for the president to grant assent to a constitutional amendment bill. 3. The 44th constitutional amendment removed the right to property from the list of fundamental rights. 4. The First Amendment was passed in 1951 by the Provisional Parliament, whose members had just completed drafting the Constitution as part of the Constituent Assembly. Choose the correct code. (1) 1 and 2 only

(2) 2, 3 and 4 only

(3) 1, 2 and 4 only

(4) 1, 2, 3 and 4

7. According to the NITI Aayog, how many people moved out of multidimensional poverty between 2015-16 and 2019-21? (1) 10 crore

(2) 12 crore

(3) 13.5 crore

(4) 15 crore

MOCK TEST Paper-5

35

8. Which of the following is/are correct? 1. National Electronic Funds Transfer (NEFT) is a nationwide centralized payment system owned and operated by the Reserve Bank of India (RBI). 2. IFSC or Indian Financial System Code is an 11 digits alpha-numeric code that uniquely identifies a bank branch participating in the NEFT system. 3. MICR Code or Magnetic ink character recognition(MICR) is a 9 digits code printed on the cheque which enables identification of the cheques and identifies the bank and branch participating in an Electronic Cheque Clearing system. 4. Swift Code (Society for Worldwide Interbank Financial Telecommunication) is an internationally recognized system for identifying banks and financial institutions globally. Choose the correct code. (1) 1 and 4 only (2) 1 and 3 only (3) 2 and 3 only (4) 1, 2, 3 and 4 9. Which of the following is TRUE? (1) Primary memory is non-volatile (2) Secondary memory is volatile. (3) ROM is non-volatile memory. (4) RAM is non-volatile memory. 10. In which communication mode, data can be transmitted in both directions at same time? (2) Full duplex (1) Simplex (4) Multiplex V (3) Half duplex 11. Clove, the commonly used spice, is obtained from the: (1) root (2) stem (4) fruit (3) flower bud 12. Match List I with List II List I List II (Neighbouring (Bordering Country) States) A. China I. Jammu and Kashmir (POK Part) B. Bangladesh II. West Bengal, Mizoram, Meghalaya, Tripura, and Assam C. Bhutan III. Arunachal Pradesh, Assam, Sikkim, and West Bengal D. Afghanistan IV. Jammu and Kashmir, Himachal Pradesh, Uttrakhand, Sikkim, and Arunachal Pradesh Choose the correct code. (1) A-1, B-2, C-3, D-4 (2) A-4, B-2, C-3, D-1 (3) A-1, B-3, C-2, D-4

(4) A-4, B-3, C-2, D-4

13. Which of the following international organisation headquarter is not located in Geneva? (1) United Nations Conference on Trade and Development (UNCTAD) (2) World Health Organisation (WHO) (3) International Labour Organisation (ILO) (4) Organisation for the Prohibition of Chemical Weapons

14. The award given for outstanding performance in sports is (1) Dronacharya Award (2) Bharat Ratna (3) Padma Shri Award (4) Arjuna Award

15. ‘Agha Khan Cup’ is related with which of the following sports? (1) Football (3) Hockey

(2) Cricket (4) Table Tennis

16. Radar is used to: (1) Locate submerged submarines. (2) Receive signal from radio receivers. (3) Detect and locate distant objects. (4) Locate geostationary satellites.

17. Which of the following acid found in Apple? (1) Malic Acid (3) Formic Acid

(2) Nitric Acid (4) Sulphuric Acid

18. Which of the following has a strong fruity fragrance? (1) Methyl chloride (3) Methanol

(2) Methanoic acid (4) Ethyl acetate

19. ‘World Ozone Day’ is celebrated every year on ______ September. (2) 16 (1) 8

(3) 12

(4) 6

20. Bhavai is a traditional dance form of ____________ (1) Gujarat

(2) Bihar

(3) Punjab

(4) Maharashtra

21. A man sold 20 apples for ` 100 and gained 20%. How many apples did he buy for ` 100?

(1) 20

(2) 22

(3) 24

(4) 25

22. If the arithmetic mean and the geometric mean

of the roots of a quadratic equation are 8 and 5 respectively, then the quadratic equation is: (1) x2 – 8x + 5 = 0 (2) x2 – 8x + 25 = 0 (3) x2 – 16x + 25 = 0

(4) 2x2 – 8x + 25 = 0

23. The sum of squares of three natural numbers is 1029, and they are in the proportion 1 : 2 : 4. The difference between greatest number and the smallest number is:

(1) 21

(2) 15

(3) 18

(4) 31

24. The length of the shadow on the ground of a tall tree of height 30 m is 10 3 m. What is the angle (in degrees) of elevation of the sun? (1) 60

(2) 15

(3) 30

(4) 45

OSWAAL CUET (UG ) 10 Mock Test Papers GENERAL TEST

36

25. A can do as much work in 4 days as B can do in 5

days, and B can do as much work in 6 days as, C in 7 days. In what time will C do a piece of work which A can do in a week?

5 (1) 10 days 24 (3) 6

4 (2) 4 days 5

8 days 15

(4) 12

(3) 3

(1) 4

(4) 5

27. A train, 150 m long, passes a pole in 15 seconds and

another train of the same length travelling in the opposite direction in 12 seconds. The speed of the second train is (2) 48 km./hr (1) 45 km./hr (3) 52 km./hr (4) 54 km./hr

28. Let x be the least number which when divided by

15, 18, 20 and 27, the remainder in each case is 10 and x is a multiple of 31. What least number should be added to x to make it a perfect square?

(1) 39

(2) 37

(3) 43

(2) 60 cc

(3) 5

(1) 72

(2) 70

25% of the number of coconut trees are mango trees and 20% of the number of mango trees are apple trees. If the number of apple trees are 1500, then the number of trees in the garden is: (2) 50000

(3) 51000

(4) 45000

31. Refer the following data table and answer the question.

(4) 66

have a radius of 84 cm. If the wire is bent to form a square, the length of a side of the square is (1) 152 cm

(2) 132 cm (3) 168 cm (4) 225 cm

37. The area of a triangle is 15 sq. cm and the radius if its incircle is 3 cm. Its perimeter is equal to: (1) 12 cm

(2) 20 cm (3) 5 cm 1 1+ 2

+

1 2+ 3

+

(4) 10 cm 1

3+ 4

(3) 2

+ ........ +

(4)

1 8+ 9

2

39. If two successive discounts of 20% and 30% are given, what is the net discount (in %)? (1) 40

(2) 44

(3) 56

(4) 60

40. A principal of ` 10,000, after 2 years compounded annually, the rate of interest being 10% per annum during the first year and 12% per annum during the second year (in rupees) will amount to: (1) ` 12,000

(2) ` 12,320

(3) ` 12,500

(4) ` 11,320

Years

GDP growth rate for the year (in %)

2011

–7

2012

–5

2013

7

“Her mother’s father’s son’s wife is my motherin-law’s only daughter”. How is Nitin related to Sanchi’s mother?

2014

4

(1) Brother

(2) Paternal Uncle

2015

–4

(3) Maternal Uncle

(4) Grandfather

If the GDP of the country was $3 trillion at the end of 2013, what was it in the beginning of 2015? (1) $3.21 trillion (2) $2.88 trillion (3) $3.12 trillion

x

(2) 2

41. Pointing to the photograph of Sanchi, Nitin said,

42. Select the Venn diagram that best illustrates the relationship between the following classes: Doctors, Salaried persons, Parents

(4) $3 trillion

32. If x2 – 3x + 1 = 0, then the value of x 2 + x + 1 + 12 is (1) 10

(3) 68

36. A piece of wire when bent to form a circle will

1 1 1 1 is BC such that AD ⊥ BC, then + + + ........ + + + + 8 + 9 1 2 2 3 3 4 (1) AB : BD = 1 : 1 (2) AB : BD = 1 : 2 (1) 1 (2) 0 (3) AB : BD = 2 : 1 (4) AB : BD = 3 : 2

30. In a big garden 60% of the trees are coconut trees,

(4) 0

average weight of X and Y be 68 kg. and that of Y and Z be 78 kg. then the weight (in kg) of Y is____________.

38. The value of

(4) 36

(2) 6

35. The average weight of X, Y and Z is 74 kg. if the

29. If ABC is an equilateral triangle and D is a point on

(1) 48000

(3) 62.5 CC (4) 64 cc

numbers?

6 days 19

the value of (7x – 2y) (2) 7

(1) 55 cc

34. What is the unit digit of the sum of first 111 whole

26. If 10-digit number 6712y76x2 is divisible by 88 then (1) 10

cc) that should be added to the mixture so that the new ratio of the volumes of water and glycerine become 2 : 3 is

(3) 6

x

(2)

(1)





(4) 8

33. The ratio of the volume of water and glycerine in 240cc of a mixture is 1 : 3. The quantity of water (in

(3)



(4)



MOCK TEST Paper-5

37

43. Read the given statements and conclusions

carefully. Assuming that the information given in the statements is true, even if it appears to be at variance with commonly known facts, decide which of the given conclusions logically follows from the statements. Statements: I. All lockets are chains. II. 100% medals are lockets. III. Some rings are chains. Conclusions: I. All chains are medals. II. All medals being rings is a possibility. (1) Only conclusion II follows. (2) Neither conclusion I nor II follows. (3) Only conclusion I follows. (4) Both conclusions I and II follows.

Choose the correct answer from the options given below : (1) (a), (c), (d), (e), (b) (2) (e), (b), (c), (d), (a) (3) (a), (c), (d), (b), (e) (4) (d), (e), (b), (a), (c)

48. Select the option in which the numbers are related in the same way as are the numbers in the given set. (11, 165, 209) (1) (15, 225, 275)

(2) (12, 180, 228)

(3) (14, 210, 276)

(4) (17, 245, 323)

49. In the following diagram, the triangle represents doctors, the circle represents players and the rectangle represents singers.

44. Which answer figure will complete the pattern in

the question figure? Question Figure:

50. A, B, C, D, E and F are sitting on the round table

with equal distances. F is sitting opposite E and between A and D. C is sitting right side of E and opposite to A. Who are the neighbours of A? (1) F and D (2) E and F (4) B and F (3) E and C

Answer Figures:

51. Vikram started from point R and walked straight



52. Question figure

(2)

(1)





(3)

(4)

D

Which region represents doctors who are singers but not players? (1) A (2) B (3) C (4) D

?



A C

B



7 km West, then turned left and walked 2 km and again turned left and walked straight 4 km. In which direction is he from R? (1) North-East (2) South-West (3) South-East (4) North-West



45. Select the option that is related to the third word in the same way as the second word is related to the first word. Jackal : Howl : Rain : ? (1) Hustle (2) Thunder (4) Drops (3) Patter 46. Sunil’s position from the left in a row of boys is 20th and Deepak’s position from the right is 36th. After interchanging their position, Sunil becomes 28th from the left. How many boys are there in the row? (1) 52 (2) 63 (3) 59 (4) 48 47. Arrange the given words in the sequence in which they occur in the dictionary. (a) Amphibian (b) Amorphous (c) Amphidextrous (d) Ambiguous (e) Ambivalent

Answer figures (1)

(2)



(3)

(4)





53. In the following question, two statements are

given each followed by two conclusions I and II. You have to consider the statements to be true even if they seem to be at variance from commonly known facts. You have to decide which of the

OSWAAL CUET (UG ) 10 Mock Test Papers GENERAL TEST

38

given conclusions, if any, follows from the given statements. Statements: (I) Life is a journey with few known people, few partially known and mostly strangers. (II) In a journey, the importance of destination is as equal as the pathway of reaching that destination. Conclusions: (I) Importance of people in life is considerable, as they are our journey mates. (II) Life teaches us many lessons to face various known and unknown difficulties. (1) Only Conclusion II follows (2) Conclusion I and II both follow (3) Neither I nor II follow (4) Only Conclusion I follows



54. If ‘P’ means ‘−’, ‘Q’ means ‘÷’, ‘R’ means ‘+’, and ‘S’ means‘×’, then: 525 Q 25 R 42 S 4 = ? (1) 178 (2) 187 (3) 189 (4) 198



(4)

(3)



56. Select the related word pair from the given alternatives. Teeth : Cut :: ....... : ......... (1) Legs : Walk (2) Broom : Fly (3) Tubelight : Heat (4) Food : Smell

57. In a row of cars, Maruti is 20th from the left end of

the row. Honda is 10th to the right from Maruti and is at the exact center of the row. How many cars are there in the row? (2) 59 (3) 57 (4) 56 (1) 54

58. Arsh is Shivam’s father and Dhruv is the son of Bimla. Eshwar is the father of Arsh. If Shivam is the brother of Dhruv, how is Bimla related of Eshwar?

55. Which answer figure will complete the pattern in

the question figure? Question Figure:

(1) Sister-in-law

(2) Mother

(3) Daughter-in-law

(4) Wife

59. Select the term that will come next in the following

?

series. BMO, EOQ, HQS, KSU, ........... (1) MTV (2) NUW (3) NTV

(4) NVW

60. Given below are numbers in the first line and symbols in the second line. Numbers and symbols are codes are each other. Choose the correct code for given symbols.

Answer Figures:



2

3







Given:

  〈 

(1) 91486



4



5

6

7







(2)

(1)

1

8

9

  ◊

(2) 91846

(3) 94816

(4) 94846

 Answer Key 1.

(4)

2.

(2)

3.

(1)

4.

(1)

5.

(1)

6.

(4)

7.

(3)

8.

(4)

9.

(3)

10.

(2)

11.

(3)

12.

(2)

13.

(4)

14.

(4)

15.

(3)

16.

(3)

17.

(1)

18.

(4)

19.

(2)

20.

(1)

21.

(3)

22.

(3)

23.

(1)

24.

(1)

25.

(1)

26.

(2)

27.

(4)

28.

(1)

29.

(3)

30.

(2)

31.

(3)

32.

(1)

33.

(2)

34.

(3)

35.

(2)

36.

(2)

37.

(4)

38.

(3)

39.

(2)

40.

(2)

41.

(1)

42.

(2)

43.

(1)

44.

(3)

45.

(3)

46.

(2)

47.

(4)

48.

(2)

49.

(1)

50.

(4)

51.

(2)

52.

(4)

53.

(4)

54.

(3)

55.

(1)

56.

(1)

57.

(2)

58.

(3)

59.

(2)

60.

(1)

MOCK TEST Paper-1

39

MOCK

6

Test Paper Maximum Marks : 250

Time : 60 Minutes

General Instructions : (i) This paper consists of 60 MCQs, attempt any 50 out of 60 (ii) Correct answer or the most appropriate answer: Five marks (+5) (iii) Any incorrect option marked will be given minus one mark (-1) (iv) Unanswered/Marked for Review will be given no mark (0) (v) If more than one option is found to be correct then Five marks (+5) will be awarded to only those who have marked any of the correct options (vi) If all options are found to be correct then Five marks (+5) will be awarded to all those who have attempted the question. (vii) Calculator / any electronic gadgets are not permitted.

1. Which of the following option is not correct related

to the Nanda dynasty? 1. It was founded by Mahapadma Nanda around 467 BC. 2. Dhanananda was the last ruler of the Nanda Dynasty and was overthrown by Chandra Gupta Maurya. 3. Alexander invaded India in 326 BC when Dana Nanda was the ruler of the Nanda dynasty. Choose the correct code. (1) 1 only (2) 2 and 3 only (3) 1 and 3 only (4) 1, 2 and 3 2. Which of the following is the correct chronological order of the following events of Indian history? 1. Lucknow Pact 2. Formation of the Swaraj Party 3. Jallianwala Massacre 4. Gandhi Irwin pact Choose the correct code. (1) 1324 (2) 2341 (3) 1234 (4) 2134 3. Which of the following is/are correctly matched Types Of Clouds Features 1. Stratus These are layered clouds covering large portions of the sky. 2. Nimbus They form at middle levels or very near to the surface of the earth. 3. Cumulonimbus These are the “thunderheads” that can be seen on a warm summer day and can bring strong winds, hail, and rain. Choose the correct code. (1) 1 only (2) 2 only (3) 1 and 3 only

(4) 1, 2 and 3

4. During which of the following session did the All India Muslim League adopt the Pakistan Resolution on 23 March 1940? (1) Karachi Session

(2) Lahore Session

(3) Lucknow Session

(4) Nagpur Session

5. Which city of India is known as ‘The Athens of The East’? (1) Allahabad

(2) Patna

(3) Madurai

(4) Kochi

6. Which river passes through maximum number of countries? (1) Rhine

(2) Danube

(3) Amazon

(4) Volga

7. Which of the following state government launched Nand Baba Milk Mission? (1) Uttar Pradesh

(2) Madhya Pradesh

(3) Bihar

(4) Chhatisgarh

8. Which Indian state launched the Namo Shetkari Mahasanman Yojana? (1) Punjab

(2) Maharashtra

(3) Assam

(4) Haryana

9. In which year was the first-ever motion to remove a Supreme Court Justice signed, by 108 members of the Parliament? (1) 1984

(2) 1991

(3) 1978

(4) 1996

10. The first stable product of photosynthesis is (1) Starch (2) Sucrose (3) Phosphoglyceric acid (4) Glucose

OSWAAL CUET (UG ) 10 Mock Test Papers GENERAL TEST

40

11. Which of the following enzyme pairs is not (1) Trypsin and Pepsin (2) Trypsin and Amylase (3) Chymotrypsin and Pepsin

(1) 85.52

(4) Pepsin and Amylase.

12. How many nations have officially attained

recognition from the World Health Organization (WHO) that they have successfully eliminated trachoma as a public health concern? (1) 14 (2) 16 (3) 18 (4) 20

13. Which among the following was the first satellite launched by Indian Space Research Organization? (2) Aryabhatta (1) Bhaskar (4) INSAT-1A (3) Rohini

14. Which of the following is NOT a vocal form of Hindustani classical music? (1) Pakhawaj (2) Drupad (3) Tarana (4) Dhamar

recently achieved the distinction of becoming the first-ever Indian to secure an individual World Title at the World Archery Championships? (2) Deepika Kumari (1) Rakhi Sharma (4) Aditi Swami (3) Savita Rathi

16. Ability to distinguish two closely placed objects is (1) Resolving power (2) Video power (3) Distinguish power (4) Magnifying power

17. Which of the following particles were made to fall on a thin gold foil by Ernest Rutherford? (1) Gamma (2) Beta (3) Electron (4) Alpha

18. Which gas is used as a fuel in a rocket? (2) Hydrogen (4) Nitrogen

Days and events 1. 2 February – World Wetlands Day 2. 22 March – World Water Day 3. 7 April – World Health Day 4. 22 April – World Earth Day Choose the correct code. (1) 1 and 4 only (2) 2, 3 and 4 only (4) 1, 2, 3 and 4 (3) 1, 2 and 3 only (1) Karl Benz

(2) Sir Frank Whittle

(3) Thomas Savery

(4) Michael Faraday

21. Average of 36 results is 18. If 2 is subtracted from each result what will be the new average of the results? (3) 17

(4) 78

23. If an 11-digit number 5y 5884805x6 where x ≠ y is divisible by 72. Then the value of xy is: (1) 7 (2) 3 (3) 7 (4) 2 7 24. Speed of boat along and against the current are 14 km/h and 8 km/h respectively. The speed of the current is (1) 11 h

(2) 6 h

(3) 5.5 h

(4) 3 h

25. If two numbers are in the ratio 2 : 3 and the ratio

(1) 10

(2) 80

(3) 40

(4) 100

26. A man travelled a distance of 80 km in 7 hrs partly on foot at the rate of 8 km per hour and partly on bicycle at 16 km per hour. The distance travelled on the foot is (1) 32 km

(2) 48 km (3) 36 km (4) 44 km

27. The sum of an Infinite geometric series is 4 and the sum of the cubes of the terms of the same GP is 192. The Common Ratio of the original geometric series is: (1) 1/2

(2) –1/2

(3) 1/4

(4) –1/4

28. The least number which is exactly divisible by 4, 5, 8, 10 and 12 is: (1) 150

(1) 2

(2) 240

(3) 180

(4) 120

(4) 17.5

(2) 3

(3) 4

(4) 5

30. The base and altitude of an isosceles triangle are 10 cm and 12 cm respectively. Then the length of each equal side is: (1) 10 cm

(2) 7.5 cm (3) 8.5 cm (4) 13 cm

31. If the perimeter of an isosceles right triangle is ( 8 2 + 1) cm, then the length of the hypotenuse of the triangle is: (1) 8 cm

20. Who invented the jet engine?

(2) 16

(3) 79

29. If 9 x  12  147 then x = ?

19. Which of the following is/are correctly matched?

(1) 9

(2) 79.35

becomes 3 : 4 when 8 is added to both the numbers, then the sum of the two numbers is

15. Who, among the following individuals, has

(1) Radon (3) Chlorine

22. The mean of scores obtained by 50 students is found to be 79.5. Later on, it was found that the score of one student was read as 94 in place of 49 and the score of another student was read as 69 in place of 89. Find the correct mean.

functional in an acidic environment?

(2) 12 cm (3) 10 cm

(4) 24 cm

32. The fourth root of 24010000 is (1) 7

(2) 49

(3) 490

(4) 70

33. Find the wrong number in the following number series. 3 7 16 35 70 153 (1) 70

(2) 16

(3) 153

(4) 35

MOCK TEST Paper-6

41

34. Simplify:

43. Two statements are given, followed by three conclusions numbered I, II and III.

 0.73    0.27  2 2  0.73    0.27    0.73    0.27  3

(1) 1

3

(2) 0.4047 (3) 0.73

(4) 0.27

35. When a discount of 25% is given on a cruise trip,

Assuming the statements to be true, even if they seem to be at variance with commonly known facts, decide which of the conclusions logically follow(s) from statements.

the profit is 41%. If the discount is 26%, then the profit is (2) 67 per cent (1) 39.12 per cent

Statements:

(3) 94.88 per cent

Conclusions:

(4) 11.24 per cent



I. All photographs are newspapers.



II. Some photographs are computers.

22 ) 7 (1) 10.5 metres

(2) 7 metres

I. Some computers are photographs. II. Some computers are newspapers. III. Some newspapers are photographs. (1) All of the conclusions follow. (2) Only conclusions I and II follow. (3) Only conclusions II and III follow. (4) Only conclusions I and III follow.

(3) 5.25 metres

(4) 21 metres

44. Select the correct option that will fill in the blank

36. A circular road runs around a circular ground. If

the difference between the circumference of the outer circle and the inner circles is 66 metres, the width of the road is:

(Take π =

37. If the area of a square is 24 sq units then what is the perimeter of the square?

(1) 2 6

(2) 4 6

and complete the series. Mhz, Ngy, Ofx, Pew, ............

(3) 16 6

(4) 8 6

38. Two toys are sold at ` 504 each. One toy brings the

(1) Qrm

(2) Rrn

(3) Qdv

(4) Qqn

45. Find the number of rectangles in the below figure

dealer a gain of 12% and the other a loss of 4%. The gain or loss per cent by selling both the toys is

(1) 3 5 % Profit 13 1 (3) 5 % Profit 13

(2) 4 (4) 2

5 % Profit 13

3 % loss 13

39. A father can do a job as fast as his two sons working

together. If one son does the job in 3 hours and the other in 6 hours, the number of hours taken by the father, to do the job alone is

(1) 1

(2) 2

(3) 3

(4) 4

40. In what time will `8,000, at 3% per annum, produce

the same interest as `6,000 does in 5 years at 4% simple interest? (1) 5 years (2) 6 years (3) 3 years (4) 4 years

41. Identify the correct option to complete the following series. 1, 3, 5, 11, 21, 43, 85, 171, ............. (1) 354 (2) 247 (3) 341

(4) 342

42. Choose the Venn diagram that best illustrates the relationship among the following classes:

Women, Entrepreneurs, Engineers (1)

(3)





(2)

(4)

(1) 25

(2) 125

(3) 225

(4) 55

46. Select the potion in which the words share the

same relationship as that shared by the given pair of words. Hive : Bee (2) Eyrie : Bear (1) Burrow : Hare (3) Sty : Dog (4) Stable : Cow 47. In a certain code language, ‘apple’ is called ‘pear’, ‘pear’ is called ‘orange’, ‘orange’ is called ‘guava’ and ‘guava’ is called ‘melon’. In this language, which one of the following will be a citrus fruit? (1) Pear (2) Melon (3) Guava (4) Orange 48. Hitesh, Sunny, Vicky, Nitin and Bharat are arranged in ascending order of the height from the top. Hitesh is at third place. Bharat is between Nitin and Hitesh while Nitin is not at the bottom. Who has the maximum height among them? (1) Hitesh (2) Sunny (3) Vicky (4) Nitin 49. Select the option in which the numbers are related in the same way as are the numbers in the given set. (269, 278, 296) (1) (109, 118, 128) (2) (577, 586, 598) (3) (313, 322, 340) (4) (419, 430, 448)

OSWAAL CUET (UG ) 10 Mock Test Papers GENERAL TEST

42

50. In a code language, if ‘you are there’ is written as

‘ter der jer’, ‘we stay here’ is written as ‘yer mer ner’, ‘we are late’ is written as ‘ser ner der’, and ‘I stay there’ is written as ‘yer fer jer’, then how would ‘you stay late’ be written in this language? (1) ter yer ser (2) ter yer mer (3) der yer ser (4) ter mer ser 51. Which answer figure will complete the pattern in the question figure? Question Figure:

?

Answer Figures:

(1)



(2)

(3)



(4)

52. In a dice, 1, 2, 3, and 4 are written on the adjacent faces, in a clockwise order, and 5 and 6 at the top and bottom. When three is at the top, what will be at the bottom? 5 4

1 (1) 4

(2) 2

(3) 6

(4) 1

53. Select the option that will correctly replace the question mark (?) in the series. RMB, POZ, NQX, LSV, ? (1) JUT (2) JVT (3) JUS

(4) JVS

54. Find out which of the answer figures can be formed

from the pieces given in problem figure. Question figure

Answer figures (1)



(2)

(3)

(4)

55. If ‘÷‘ means ‘+‘, ‘−‘ means ‘ב, ‘ב means ‘−‘, and ‘+‘ means ‘÷‘, then: 664 + 4 ÷ 34 × 28 =? (1) 165 (2) 162 (3) 172 (4) 185 56. In the following question, two statements are given each followed by two conclusions I and II. You have to consider the statements to be true even if they seem to be at variance from commonly known facts. You have to decide which of the given conclusions, if any, follows from the given statements. Statements: (I) Corruption has complete establishment in each sector of our country. (II) Corruption has gone into their deep roots of every process and system. Conclusions: (I) People think about corruption in a routine way and not especially. (II) The eradication of corruption depends upon every individual. No government can enforce rules or regulation on people. (1) Only conclusion II follows. (2) Conclusion I and II both follow. (3) Neither I nor II follow. (4) Only conclusion I follow. 57. Select the related word pair from the given alternatives. Celsius : Temperature :: ........... : .......... (1) Kilometer : Distance (2) Bank note : Office (3) Meter : Water (4) Bulb : Electric current 58. There are five students - P, Q, R, S and T having different heights in a class. P’s height is more than only one student. Q’s height is more than S and P but not more than R. S’s height is more than P. R is not the smallest. Who is having the maximum height in the class. (1) Q (2) R (3) S (4) T 59. A%B’ means ‘A is the mother of B’. ‘A $ B’ means ‘A is the father of B’. ‘A #B’ means ‘A is the brother of B’. ‘A & B’ means ‘A is the sister of B’. If J $ H # R % N & T # U % P, then which of the following statements is NOT correct? (1) J is the maternal grandfather of N. (2) R is the maternal grandfather of P. (3) R is the mother of U. (4) N is the sister of P.

MOCK TEST Paper-6

43

60. Arrange the following words in the correct order (a) Omelete

(b) Hen

(c) roll

(d) serve

Choose the correct answer from the options given below:

(e) egg

(1) b, e, a, d, c

(2) e, b, a, c, d

(3) b, e, a, c, d

(4) b, a, e, c, d

 Answer Key 1.

(4)

2.

(1)

3.

(4)

4.

(2)

5.

(3)

6.

(2)

7.

(1)

8.

(2)

9.

(2)

10.

(3)

11.

(2)

12.

(4)

13.

(2)

14.

(1)

15.

(4)

16.

(1)

17.

(4)

18.

(2)

19.

(4)

20.

(2)

21.

(2)

22.

(3)

23.

(3)

24.

(4)

25.

(3)

26.

(1)

27.

(2)

28.

(4)

29.

(2)

30.

(4)

31.

(1)

32.

(4)

33.

(1)

34.

(1)

35.

(1)

36.

(1)

37.

(4)

38.

(1)

39.

(2)

40.

(1)

41.

(3)

42.

(3)

43.

(1)

44.

(3)

45.

(1)

46.

(1)

47.

(3)

48.

(4)

49.

(3)

50.

(1)

51.

(1)

52.

(3)

53.

(1)

54.

(1)

55.

(3)

56.

(3)

57.

(1)

58.

(2)

59.

(3)

60.

(1)

OSWAAL CUET (UG ) 10 Mock Test Papers GENERAL TEST

44

MOCK

7

Test Paper Maximum Marks : 250

Time : 60 Minutes

General Instructions : (i) This paper consists of 60 MCQs, attempt any 50 out of 60 (ii) Correct answer or the most appropriate answer: Five marks (+5) (iii) Any incorrect option marked will be given minus one mark (-1) (iv) Unanswered/Marked for Review will be given no mark (0) (v) If more than one option is found to be correct then Five marks (+5) will be awarded to only those who have marked any of the correct options (vi) If all options are found to be correct then Five marks (+5) will be awarded to all those who have attempted the question. (vii) Calculator / any electronic gadgets are not permitted.

1. Which of the following is/are correctly matched? Chinese traveller

visited the court

1. Fa Hein

Chandragupta II

2. Hiuen Tsang

Harsha Vardhana

3. I-Tsing

Chandragupta I

Choose the correct code (1) 2 only

(2) 1 and 3 only

(3) 1 and 2 only

(4) 1, 2 and 3

2. Which of the following is/are not correctly matched? Territories

Year of Creations

1. Puducherry

1954

2. Dadra and Nagar Haveli

1961

3. Lakshadweep

1956

4. Delhi

1969

Choose the correct code. (1) 1 only

(2) 4 only

(3) 1 and 4 only

(4) 2 and 3 only

3. Which of the following statement is/are correct? 1. Clement Attlee held the position of Prime Minister of England at the time of India's independence. 2. Within the context of India's Freedom Movement, James Ramsay MacDonald made the declaration of the Communal Award in 1932. Choose the correct code.

4. Which type of federalism is seen in India? (1) Coming together federation (2) Holding together federation (3)  Coming together federation and Holding together federation both (4) None of these

5. Which of the following is/are correctly matched? Articles Provisions 1. Article 360

Financial Provisions

2. Article 324

Election Commission of India

3. Article 161

Pardoning powers of Governor

4. Article 143

Advisory jurisdiction of the Supreme Court

Choose the correct code. (1) 1 and 4 only

(2) 2 and 3 only

(3) 1,2 and 4 only

(4) 1, 2, 3 and 4

6. Which of the following seas is Earth’s lowest elevation on land and is located at the border of Israel and Jordan? (1) Red Sea

(2) Black Sea

(3) Dead Sea

(4) Arabian Sea

7. To which of the following Indian states does the tribe of ‘Nyishi’ belong? (1) Tamil Nadu

(2) Bihar

(3) Arunachal Pradesh (4) Chhattisgarh

8. Which of the following is/are correctly matched? Tribe Locations 1. Masai Kenya 2. Bedouin

Arabia

(1) 1 only

(2) 2 only

3. Hausa

Northern Nigeria

(3) Both 1 and 2

(4) Neither 1 nor 2

4. Bushmen

China

MOCK TEST Paper-7

45 16. Which of the following reaction is an example of

Choose the correct code. (1) 1 and 2 only

(2) 2 and 3 only

(3) 1, 2 and 3 only

(4) 1, 2, 3 and 4

9. When the fiscal deficit is high, what happens to prices? (1) Prices decrease (2) There is no direct impact on prices

exothermic reaction? (1) Burning of natural gas (2) Respiration The decomposition of vegetable matter into (3)  compost (4) All Options are correct

17. What is the chemical name of the deadly ‘cyanide’?

(3) Prices remain stable (4) Prices increase

10. Which of the following is NOT a logic gate? (1) AND

(2) OR

(3) NAND

(4) MAD

(1) Sulfuric acid (3) Nitric acid

(2) Hydrochloric acid (4) Prussic acid

18. Which of the following pair is/are correctly matched?

11. All the arithmetic and logical operations in a computer are done by __________ (1) ALU

(2) CU

(3) Register

(4) No option is correct

12. Amino acids such as citrulline, ornithine, and

Instruments

Functions

1. Sonometer

It is used to measure blood pressure.

2. Hydrometer

It is an instrument that measures that specific gravity (relative density) of liquids.

3. Sphygmomanometer

The instrument used to study the laws of vibrating string.

4. Electrometer

It is used to measure electric charge or electrical potential difference.

gamma-aminobutyric acid are referred to as: (1) non-protein amino acids (2) essential amino acids (3) protein amino acids (4) acidic amino acids

13. Which of the following is/are correct? 1. The pancreas releases enzymes that assist in the process of digestion and also produce hormones that play a role in regulating sugar metabolism. 2. The main function of the liver is the production of bile. 3. The small intestine is the principal organ for the absorption of nutrients. 4. Absorption of water, some minerals, and drugs takes place in the large intestine. Choose the correct code.

Choose the correct code. (1) 1 and 3 only

(2) 2 and 4 only

(3) 1 and 2 only

(4) 1, 2, 3 and 4

19. Which of the following is not a folk dance belonging to the union territory of Jammu and Kashmir?

(1) 2 and 3 only

(2) 2 and 4 only

(1) Dumhal

(2) Hafiza

(3) 2, 3 and 4 only

(4) 1,2, 3 and 4

(3) Rouf

(4) Dangi

14. Which one among the following administers the International Development Association (IDA)? (1) UNDP

(2) UNIDO (3) IFAD

(4) IBRD

15. Which of the following is/are not correctly matched? 1. Padma Bhushan: Third-highest civilian honour in India.

20. Who is the first Indian Gold Medal Winner in individual category in Olympic Games? (1) Abhinav Bindra

(2) Sushil Kumar

(3) R. V. S. Rathore

(4) Leander Paes

21. Which of the following successive discounts is better to a customer?

2. Bharat Ratna: The highest civilian award in India.

(a) 20%, 15%, 10%

3. Padma Vibhushan: It is the second-highest civilian honour in India.

(2) (b) is better

4. Padma Shri: It is the fourth-highest civilian honour in India. Choose the correct option.

(b) 25%, 12%, 8%

(1) (a) is better (3) (a) or (b) (both are same) (4) None of these

22. If the mean of 5 observations x, x + 2, x + 4, x + 6

(1) 2 and 3 only

(2) 1 and 4 only

and x + 8 is 11, then value of x:

(3) 2 and 4 only

(4) None of the above

(1) 7

(2) 43

(3) 51

(4) 8

OSWAAL CUET (UG ) 10 Mock Test Papers GENERAL TEST

46

23. If a certain sum becomes two times in 5 years at

compound interest, then in how many years, it will become eight times? (1) 10 (2) 20 (3) 15 (4) 30

24. What is the least value of x such that 517x324 is divisible by 12?

(2) 1

(1) 3

(3) 0

(4) 2

25. A bag contains 6 red, 4 blue, and 10 white balls. A

ball is picked from the bag at random What is the probability that it is neither white nor blue? (2) 3/10

(1) 20/6

(3) 2/5

(4) 1/2

26. The average weight of a class of 50 students is 48.6 kg. If the average weight of the 20 boys is 54 kg. Then find the average weight (in kg) of the girls in the class.

(1) 40

(2) 46

(3) 45

(4) 42

27. What is the least number which when divided by 15, 18 and 36 leaves the same remainder 9 in each case and is divisible by 11?

(1) 1269

(2) 1089

(3) 1080

(4) 1071

28. 18 persons working 8 hours a day can complete 3

units of works in 10 days. How many persons are required to complete 5 units of that work in 16 days working 6 hours a day? (2) 15

(1) 25

(3) 20

(4) 9

29. If a + 2b + a − 2b = 3 , then a : b is equal to a + 2b − a − 2b

(1) 2 : 3

(2)

3 : 4 (3)

3 : 2 (4) 4 : 3

30. ∆ABC is an equilateral triangle. O is the point of intersection of altitudes AL, BM and CN. If OA = 16 cm, what is the semi-perimeter (in cm) of the triangle ABC? (2) 12 3 (3) 16 3

(1) 8 3

(4) 24 3

31. Twenty one times of a positive number is less that its square by 100. The value of the positive number is (1) 25 (2) 26 (3) 42 (4) 41

1

32. If 2 x  a  x2 − 1 x − x2 − 1

a

and a  0 , then the value of

is 1  a  1 2 (4) a – 1

(1) a + 1

(2)

1  a  1 2 33. The table shows the production of different types of cars (in thousands). Study the table carefully and answer the questions. (3)

Years/ Cars A B C D E

2012

2013

2014

2015

2016

30 42 48 51 20

35 48 36 24 42

48 40 38 30 40

45 38 35 46 35

56 56 44 54 43

If the data related to the production of cars of type E is represented by a pie chart, then the central angle of the sector representing the data of production of cars in 2013 will be: (2) 84 (3) 70 (4) 80 (1) 102 34. The radius of a wheel is 21 cm. What is the distance (in cm) travelled by the wheel in 10 revolutions? (1) 660 (2) 1320 (3) 1980 (4) 2640 35. In a triangle, if the measures of two sides are 5 cm and 8 cm then the third side can be: (1) 3 cm (2) 4 cm (3) 14 cm (4) 2 cm 36. A man bought oranges at the rate of 8 for ` 34 and sold them at the rate of 12 for ` 57. How many oranges should be sold to earn a net profit of ` 45? (2) 100 (3) 135 (4) 150 (1) 90 37. A person bought 50 pens for ` 50 each. He sold 40 of them at a loss of 5%. He wants to gain 10% on the whole. Then his gain percent on the remaining pens should be (1) 15% (2) 40% (3) 50% (4) 70% 38. Two vessels contain milk and water in the ratio 3 : 2 and 7 : 3. Find the ratio in which the contents of the two vessels have to be mixed to get a new mixture in which the ratio of milk and water is 2 : 1. (1) 2 : 1 (2) 1 : 2 (3) 4 : 1 (4) 1 : 4 39. The sum of three numbers is 280. If the ratio between the first and second numbers is 2 : 3 and the ratio between second and third numbers is 4 : 5, then find the second number. (1) 80 (2) 90 (3) 86 (4) 96 40. In simple interest rate per annum a certain sum amounts to ` 5,182 in 2 years and ` 5,832 in 3 years. The principal in rupees is (1) ` 2882 (2) ` 5000 (3) ` 3882 (4) ` 4000 41. A + B means ‘B is the daughter of A’. A – B means ‘B is the sister of A’. A × B means ‘B is the husband of A’. A ÷ B means ‘A is the father of B’. If P ÷ R × T + Q – S × U + Z, Then how is R related to Z? (1) Paternal grandfather (2) Maternal grandfather (3) Paternal grandmother (4) Maternal grandmother 42. Choose the Venn diagram that best illustrates the relationship among the following classes: Crocodiles, Aquatic animals, Reptiles (1)

(2)

(4)

(3)



MOCK TEST Paper-7

47

43. Two statements are given, followed by two

conclusions numbered I and II. Assuming the statements to be true, even if they seem to be at variance with commonly known facts, decide which of the conclusions logically follow(s) from statements. Statements: I. No plant is a tree. II. All ornaments are plants. Conclusions: I. No ornament is a tree. II. Some plants are ornaments. (1) Both conclusions follow. (2) Only conclusion I follows. (3) Only conclusion II follows. (4) Either conclusion I or II follows.

51. In a certain code language. ‘YOU TOOK MY PLACE IN’ is written as ‘COU POON LC QVAKE IJ’. How will ‘COMPILE’ be written in that language? (3) KOLTYLE

(4) YOLTILE

transparent sheet will be folded.

a circle, corrupt represents a triangle, the poet represents a square, and married represents a rectangle. The number representing unmarried police officers who are not corrupt but are poets is 11

(1)



(2)

(3)



(4)

10 5 13 4 12 3 8

2

53. In the following question, two statements are

6 7

9 (1) 8

(2) KOLQIVE

52. Which image will appear when the following

44. In the following diagram, the police represent

1

(1) KOLQIME

(2) 9

(3) 2

(4) 4

45. Time appears in the mirror at 11 : 09. Then, what time it will appear on the clock? (2) 12 : 09 (3) 12 : 51 (4) 1 : 09 (1) 1 : 51

46. If ‘L’ means ‘×’, ‘N’ means ‘+’, ‘O’ means ‘÷’, and ‘M’ means ‘−’, then: 1000 O 5 N 66 M 33 = ? (1) 433 (2) 333 (3) 483 (4) 233

47. Beverages’ is related to ‘Tea’, in the same way as ‘Medicine’ is related to (1) Analgesics

(2) Disease

(3) Doctor

(4) Hospital

48. In a code language, if ‘fast and furious’ is written as ‘co mo jo’, ‘do it fast’ is written as ‘cha mo ga’, and ‘she did it’ is written as ‘ga la nop’, then what will be the code for the word ‘do’ in this language? (1) ga (2) mo (3) nop (4) cha

49. In a row of 74 girls, Shweta is 27th from the left end.

given each followed by two conclusions I and II. You have to consider the statements to be true even if they seem to be at variance from commonly known facts. You have to decide which of the given conclusions, if any, follows from the given statements. Statements: (I)  Wages are determined by minimum wage legislation in each country. (II) Increase in wage rate is the sign of growth in an economy. Conclusions: (I) Average wage rate signifies the wealth of a country. (II) M  inimum wage legislation does not account wage imbalances. (1) Only conclusion II follows. (2) Conclusion I and II both follow. (3) Neither I nor II follow. (4) Only conclusion I follows.

54. Find out which of the answer figures can be formed from the pieces given in problem figure. Question figure

Palak is 7th to the right of Shweta. What is Palak’s position from the right end of the row? (1) 40 (2) 41 (3) 42 (4) 44

50. Select the option in which the numbers do not share the same relationship as that shared by the given triad of numbers.

(23, 115, 207) (1) (8, 35, 72)

(2) (6, 30, 54)

(3) (16, 80, 144)

(4) (21, 105, 189)



Answer figures

(1)



(2)

OSWAAL CUET (UG ) 10 Mock Test Papers GENERAL TEST

48

56. Pointing to a lady in the photograph, Amit said, “She (4)



is the mother of the only grandson of my mother”. How is the lady related to Amit? (1) Mother (2) Daughter (4) Wife (3) Niece

(4)



55. Which answer figure will complete the pattern in

the question figure? Question Figure:



57. Sunita travels 6 km towards North-East. Then travels 9 km towards West. From there, she goes 6 km SouthWest and finally goes 3 km towards East. How far is she from her initial position? (1) 3 km (2) 6 km (3) 9 km (4) 12 km

58. Select the pair of letters that will follow next in the

?

given series. AB, GH, MN, ST, ? (1) YZ (2) VW

Answer Figures:

(3) UV

(4) XY

59. Pearl Tower is taller than Sky Tower but shorter (1)

than Unity Tower. Unity Tower and Cyber Tower are of the same height. Pearl Tower is shorter than Indus Tower. Amongst the buildings, which is the second shortest? (1) Pearl Tower (2) Sky Tower (3) Indus Tower (4) Unity Tower

(2)



60. Select the correct option that will fill in the blank (3)

and complete the series. 12, 12, 13, 26, 28, 84, ? (1) 93 (2) 87 (3) 84

(4)



(4) 95

 Answer Key 1.

(3)

2.

(2)

3.

(3)

4.

(2)

5.

(3)

6.

(3)

7.

(3)

8.

(3)

9.

(2)

10.

(4)

11.

(1)

12.

(1)

13.

(4)

14.

(4)

15.

(4)

16.

(4)

17.

(4)

18.

(2)

19.

(4)

20.

(1)

21.

(2)

22.

(1)

23.

(3)

24.

(4)

25.

(2)

26.

(3)

27.

(2)

28.

(1)

29.

(4)

30.

(4)

31.

(1)

32.

(3)

33.

(2)

34.

(2)

35.

(2)

36.

(1)

37.

(4)

38.

(2)

39.

(4)

40.

(3)

41.

(4)

42.

(2)

43.

(1)

44.

(1)

45.

(3)

46.

(4)

47.

(1)

48.

(4)

49.

(2)

50.

(1)

51.

(2)

52.

(3)

53.

(4)

54.

(2)

55.

(2)

56.

(4)

57.

(2)

58.

(1)

59.

(1)

60.

(2)

MOCK TEST Paper-1

49

MOCK

8

Test Paper Maximum Marks : 250

Time : 60 Minutes

General Instructions : (i) This paper consists of 60 MCQs, attempt any 50 out of 60 (ii) Correct answer or the most appropriate answer: Five marks (+5) (iii) Any incorrect option marked will be given minus one mark (-1) (iv) Unanswered/Marked for Review will be given no mark (0) (v) If more than one option is found to be correct then Five marks (+5) will be awarded to only those who have marked any of the correct options (vi) If all options are found to be correct then Five marks (+5) will be awarded to all those who have attempted the question. (vii) Calculator / any electronic gadgets are not permitted.

1. Which of the following is not correctly matched? Vedic River Name

Modern River Name

1. Shutudri Sutlej 2. Parushini

Ravi

3. Askini

Chenab

4. Vitasta

Beas

1. Best Feature Film: Rocketry 2. Best Popular Film Providing Wholesome Entertainment: RRR 3. Best Actor: Allu Arjun, Pushpa 4. Best Actress: Alia Bhatt, Gangubai Kathiawadi and Kriti Sanon, Mimi Choose the correct code.

Choose the correct code. (1) 1 only

(2) 2 and 4 only

(3) 4 only

(4) 2, 3 and 4 only

(1) 1 only

(2) 2, 3, and 4 only

(3) 1, 2 and 4 only

(4) 1, 2, 3 and 4

2. Which of the following groups of countries is

5. What is the government’s goal under the “lakhpati

a member of the South Asian Association for Regional Cooperation (SAARC) and Bay of Bengal Initiative for Multi-Sectoral Technical and Economic Cooperation (BIMSTEC)?

(1) To create two crore prosperous sisters in villages.

(1) Bangladesh, Bhutan, India

(3) To help women earn over ` 1 lakh per year.

(2) Bangladesh, Maldives, India

(4) All of the above.

(2) To provide skill training to women in villages.

6. ____________ expansion makes the Eiffel Tower

(3) Pakistan, Bhutan, India (4) Afghanistan - Bhutan - Maldives

3. Which of the following is correctly matched? First Chairman

Constitutional Bodies

didis” scheme?

1. Election Commission of India Sukumar Sen

taller during summers. (1) Thermal

(2) Gradient

(3) Gravitational

(4) Chemical

7. Among all the millets, which of the following is the

2. Finance Commission of India K. C. Niyogi

most abundantly cultivated in India?

3. Comptroller General of India V. Narhari Rao

(1) Pearl millet

(2) Sorghum

4. Attorney General of India

(3) Ragi

(4) Foxtail millet

M. C. Stelvad

Choose the correct code. (1) 1, 2 and 4 only

(2) 2 and 4 only

(3) 1, 3 and 4 only

(4) 1, 2, 3 and 4

8. Which of the following is the largest fish-exporting region of the world? (1) The north-east Atlantic region

4. Which of the following pair is/are correctly

(2) The north-east Pacific region

matched regarding the 69th National Film Awards 2023?

(4) The south-east Asian region

(3) The north-west Pacific region

OSWAAL CUET (UG ) 10 Mock Test Papers GENERAL TEST

50

9. Which of the following pair is/are correctly matched? Tribes Native State 1. Hatti tribe Himachal Pradesh 2. Narikoravan tribe Tamilnau 3. Toda tribe Tamilnadu 4. Binjhia tribe Jharkhand Choose the correct code. (1) 1 and 2 only (2) 2 and 4 only (3) 1, 2, and 4 only (4) 1, 2, 3 and 4 10. Which among the following is used to transmit data in optical fibre? (1) Vibrations (2) Light (3) Sound (4) Electrical current 11. Which among the following is a male part of a flower? (1) Stigma (2) Stamen (3) Style (4) Pistil 12. Which of the following is/are not correctly matched? Dates Important Days 1. 10 January World Hindi Day National Youth Day 2. 12 January National Girl Child Day 3. 24 January 4. 30 January Martyrs Day or Shaheed Diwas Choose the correct code. (1) 1 and 2 only (2) 1, 2, 3 and 4 (3) 2 and 4 only (4) None of the above 13. Who among the following is the first Indian to swim across English Channel? (2) Mihir Sen (1) Aarti Saha (3) P. K. Bannerji (4) Vikram Merchant 14. Match the following Scientist

Discovery

1. Louis Pasteur

A. Fermentation

2. Edward Jenner

B. Small Pox Vaccine

3. James Watson and Francis Crick

C. DNA Structure

4. Alexander Fleming

D. Penicillin

Choose the correct code. (1) 1-A 2-B 3-C 4-D (2) 1-B 2-A 3-C 4-D (3) 1-D 2-C 3-A 4-B (4) 1-C 2-A 3-B 4-D

15. Match the following:

Choose the correct code. (1) 1-A 2-B 3-C 4-D

(2) 1-B 2-A 3-C 4-D

(3) 1-A 2-D 3-C 4-B

(4) 1-C 2-A 3-B 4-D

16. Marfati songs are traditional folk songs of ____________ (1) Afghanistan

(2) Bangladesh

(3) Pakistan

(4) Nepal

17. Veteran freedom fighter, social reformer and feminist Savitribai Phule hailed from which of the following states of India? (1) Rajasthan

(2) Gujarat

(3) Maharashtra

(4) Odisha

18. In which of the following state Archaeological Survey of India discover a 1300-year-old Buddhist Stupa? (1) Bihar

(2) Jharkhand

(3) Odisha

(4) Madhya Pradesh

19. Which of the following substances is used as a substitute for chlorine in bleaching? (1) Ethanal (2) Liquid carbon dioxide (3) Hydrogen peroxide (4) Tetra chloroethene

20. Which of the following metals in pure form has the highest melting point? (1) Tungsten

(2) Copper

(3) Platinum

(4) Gold

21. In what proportion must a grocer mix sugar at ` 12 per kg and ` 7 per kg so as to make a mixture worth ` 8 per kg? (1) 7 : 12

(2) 1 : 4

(3) 2 : 3

(4) 12 : 7

22. Please read the following carefully and answer the questions: The marks of 7 students in a unit test are given below 9, 10, 7, 6, 9, 3, 5 Median of the data is. (1) 6

(2) 10

(3) 9

(4) 7

23. The single discount equal to three consecu­tive discounts of 10%, 12% and 5% is (1) 26.27%

(2) 24.76% (3) 9%

(4) 11%

24. If the six digit number 4x573y is divisible by 72 then the value of (x + y) is: (1) 9

(2) 4

(3) 8

(4) 6

25. A sum of ` 2000 amounts to ` 4000 in two years

Leaders

Nicknames

1. Bal Gangadhar

A. Tilak Father of Indian Unrest

2. Dr Rajendra Prasad

B. Deshbandhu

3. Dadabhai Naoroji

C. Grand old man of India

getting the sum of numbers on the two faces an odd-prime number?

4. Chittaranjan Das

D. Desh Ratna

(1) 7/18

at compound interest. In how many years will the same amount become ` 8000? (1) 2

(2) 4

(3) 6

(4) 8

26. A pair of dice is thrown. What is the probability of

(2) 1/6

(3) 1/2

(4) 5/12

MOCK TEST Paper-8 27. If 1 

51 37. Let ABC be an equilateral triangle and AX, BY, CZ

x3 3  , then x equals 100 5

(1) 2

(2) 4

(3) 16

(4) (136)1/3

28. The table shows the production of different types

of cars by a company (in thousands) in 5 years. Study the table and answer the questions. Car/ Year 2014 2015 2016 2017 2018

A

B

C

D

E

52 47 48 43 38

54 45 47 50 40

48 53 56 57 54

46 50 54 67 68

64 45 65 63 70

If the data related to the production of cars in 2018

is represented by pie chart then the central angle of the sector representing the production of type C cars will be:

(1) 72°

(2) 59°

(3) 93°

itself. What is the number? (2) 900

(3) 1500

(4) 1200

30. The greatest number of four digits which when divided by 3, 5, 7, 9 leave remainders 1, 3, 5, 7 respectively is: (1) 9763

(2) 9764

(3) 9766

(4) 9765

31. What is the sum of the first 17 terms of an arithmetic progression if the first term is –20 and last term is 28? (1) 68

(2) 156

(3) 142

(3) AX = BY ≠ CZ

(4) 242

(4) AX ≠ BY ≠ CZ

38. ABC is an equilateral triangle and CD is the internal bisector of ∠C. If DC is produced to E such that AC = CE, then ∠CAE is equal to (2) 75°

(1) 45°

(3) 30°

(4) 15°

39. A can complete a piece of work in 18 days, B

in 20 days and C in 30 days. B and C together start the work and are forced to leave after 2 days. The time taken by A alone to complete the remaining work is (1) 10 days (2) 12 days (3) 15 days

(4) 16 days

40. If N = 7 − 3 , then what is the value of N + 1 ? N

7+ 3

(4) 91°

29. A number is increased by 84, it becomes 107% of (1) 600

be the altitudes. Then the right statement out of the four given responses is (1) AX = BY = CZ (2) AX ≠ BY = CZ

(1) 2 2

(2) 5

(3) 10

(4) 13

41. Select the term that will come next in the following series. a, cd, fgh, jklm, opqrs, ........ (1) uvwxyz (2) vwxyza (3) ouvwsy (4) tuvwxy

42. Select the Venn diagram that best illustrates the relationship between the following classes: Graduates, Teachers, Literates

(2)

(1)

32. A person has to travel a distance of 30 km. He finds

5 that he has covered th of the distance in 3 hours 6 is his speed in kmph? and 20 minutes. What

(1) 5.4

(2) 7.5

(3) 6.3

(4) 6

33. A circular wire of length 168 cm is cut and bent in the form of a rectangle whose sides are in the ratio of 5 : 7. What is the length (in cm) of the diagonal of the rectangle? 4127 (2) 3137 (3) 1813 (4) 3626 34. A sold a horse to B for ` 4800 by losing 20%. B sells it to C at a price which would have given A a profit of 15%. B’s gain is (1)

(1) ` 1800

(2) ` 1900 (3) ` 2000 (4) ` 2100

35. A man sold an article at a gain of 5%. Had he sold it for ` 240 more, he would have gained 8%. The cost price of the article is (1) ` 6,000

(2) ` 10,000

(3) ` 12,000

(4) ` 8,000

36. The number to be added to each of the numbers 7, 16, 43, 79 to make the numbers in proportion is (1) 2

(2) 3

(3) 5

(4) 1



(3)



(4)

43. Two statements are given, followed by three

conclusions numbered I, II and II. Assuming the statements to be true, even if they seem to be at variance with commonly known facts, decide which of the conclusions logically follow(s) from statements. Statements: I. No tree is a plum. II. All pomegranates are trees. Conclusions: I. No pomegranate is a plum. II. No plum is a tree. III. Some trees are pomegranates. (1) Only conclusions I and III follow. (2) Only conclusions II and III follow. (3) All the conclusions follow. (4) Only conclusions I and II follow.

OSWAAL CUET (UG ) 10 Mock Test Papers GENERAL TEST

52

44. Five monkeys A, B, C, D and E are sitting on a

branch of a tree. C is sitting next to D. D is not sitting with E. E is on the left end of the branch. C is on second position from right. A is immediate right of B and B is on the right side of E. A and are sitting together. Who is sitting at the centre? (2) C (3) A (4) B (1) D



50. In a class, P has more marks than Q and R does not have the least marks. S has more marks than T and T has more marks than P, who among them will have the least marks? (1) P

46. In the following question, two statements are

given each followed by two conclusions I and II. You have to consider the statements to be true even if they seem to be at variance from commonly known facts. You have to decide which of the given conclusions, if any, follows from the given statements.

Statements:

Cricket

Hockey (1) 17

(II)  Cashless transactions will further curb the black marketing of money in upcoming time. (1) Only conclusion II follows (2) Conclusion I and II both follow (3) Neither I nor II follow (4) Only conclusion I follows

Tennis

9

17 12 15 21 6

8

(2) 32

(3) 15

(4) 27

52. In a certain code language. ‘DRONE’ is written as ‘SERFO’. How will ‘HOUSE’ be written as in that language? (1) PIXFT

(2) QJXGB

(3) PQWGS

(4) PIVFT

53. Rohan walks a distance of 3 km towards North, then turns to his left and walks for 2 km. He again turns left and walks for 3 km. At this point he turns to his left and walks for 3 km. How far is he from the starting point?

(II) EPS will enable people to go cashless and use plastic money for most of their transactions. (I) EPS is quick, safe, and easy way of transaction of money from one account to another.

(4) T

How many people like Cricket and Tennis both?

(I) Demonetization has evolved need for learning electronic payment system (EPS).

Conclusions:

(3) S

51. Read details carefully for questions

45. Which of the four changes given in the options

would make the following equation correct? 30 × 3 − 3 = 13 (1) × Interchanged with − (2) × changed to ÷; − changed to + (3) × changed to +; − changed to ÷ (4) × Interchanged with −; 3 interchanged with 30

(2) Q

(1) 5 km

(2) 3 km

(3) 2 km

(4) 1 km

54. Identify the correct option to complete the following series. 5, 8, 13, 20, 29, ........., 53, 68, 85 (1) 40

(2) 39

(3) 35

(4) 36

55. Which answer figure will complete the pattern in the question figure?

Question Figure:

47. Select the correct option that will fill in the blank and complete the series.

AA, BD, CI, DP, ...... (2) FR (1) EY

(3) ER

(4) FY

48. What is the meaningful/logical order of the words

given below? (a) Type (b) Save (c) Open (d) Print (e) Close Choose the correct answer from the options given below. (1) (c), (a), (d), (b), (e) (2) (a), (c), (b), (d), (e) (3) (c), (a), (b), (d), (e) (4) (a), (c), (d), (b), (e)

49. In a certain code language, ‘FIGURE’ is written as

‘DJEVPF’. How will ‘DECODE’ be written as in that language? (1) BFAPBF (2) BFDNBF (3) BDDNBD (4) BDANBD

Answer Figures:

(1)



(2)

(3)



(4)

56. Find out which of the answer figures can be formed from the pieces given in problem figure.

MOCK TEST Paper-8

53

Question figure

58. Rana walks 20 metre straight in North direction and 20 m to the right. Then every times turning to his left he walks 5, 25 and 25 metres respectively. How far is he from his starting point now?

Answer figure (1)

(1) 5 metre

(2) 20 metre

(3) 25 metre

(4) 30 metre

59. A + B means ‘B is the sister of A’. A – B means ’A is the father of B’.

(2)



A × B means ‘B is the son of A’. A ÷ B means ‘B is the husband of A’.

(3)

If E ÷ F – H + J × G + D, then how is D related to H? (1) Aunt (2) Father (3) Uncle (4) Niece

(4)



60. Select the option that is related to the third term in the same way as the second term is related to the first term.

57. Introducing Kavi, Veena Said, “She is the sister of the son of the wife of my husband”. How is Veena related to Kavi?

India : Country :: Delhi : ..........

(1) Daughter

(2) Sister

(1) Continent (2) District

(3) Mother

(4) Aunt

(3) Capital

(4) Red Fort

 Answer Key 1.

(3)

2.

(1)

3.

(1)

4.

(4)

5.

(4)

6.

(1)

7.

(1)

8.

(1)

9.

(4)

10.

(2)

11.

(2)

12.

(4)

13.

(2)

14.

(1)

15.

(3)

16.

(2)

17.

(3)

18.

(3)

19.

(3)

20.

(1)

21.

(2)

22.

(4)

23.

(2)

24.

(3)

25.

(2)

26.

(1)

27.

(2)

28.

(1)

29.

(4)

30.

(1)

31.

(1)

32.

(2)

33.

(4)

34.

(4)

35.

(4)

36.

(3)

37.

(1)

38.

(4)

39.

(3)

40.

(2)

41.

(1)

42.

(1)

43.

(3)

44.

(3)

45.

(2)

46.

(3)

47.

(1)

48.

(4)

49.

(1)

50.

(2)

51.

(1)

52.

(1)

53.

(4)

54.

(1)

55.

(1)

56.

(2)

57.

(3)

58.

(1)

59.

(4)

60.

(3)

OSWAAL CUET (UG ) 10 Mock Test Papers GENERAL TEST

54

MOCK

9

Test Paper Maximum Marks : 250

Time : 60 Minutes

General Instructions : (i) This paper consists of 60 MCQs, attempt any 50 out of 60 (ii) Correct answer or the most appropriate answer: Five marks (+5) (iii) Any incorrect option marked will be given minus one mark (-1) (iv) Unanswered/Marked for Review will be given no mark (0) (v) If more than one option is found to be correct then Five marks (+5) will be awarded to only those who have marked any of the correct options (vi) If all options are found to be correct then Five marks (+5) will be awarded to all those who have attempted the question. (vii) Calculator / any electronic gadgets are not permitted.

1. Which of the following ruler empire was spread over almost whole of western India which included Sindh, Gujarat, Konkan, Narmada Valley, Malwa and Kathiawar except the Nashik and Pune areas? (1) Rudradaman I (2) Kanishka (3) Vasishka (4) Pushyamitra Sunga

2. Match the following Articles of Indian Constitution

Provisions

1. Article 52

A. There shall be a president of India.

2. Article 74

B. Council of Ministers to aid and advise President.

3. Article 78

4. Article 61

C. Prime Minister’s responsibilities in terms of providing information to the President, etc. D. Procedure for impeachment of the President.

Choose the correct code. (1) 1-A 2-B 3-C 4-D (2) 1-B 2-A 3-C 4-D (3) 1-D 2-C 3-A 4-B (4) 1-C 2-A 3-B 4-D

3. Which of the following article is related to the ordinance in Indian constitution? (1) Article 123 (2) Article 213 (3) Both Article 123 and 213 (4) None of the above

4. Which one of the following organizations headquarter is not located in Paris? (1) United Nations Educational Scientific and Cultural Organisation (UNESCO) (2) Organization for Economic Cooperation and Development (OECD) (3) International Council on Monuments and Sites (ICOMOS) (4) World Intellectual (WIPO)

Property

Organisation

5. Which of the following is not correctly matched? (1) Ukraine – The Bread Basket of Europe (2) Bhutan – Land of the Thunder Dragon (3) North Korea – The Hermit Kingdom (4) Pakistan – Graveyard of Empires

6. According to ___________, pressure is equal to the force divided by the area on which it acts. (1) Hooke’s law (2) Stefan-Boltzmann law (3) Newton’s law (4) None of the above

7. An economic condition when there is one buyer and many sellers is called __________. (1) Monopoly

(2) Oligopoly

(3) Monopsony

(4) Perfect Competition

8. Lavasa, India’s inaugural privately developed hill station, was acquired by Darwin Platform Infrastructure for how many thousand crore rupees? (1) 1.8 thousand crore (2) 1.9 thousand crore (3) 2.0 thousand crore (4) 2.1 thousand crore

MOCK TEST Paper-9

55

9. Which of the following is managed by operating system?

20. Match the following

India among the options below?

Martial Dance Related State 1. Silambam A. Maharashtra 2. Kalaripayattu B. Kerala 3. Thang-Ta C. Manipur 4. Mardani Khel D. Tamil Nadu Choose the correct code. (1) 1-A 2-B 3-C 4-D (2) 1-B 2-A 3-C 4-D (3) 1-D 2-B 3-C 4-A (4) 1-C 2-A 3-B 4-D

(1) Nagappattinam, Chinsura, Machilipatnam

21. The L.C.M of two prime numbers x and y,

I. Memory II. Processor III. Input/Output devices (1) Only I

(2) I, II and II

(3) I and II

(4) Only II

10. Where did the Dutch set up their trading hubs in

(2) Surat, Bharuch, Agra

(x > y) is 161. The value of (3y – x):

(3) Cochin, Ahmedabad, Patna

(1) –2

(3) 1

(4) 2

22. Successive discounts of 10% and 20% are equivalent

(4) All of the above

11. Chepu is a unique cultural symbol of ___________. (1) Bhutan

(2) Myanmar (Burma)

(3) China

(4) Nepal

12. Panama Canal connects which of the following water bodies? (1) Mediterranean Sea and Black Sea (2) Mediterranean Sea and Red Sea (3) Atlantic Ocean and Pacific Ocean (4) Mediterranean Sea and Arctic Ocean

13. Which of the following is not correctly matched? Place

Major Passes

(1) Jammu Kashmir

Khardung pass

(2) Sikkim

Jelep La

(3) Uttarakhand

Muling La

(4) Himachal Pradesh

Bomdi La

14. Identify the part of brain that controls the maintenance of posture balance and equilibrium. (1) Diencephalon

(2) Brainstem

(3) Cerebrum

(4) Cerebellum

15. The structure which helps in the nutrition of the Embryo is (1) Yolk sac

(2) Amniotic membrane

(3) Crypts

(4) Placenta

16. Who among the following is known as the Rocket Women of Chandrayaan 3? (1) Ritu Karidhal

(2) –1

(2) Kalpana Kalahasti

(3) Nandini Harinath (4) Tessy Thomas

17. Who amongst the following invented ‘Chronometer’? (1) John Harrison

(2) William Harvey

(3) Friese Greene

(4) Robert Koch

18. Calcium sulphate dihydrate is commonly known as (1) Gypsum

(2) Glass

(3) Asbestos

(4) Limestone

19. Which of the following is NOT a noble gas? (1) Neon

(2) Hydrogen

(3) Helium

(4) Argon

to a single discount of:

(1) 30%

(2) 15%

(3) 28%

(4) 12%

23. Two racers run at the speeds of 100 metres/minute

and 120 metres/minute. If the second racer takes 10 minutes less than the first to complete the run, how long is the race?

(1) 1 km

(2) 6 km

24. Find the A.P. whose 72. (1) 6, 12, 18....

3rd

(3) 12, 15, 18....

(3) 4 km

(4) 2 km

term is 18 and 12th term is (2) 0,9, 18.... (4) 4.5, 9, 18...

25. If the sum of the digits of a three digit number is subtracted from that number then it will always be divisible by (1) 3 only (2) 2 only (3) Both 3 and 9

(4) All of 3,6 and 9

26. What is the sum of the first 13 terms of an arithmetic progression if the first term is –10 and last term is 26?

(1) 104

(2) 140

(3) 84

(4) 98

27. A game consists of tossing a coin 3 times. Hanif

wins if all the tosses give the same result. What is the probability that he loses the game?

(1) 1/2

(2) 3/4

(3) 1/3

(4) 2/3

28. Kamal has some apples. He sold 40% more than he ate. If he sold 70 apples, how many did he eat?

(1) 18

(2) 42

(3) 50

(4) 90

29. If 0.03  0.3  a  0.3  0.3  b , value of a is b

(1) 0.009

(2) 0.03

(3) 0.9

(4) 0.08

30. For an equilateral triangle, the ratio of the in-radius and the ex-radius is

(1) 1 : 2

(2) 1 : 2

(3) 1 : 3 (4) 1 : 3

31. The average marks obtained by a class of 60 students are 65. The average marks of half of the students is found to be 85. The average marks of the remaining students is

(1) 35

(2) 45

(3) 55

(4) 65

OSWAAL CUET (UG ) 10 Mock Test Papers GENERAL TEST

56

32. The table shows the production of different types of cars (in thousands). Years/ Cars

2013

2014

2015

2016

2017

A

35

40

48

50

36

B

39

45

54

60

72

C

52

25

32

54

45

D

50

42

45

46

47

E

36

46

42

48

55

The ratio of the total production of cars of type C and E taken together in 2013 to the total production of cars of type D in 2014 and 2016 and type E in 2017 taken together is: (1) 8 : 13

(2) 5 : 8

40. If 2 = 1.4142 , find the value of 1 1 2 2 2  2  2 2 2 (1) 1.4144 (2) 2.8284 (3) 28.284 (4) 2.4142

41. Six friends, P, Q, R, S, T and U, are sitting in a closed circle facing the centre such that each one is facing another one. P is facing S. R is between P and Q, U is between P and T who is sitting right of S. Who is sitting to the immediate right of R? (1) P (2) T (3) Q (4) S

42. Select the Venn diagram that best illustrates the relationship between the following classes: Villagers, Poor persons, Males

(3) 13 : 32 (4) 8 : 11



and Assam tea are 4 : 7 and 2 : 5. The ratio in which these two blends should be mixed to get the ratio of Darjeeling and Assam tea in the new mixture as 6 : 13 is

(1) 22 : 35

amounts to ` 800 in 3 years and to ` 840 in 4 years. The rate of interest per annum is:

(1) 2 1 % 2

(2) 4%

(3) 5%

2 (4) 6 % 3

35. If a = 64 and b = 289, then the value of



a + b  (1) 21/2

b a (2) 2



1 2

is : (4) –2

36. The inner and outer radii of a circular track are,

respectively, 29 metre and 23 metre. The cost of levelling the track at `7 per sq. metre is.

(1) `3,284

(2) `5,300 (3) `7,215 (4) `6,864

37. A, B and C can do a piece of work in 30, 20 and 10

days respectively. A is assisted by B on one day and by C on the next day, alternately. How long would the work take to finish?

3 (1) 9 days 8 (3) 8

8 (2) 4 days 8

4 days 13

(4) 3

9 days 13

38. Alipta got some amount of money from her father.

In how many years will the ratio of the money and the interest obtained from it be 10:3 at the rate of 6% simple interest per annum?

(1) 7 years

(2) 3 years (3) 5 years (4) 4 years

39. Three numbers are in the ratio 5 : 7 : 12. If the sum

of the first and the third numbers is greater than the second number by 50. The sum of the three numbers is

(1) 125

(2) 120

(3) 95

(4) 85



(4)



43. Two statements are given, followed by two

conclusions numbered I and II. Assuming the statements to be true, even if they seem to be at variance with commonly known facts, decide which of the conclusions logically follow(s) from statements. Statements:

(3) 4



(3)

(2) 26 : 35 (3) 35 : 78 (4) 13 : 22

34. A sum of money invested at compound interest

(2)

(1)

33. In two blends of mixed tea, the ratios of Darjeeling

I. All farmers are doctors.

II. All doctors are magicians. Conclusions: I. All farmers are magicians. II. No farmer is a magician. (1) Only conclusion II follow. (2) Both the conclusions follow (3) Either conclusion I or II follows (4) Only conclusion I follows. 44. A + B means ‘A is the wife of B’; A – B means ‘B is the daughter of A’; A × B means ‘B is the brother of A’; A ÷ B means ‘A is the father of B’; If P + R × T – Q + S ÷ U, then how is P related to the mother of U? (1) Aunt (2) Paternal grandmother (3) Mother (4) Maternal grandmother

45. In the following question, two statements are

given each followed by two conclusions I and II. You have to consider the statements to be true even if they seem to be at variance from commonly known facts. You have to decide which of the

MOCK TEST Paper-9

57

given conclusions, if any, follows from the given statements. Statements: (I)  Happiness is around everyone, we have to find it. (II) Happiness is a state of mind. If we can’t find it, we should create it. Conclusions: (I) Creation of happiness itself makes an individual satisfied and happy because creativity is constructive. (II) It’s all about analyzing the situational factors to understand happiness. (1) Only Conclusion II follows (2) Conclusion I and II both follow (3) Neither I nor II follow (4) Only Conclusion I follows

46. Ravi is walking in the East direction. After covering a distance of one kilometre, he turns 45° left and then 90° right. In which direction is he now? (1) North-East (2) West (3) North-West (4) North

51. A, B, C, D, E, F, G and H are sitting around a circular table facing towards centre. H is on the immediate left of A but is not the neighbor of D or E. F is on the immediate right of B. C is between E and F, G is the neighbor of E. Who is between A and B? (2) G

(1) D

(3) F

52. What is the mirror image of SMILE? (1) SM LEI

(2) SM I E L

(3) S

(4) SM I L E

M

L EI

53. Select the option that is related to the third term in the same way as the second term is related to the first term and sixth term is related to fifth term. 72 : 14: : 87 : ? :: 96 : 54 (1) 52

(2) 56

(3) 15

the question figure? Question Figure:

(4) MIL

48. Find out the two signs to be interchanged to make

Answer Figures:

the following equation correct. 25 + 5 × 7 − 12 ÷ 3 = 26 (2) + and × (1) + and ÷ (3) − and ÷ (4) + and −

(2)

(1)

49. CDF : GHJ :: KLN : ? (1) YED

(2) EDU

(3) OPS



(4) OPR

50. Choose the correct mirror image:

(3)

(4)



1.

55. Question figure

2. Answer figure

(1) 3.



(2)

4. (3)

(1) 1

(2) 2

(3) 3

(4) 4

(4) 29

54. Which answer figure will complete the pattern in

47. Select the correct option that will fill in the blank and complete the series. AEI, EIO, IOU, ........ (1) OAU (2) OUA (3) IOA

(4) H

(4)



OSWAAL CUET (UG ) 10 Mock Test Papers GENERAL TEST

58

56. Saksham introduced Nidhi to his friend, “She is the daughter of the only son of my father’s wife.” How is Saksham related to Nidhi? (1) Son (2) Cousin (3) Father (4) Brother

57. Directions: If you start running from a point towards North and after covering 4 km, you turn to yours left and run 5 km and then again turn to your left and run 5 km and then turn to left again and run another 6 km and before finishing you take another left turn and run 1 km, then answer the following question based on this information. Question: From the finishing point if you have to reach the point from where you started, in which direction will you have to run? (2) East (3) South (4) North (1) West

59. A + B means ‘A is the father of B’. A – B means ‘A is the sister of B’. A × B means ‘A is the brother of B’. A ÷ B means ‘A is the mother of B’. If, U + H × L – Q ÷ R – Y, Then how is L related to Y? (1) Mother’s brother (2) Mother’s sister (3) Maternal grandmother (4) Sister

60. Select the related word pair from the given alternatives. Afghanistan : Kabul :: ....... : ......... (1) Iran : Dublin

58. Select the number that will come next in the

(2) Nepal : Monaco

following series. 1, 5, 9, 17, 25, 37, 49, 65, 81, ......... (1) 99 (2) 110 (3) 100

(3) Pakistan : Islam Nagar (4) China : Beijing

(4) 101



Answer Key 1.

(1)

2.

(1)

3.

(3)

4.

(4)

5.

(4)

6.

(4)

7.

(4)

8.

(1)

9.

(2)

10.

(1)

11.

(4)

12.

(3)

13.

(4)

14.

(4)

15.

(1)

16.

(4)

17.

(1)

18.

(1)

19.

(2)

20.

(3)

21.

(1)

22.

(3)

23.

(2)

24.

(3)

25.

(3)

26.

(1)

27.

(2)

28.

(3)

29.

(3)

30.

(1)

31.

(2)

32.

(1)

33.

(1)

34.

(3)

35.

(1)

36.

(4)

37.

(1)

38.

(3)

39.

(2)

40.

(2)

41.

(3)

42.

(4)

43.

(4)

44.

(1)

45.

(3)

46.

(1)

47.

(2)

48.

(1)

49.

(4)

50.

(3)

51.

(1)

52.

(4)

53.

(2)

54.

(4)

55.

(1)

56.

(3)

57.

(1)

58.

(4)

59.

(2)

60.

(4)

MOCK TEST Paper-1

59

MOCK

Test Paper Maximum Marks : 250

10 Time : 60 Minutes

General Instructions : (i) This paper consists of 60 MCQs, attempt any 50 out of 60 (ii) Correct answer or the most appropriate answer: Five marks (+5) (iii) Any incorrect option marked will be given minus one mark (-1) (iv) Unanswered/Marked for Review will be given no mark (0) (v) If more than one option is found to be correct then Five marks (+5) will be awarded to only those who have marked any of the correct options (vi) If all options are found to be correct then Five marks (+5) will be awarded to all those who have attempted the question. (vii) Calculator / any electronic gadgets are not permitted.

1. According to which inscription Samudragupta

defeated 9 kings in the North, and 12 Kings in the South, and reduced all the Atavika states to vassalage? (1) Iron Pillar (2) Sun Pillar (3) Vijaya Stambha (4) Allahabad Pillar

2. Which of the following is correctly matched? 1. National Good Governance Day: 25 December 2. Armed Forces Flag Day: 7 December 3. Vijay Diwas: 16 December 4. Hindi Day: 14 September Choose the correct code. (1) 1 and 2 only (2) 2 and 4 only (3) 1, 2 and 4 only (4) 1, 2, 3 and 4

3. Which of the following statement is/are correct 1. Sucheta Kriplani was India’s first woman chief minister. 2. Sarojini Naidu was the first woman Governor of the state of Uttar Pradesh. Choose the correct code. (1) 1 only (2) 2 only (3) Both 1 and 2 (4) Neither 1 nor 2

4. The first woman to preside over the UN General Assembly: (1) Rajkumari Amrit Kaur (2) Vijaylakshmi Pandit (3) Kamla Nehru (4) Indira Gandhi

5. Which among the following is a light sensitive device used for converting images to their digital form? (1) Printer (2) Monitor (3) Scanner

(4) RAM

6. Router is used in which of the following layer? (1) Transport layer (3) Physical layer

(2) Network layer (4) Application layer

7. Match the following: 1. The Battle of Ghaghra A. 1529 2. Battle of Khatauli B. 1518 3. Battle of Gagron C. 1519 4. The third battle of Panipat D. 1761 Choose the correct code. (1) 1-A 2-B 3-C 4-D (2) 1-B 2-A 3-C 4-D (3) 1-D 2-C 3-A 4-B (4) 1-C 2-A 3-B 4-D 8. Which of the Mughal emperors is commemorated with a mosque bearing his name in Fatehabad? 1. Akbar 2. Babur 3. Humayun 4. Jahangir 9. Which of the following statement is correct? 1. The Rourkela Steel Plant was established in 1959 within the Sundargarh district of Odisha through a partnership with Germany. 2. The Bhilai Steel Plant, situated in the Durg District of Chhattisgarh, initiated production in 1959 with collaborative efforts from Russia. 3. The Durgapur Steel Plant in West Bengal commenced its operations in 1962 with the assistance of the United Kingdom’s government. 4. The Bokaro Steel Plant, founded in 1964 in Bokaro, was established through collaboration with Russia. Choose the correct code. (1) 1 and 2 only (2) 2,3 and 4 only (3) 1, 2 and 4 only (4) 1, 2, 3 and 4

OSWAAL CUET (UG ) 10 Mock Test Papers GENERAL TEST

60

10. Wind turbines convert _____________ energy into mechanical power. (1) Chemical (3) Gravitational

belonging to the state of: (2) Nuclear (4) Kinetic

(1) Kerala

(2) Hypothalamus (4) Pineal

12. Taiga is an example of which type of ecosystem?

21. Allowing 20% and 15% successive discounts, the selling price of an article becomes ` 3,060; then the marked price will be (1) ` 4,000

(2) ` 4,400 (3) ` 5,000 (4) ` 4,500

22. The compound interest on ` 30,000 at 7% per annum for a certain time is ` 4,347. The time is

(1) Grassland ecosystem (2) Forest ecosystem (3) Tundra ecosystem (4) Desert ecosystem

(1) 3 years

(2) 4 years

(3) 2 years

(4) 2.5 years

23. What is the value of (1 + 1/n) + (1 – 2/n) + (1 – 3/n)

13. Which of the following states share a boundary with Bhutan? 2. 1. West Bengal 4. 3. Sikkim Choose the correct code. (2) (1) 2, 3 and 4 only (3) 1, 2 and 4 only (4)

(2) Odisha

(3) Himachal Pradesh (4) Arunachal Pradesh

11. Which of the following glands is present between the lungs? (1) Pituitary (3) Thymus

20. ‘Mahuri’ is a traditional musical instrument

Arunachal Pradesh Assam 1 and 4 only 1, 2, 3 and 4

14. A worldwide financial messaging network which exchange messages between banks and financial institutions is known as_________

(1) CHAPS (2) SWIFT (3) NEFT

(4) SFMS

15. Who among the following is the first Indian Prime

Minister to receive the Grand Cross of the Order of Honor from Greece? (1) Indira Gandhi (2) Atal Bihari Vajpayee (3) Narendra Modi (4) Dr. Manmohan Singh

16. Match the following. List I (Unit)

List II (Physical Quantity)

1. Pascal

A. Pressure

2. Joule

B. Work

3. Fahrenheit

C. Temperature

4. Lightyear

D. Distance

Choose the correct code. (1) 1-A 2-B 3-C 4-D (2) 1-B 2-A 3-C 4-D (3) 1-D 2-C 3-A 4-B (4) 1-C 2-A 3-B 4-D

17. Among the listed bacteria, which one exhibits

penicillin resistance due to its absence of a cell wall? (1) Spirochetes (2) Mycoplasmas (3) Cyanobacteria (4) Bdellovibrios 18. Corundum is an ore of: (1) Copper (2) Silver (3) Iron (4) Aluminum 19. Where is the headquarters of the Inter­national Olympic Committee located? (1) Italy

(2) Switzerland

(3) Belgium

(4) France

+……... up to n terms? (1) 1/n

(2) n/2

(3) n(n – 1)/2

(4) (n – 1)/2

24. A man can row 6 km/h in still water. If the speed of the current is 2 km/h, it takes 3 hours more in upstream than in the downstream for the same distance. The distance is (2) 24 km (3) 20 km (4) 32 km (1) 30 km

25. If x2 – y2 = 80 and x – y = 8, then the average of x and y is (1) 2

(2) 3

(3) 4

(4) 5

26. The table shows the production of different types of cars (in thousand). Study the table carefully and answer the question: Years/ Cars A

2014

2015

2016

2017

2018

64

56

57

63

70

B

48

54

63

64

72

C

33

42

48

57

64

D

25

45

40

55

35

E

40

48

52

61

60

The ratio of the total production of type A cars in 2015 and type B cars in 2014 taken together to the total production of type C cars in 2017 and type E cars in 2018 taken together is: (3) 8 : 9 (4) 34 : 39 (1) 16 : 19 (2) 4 : 5

27. If A earns 25% more than B then how much percent does B earns less than A:

(1) 15%

(2) 20%

(3) 25%

(4) 30%

28. In ∆ABC, the line parallel to BC intersects AB and AC at P and Q respectively. If AB : AP = 5 : 3, then AQ : QC is: (1) 3 : 2

(2) 2 : 3

(3) 3 : 5

(4) 1 : 2

29. The altitude of an equilateral triangle of side 2 3 (1)

cm is: 4 m 3

(2)

4 3

cm (3)

4 M 3

(4) 1 cm

MOCK TEST Paper-10

61

30. The fifth term of the sequence for which

40. The ratio of boys and girls in a school is 27 : 23. If the difference between the number of boys and girls is 200, then find the number of boys. (1) 1350 (2) 1250 (3) 1300 (4) 1200

t1 = 1, t2 = 2 and tn+2 =tn + tn+1, is (1) 5

(2) 10

(3) 6

(4) 8

31. The average weight of 15 oarsmen in a boat is increased by 1.6 kg when one of the crew who weighs 42 kg is replaced by a new man. Find the weight of the new man (in kg).

(1) 67

(2) 65

(3) 66

(4) 43

32. A student was asked to divide a number by 6 and

add 12 to the quotient. He, however first added 12 to the number and then divided it by 6. Getting 112 as the answer. The correct answer should have been

(1) 124

(2) 122

(3) 118

41. Find out the two signs to be interchanged to make the following equation correct. 15 + 15 × 15 − 15 ÷ 15 = 15 (1) − and ÷ (2) + and − (4) + and × (3) × and ÷

42. Select the Venn diagram that best illustrates the relationship between the following classes: Indians, Females, Voters

(4) 114

(1)

33. By selling cloth at ` 9 per metre, a shopkeeper loses 10%. Find the rate at which it should be sold so as to earn profit of 15%. (1) ` 11.20



(3)

(1)

3+ 2

(2)

3− 2

(3)

2± 3

(4)

2− 3

43. Read the given statements and conclusions

35. A student takes 1.25 hours to travel from home to

school at a speed of 4 km/h. By what percentage should he increase his speed to reduce the time by 25% to cover the same distance from school to home? (3) 25% (4) 50% (1) 33 1 % (2) 45% 3

36. The radius of a wheel is 14 cm. What is the distance (in cm) travelled by the wheel in 15 revolutions? (2) 660

(3) 1320

(4) 1980

37. PQRS is square whose side is 20 cm. By joining

opposite vertices of PQRS are get four triangles. What is the sum of the perimeters of the four triangles?



 (4)  40

 2  80 

(2) 80 2  80

2 cm



(3) 40 2  40

(4)

3− 2

(1) 40



(2) ` 11.30 (3) ` 11.40 (4) ` 11.50

34. The square root of 3 + 2 is

(1) 440

(2)

carefully. Assuming that the information given in the statements is true, even if it appears to be at variance with commonly known facts, decide which of the given conclusions logically follows from the statements. Statements: I. Some bottles are glasses. II. Some glasses are bowls. Conclusions: I. Some bowls are bottles. II. Some glasses are bottles. III. No bowl is a bottle. (1) All Conclusions I, II and III follow. (2) Either Conclusion II or III follows. (3) Only Conclusion II and either Conclusion I or III follow. (4) Only Conclusions II and III follow.



44. Question figure

38. Ram and Shyam can complete a task in 6 2 days

3 and 15 days, respectively. They work together for 4 days, and then Ram leaves. In how many days after Ram leaves, will Shyam complete the remaining task alone?

1 (1) 1 days (2) 2 days (3) 3 days (4) 4 days 2 39. If a sum of money becomes ` 4000 in 2 years and ` 5500 in 4 years 6 months at the same rate of simple interest per annum, then the rate of simple interest is (1) 21 3 % (2) 21 2 % (3) 21 1 % (4) 21 5 % 7 7 7 7

Answer figure

(2)

(1)





(4)

(3)



OSWAAL CUET (UG ) 10 Mock Test Papers GENERAL TEST

62

45. A clock is started at noon. By 20 minutes part 6, the hand had has turned through: (1) 175°

(2) 180°

(3) 185°

50. WOLF : FLOW :: DRAW : ? (1) WRAD

(4) 190°

46. Which answer figure will complete the pattern in the question figure? Question Figure:

(2) WARD (3) RWAD (4) DARW

51. In a code language, ‘PLACARD’ is written as ‘TPEYEVH’. How will ‘MONSTER’ be written in that language? (1) RTSOXIV

(2) QSROXIV

(3) QSRRXIV

(4) PSSOXJV

52. Five friends P, Q, R, S and T are sitting around a circular table facing the center of the table. S is sitting to the immediate right of P. T is sitting to the immediate left of Q. P is between S and R. Who is sitting at the second place to the left of T? (1) Q

(2) R

(3) P

(4) S

53. Select the option that is related to the third number

Answer Figures:

in the same way as the second number is related to the first number. (2)

(1)

6 : 252 :: 5 : ?



(1) 225

(2) 150

(3) 175

(4) 125

54. Four villages A, B, C and D lie in a straight line. D is (3)

(4)



47. Introducing Kumar, Vikram said, “His mother is

10 km from B. A is exactly between D and C and C from B is 2 km more than it is from D. How far is C from B? (1) 4 km

(2) 6 km

(3) 8 km

(4) 2 km

the wife of my father’s son. Brother and sisters I have none.” How is Kumar related to Vikram?

55. Which number will follow next in the given

(1) Uncle

(2) Son

3, 18, 48, 93, ?

(3) Brother

(4) Father

48. If today is Monday, After 62 it will be: (1) Wednesday

(2) Saturday

(3) Sunday

(4) Thursday

49. In the following question, two statements are given each followed by two conclusions I and II. You have to consider the statements to be true even if they seem to be at variance from commonly known facts. You have to decide which of the given conclusions, if any, follows from the given statements. Statements:

number series? (1) 247

(2) 147

(3) 153

X is the neighbour of Y and P. Q is the neighbour of Z and R, P is to the right of X and P and Z are immediate neighbours. Which pair sits on the extreme ends? (1) Y, P

(2) Y, R

(3) X, P

and complete the series. J, N, R, V, ......... (1) D

(2) Z

(3) V

A – B means ‘A is the brother of B’;



(II)  Marketers identified the need and grabbed the opportunity of polluted atmosphere and created a new product for people.

A ÷ B means ‘A is the daughter of B’.

(I) The reason for invention of air purifier is rising levels of pollution.



(II) The need is created in market about the product and then it is established. (1) Only Conclusion II follows. (2) Conclusion I and II both follow. (3) Neither I nor II follow. (4) Only Conclusion I follows.

(4) G

58. A + B means ‘A is the mother of B’;

(I)  Air purifier is the latest innovation of this decade.



(4) P, Z

57. Select the correct option that will fill in the blank



Conclusions:

(4) 202

56. X, Y, Z, P, Q and R are sitting in a row facing north.

A × B means ‘A is the father of B’; If, P – K × Y – J ÷ S + R, then which of the following statements is NOT correct ? (1) J is daughter of P. (2) K is husband of S. (3) Y is son of S. (4) P is paternal uncle of R. 59. Select the related word pair from the given alternatives. Month : Year :: .......... : ..........

MOCK TEST Paper-10

63

(1) Litre : Liquid (2) Speed : Vehicle (3) Time : Distance (4) Minute : Hour 60. A is located to the West of B. C is located at North in between A and B. D is exactly to the South of B

and also in line with B. In which direction of C is D located? (1) South (2) South-East (3) West (4) South-West

 Answer Key 1.

(4)

2.

(4)

3.

(3)

4.

(2)

5.

(3)

6.

(2)

7.

(1)

8.

(3)

9.

(4)

10.

(4)

11.

(3)

12.

(2)

13.

(4)

14.

(2)

15.

(4)

16.

(1)

17.

(2)

18.

(4)

19.

(2)

20.

(2)

21.

(4)

22.

(3)

23.

(4)

24.

(2)

25.

(4)

26.

(3)

27.

(2)

28.

(1)

29.

(4)

30.

(4)

31.

(3)

32.

(2)

33.

(4)

34.

(1)

35.

(1)

36.

(3)

37.

(2)

38.

(2)

39.

(1)

40.

(1)

41.

(1)

42.

(1)

43.

(3)

44.

(4)

45.

(3)

46.

(4)

47.

(2)

48.

(1)

49.

(4)

50.

(2)

51.

(2)

52.

(3)

53.

(2)

54.

(2)

55.

(3)

56.

(2)

57.

(2)

58.

(1)

59.

(4)

60.

(2)

SOLUTIONS OF

Mock Test Paper 1. Option (3) is correct. Explanation: The practice and institution of Langar were originally initiated by Baba Farid, a Muslim belonging to the Chishti Sufi order. This tradition was already prevalent among Sufis in the 12th and 13th centuries within the Indian subcontinent. In Sikhism, Langar holds significant importance and was established when Guru Nanak Ji, the founder of Sikhism, provided food to hungry holy men. Guru Amardas Sahib Ji Introduced Anand Karaj (Marriage Ceremony). Mata Khivi, the wife of the second Sikh Guru Angad Dev, played a crucial role in the development of Langar. She actively participated in serving at Gur ka Langar, the Guru's free kitchen, alongside the first five Gurus. The third Guru, Amar Das, further refined the concept of Langar by introducing "pangat sangat," where individuals of all ranks sit and eat together as equals in the congregation. In present times, Langar remains an integral part of every gurdwara and Sikh worship service. Its provision, preparation, service, and cleanup are entirely voluntary, symbolizing the spirit of selfless service and equality within the Sikh community.

2. Option (1) is correct. Explanation: Fertile riverine alluvial soil is best suited for producing sugarcane, rice and plantain. Alluvial soil is rich in potassium. Red soil is ideal for crops like corn, red gram, Bengal gram, green gram, groundnut and castor seed.

3. Option (4) is correct. Explanation: Andaman and Nicobar is a group of 572 islands out of which 37 are inhabited at the junction of the bay of Bengal and the Andaman sea.

4. Option (3) is correct. Explanation: 17th Parallel: The 17th Parallel was the border between North Vietnam and South Vietnam during the Vietnam War. It was drawn up by the Geneva Accords of 1954, which ended the First Indochina War. The 17th Parallel was never a permanent border,

1

and it was eventually abandoned after the reunification of Vietnam in 1976. 38th Parallel: The 38th Parallel is the border between North Korea and South Korea. It was drawn up by the United States and the Soviet Union at the end of World War II. The 38th Parallel is also known as the Demilitarized Zone (DMZ), and it is one of the most heavily fortified borders in the world. 22nd Parallel: The 22nd Parallel is the border between Egypt and Sudan. It was drawn up in 1899 by the British and the French, who were the colonial powers in the region at the time. The 22nd Parallel is a natural boundary, as it follows the course of the Nile River. Durand Line: The Durand Line is the border between Pakistan and Afghanistan. It was drawn up in 1893 by Sir Mortimer Durand, the British Foreign Secretary. The Durand Line is a controversial border, as it is not recognized by Afghanistan.

5. Option (2) is correct. Explanation: Articles 245 to 255 in Part XI of the Constitution deal with the legislative relations between the Centre and the states. Constitution of India, being federal in structure, divides all powers (legislative, executive and financial) between the Centre and the states.

6. Option (4) is correct. Explanation: On 21st January, 1972, all the three states (Manipur, Meghalaya and Tripura) became full-fledged states under the NorthEastern Areas (Reorganisation) Act 1971.

7. Option (1) is correct. Explanation: Iris is a potency in our eye that regulates the measure of the pupil. It widens and lessen because of which extent of the pupil changes. Iris controls the size of the pupil.

8. Option (4) is correct. Explanation: Treaty of Madras (1769): This treaty was signed between the British East India Company and Hyder Ali of Mysore to end the First Mysore War. The treaty forced Hyder Ali to cede some territory to the British and to pay a large indemnity.

MOCK TEST Paper-1 reaty of Wadgaon (1779): This treaty T was signed between the British East India Company and the Maratha Empire to end the Second Phase of the First Anglo-Maratha War. The treaty forced the Marathas to cede some territory to the British and to pay a large indemnity. Treaty of Salbai (1782): This treaty was signed between the British East India Company and the Maratha Empire to end the First AngloMaratha War. The treaty restored most of the territory that the Marathas had ceded to the British in the Treaty of Wadgaon, but it also forced the Marathas to recognize the British as the dominant power in India. Treaty of Seringapatam (1792): This treaty was signed between the British East India Company and Tipu Sultan of Mysore to end the Third Anglo-Mysore War. The treaty forced Tipu Sultan to cede much of his territory to the British, including the city of Seringapatam, and to pay a large indemnity.

9. Option (3) is correct. Explanation: First Five-Year Plan (1951-1956): It is based on Harrod-Domar Model. It mainly focused on Agriculture. Community Development Program is also introduced in this Plan. Second Five-Year Plan (1956-1961): It is based on Mahalanobis Model. It mainly focused on Industry. Steel plants at Bhilai, Durgapur, and Rourkela were established with the help of Russia, Britain (the U.K.), and West Germany respectively. Fourth Five-Year Plan (1969-1974): It is based on Ashok Rudra Alon Model. The smiling Buddha project was also introduced in this Plan. 14 Commercial Banks were also Nationalised in this Plan. Fifth Five-Year Plan (1974-1979): It is based on D.P. Dhar Model. The slogan "Garibi Hatao" was also introduced in this plan. Regional Rural Bank was established In this Plan (2 October 1975).

10. Option (4) is correct. Explanation: UNICEF (United Nations Children's Fund): UNICEF is headquartered in New York City, United States. It was established in 1946 by the United Nations General Assembly to provide humanitarian and developmental assistance to children and mothers in developing countries. The World Bank: The World Bank is headquartered in Washington, D.C., United States. It was established in 1945 to provide loans to developing countries for economic development projects.

65 The World Trade Organization (WTO): The World Trade Organization is headquartered in Geneva, Switzerland. It was established in 1995 to promote free trade between member countries.

11. Option (4) is correct. Explanation: Sir Syed Ahmad Khan found the Muslim society to be educationally, socially and culturally backward. The movement came to be known as the Aligarh movement after sir Syed established his school at Aligarh which later became the center of the movement.

12. Option (2) is correct. Explanation: Linux is not a computer language, it is a Unix-like computer operating system assembled under the model of free and open-source software development and distribution. Among all general- purposeoperating systems, Linux has the largest installed base. Also, it is the leading operating system on servers and other big iron systems (such as mainframe computers).

13. Option (3) is correct. Explanation: Maria Ressa was not honoured with the Nobel Peace Prize in 2022. She was awarded the prize in 2021, along with Dmitry Muratov, for their "courageous fight for freedom of expression" in the Philippines and Russia, respectively. The other three individuals or organizations were honoured with the Nobel Peace Prize in 2022: Ales Bialiatski is a Belarusian human rights defender and the founder of the Belarusian Helsinki Human Rights Center. Russian human rights organisation Memorial is a Russian human rights organization that documents human rights abuses in Russia. Ukrainian human rights organisation Center for Civil Liberties is a Ukrainian human rights organization that promotes democracy and human rights in Ukraine. The Nobel Peace Prize 2022 was awarded to these individuals and organizations for their "efforts to safeguard freedom of speech in Belarus, Russia and Ukraine."

14. Option (2) is correct. Explanation: Surendranath Banerjee founded the Indian National Association with Ananda Mohan Bose on 26 July 1876. Surendranath Banerjee later joined the Indian National Congress, he supported the MontaguChelmsford reforms in contrast to congress. Later, he left congress and formed Indian National Liberation Federation in 1919.

OSWAAL CUET (UG ) 10 Mock Test Papers GENERAL TEST

66 15. Option (3) is correct.

Explanation: T he Indian Space Research Organisation (ISRO) has planned to keep the names of the Chandrayaan-2 lander and rover for their corresponding components in the Chandrayaan-3 mission as well. The Chandrayaan-3 lander will be named 'Vikram,' paying tribute to Vikram Sarabhai, the pioneer of the Indian space program. The rover will be called 'Pragyan.' According to available information, the propulsion module will transport the lander-rover configuration to a lunar orbit at a distance of 100 kilometres from the moon's surface. Upon a successful landing of the 'Vikram' module on the lunar surface, it will release the 'Pragyan' rover, which will then conduct an in-situ chemical analysis of the lunar terrain during its mobility.

16. Option (2) is correct. Explanation: Pedology is the study of soils in their natural environment. There are two main branches of soil science Pedology and edaphology. Edaphology studies the way soils influence plants, fungi and other living things while Pedology deals with soil morphology, pedogenesis and soil classification.

17. Option (1) is correct. Explanation: Terylene is not a raw fiber. It is an artificial polyester fiber assembled by petroleum-derived polymerizing ethylene glycol and terephthalic acid. Terylene is the trademark of the first polyester fabric ever created for synthetic material. It was first made by Chemist JR Whinfield in 1941.

18. Option (4) is correct. Explanation: An insulator is a significance that accomplishes not permitting electricity to hand via it. Glass is a fine insulator as it does not enable an outpour of electricity.

19. Option (3) is correct. Explanation: Indian Surf Team won the bronze medal at the Asian Surfing Championship, held at Thulisdhoo Island in Maldives. 18 countries from Asia took part in this championship. The Surfing Federation of India sent a team comprising 4 senior Surfers in the men’s category and 2 Surfers in the Under 18 category. 5 out of the 6 Surfers were from Tamilnadu. Srikanth from Kovalam, Tamil Nadu made it to the semi-finals in the Men's Category. The President of the Surfing Federation of India Arun Vasu said that the Surfing team finished overall 3rd with a bronze medal. Indian Surfers will now get ready for the firstever World Surfing League (WSL) event in India to be held in Mahabalipuram in Tamil Nadu from August 14 to 20 this year.

20. Option (1) is correct. Explanation: The Biraja Temple, the Rajarani Temple and the Samaleswari Temple are all located in Odisha. Biraja Temple or Biraja Kshetra is one of the most famous Hindu temples located in the Jajpur District of Odisha, India. Rajarani temple was built during the later stages of the Somavamshi rule in the 11th century A.D. The Samaleswari temple is located in Sambalpur, Odisha.

21. Option (2) is correct. Explanation: 789x531y is divisible by 72 So, 789x531y is divisible by 8 and 9. Given number is divisible by 8. 31y is divisible by 8. ∴y=2 Given number is divisible by 9 7 + 8 + 9 + x + 5 + 3 + 1 + 2 = 36 ⇒ x + 35 = 36 ⇒ x =36 – 35 = 1 ∴ 5x – 3y = 5 × 1 – 3 × 2 = 5 – 6 = –1

22. Option (3) is correct.

Explanation: Here, BC = Height of the unfinished tower AB = Height of the finished tower In DOAB, C AB tan 30° = x OA 1 h = B 3 78 h= In DOAC,

78 3

60°

...(i)

m

O

AC OA h+x 3= 78

tan 60° =

30° 78 m

h A

78 3 = h + x 78 3 =

78 3

+x

x = 78 3 − = 78 3 −

[From equation (i)] 78 3 78 3

×

3 3

78 3 = 78 3 − 3 1 2  = 78 3  1 −  = 78 3 ×  3 3 x = 52 3 m Hence, the tower will be raised 52 3 m.

MOCK TEST Paper-1 23. Option (2) is correct. Explanation: Average = A1 x1 + A2 + x 2 + ... + An xn x1 + x 2 + ... + xn Where, A1, A2, A3 … An are frequencies. ∴ Required mean =

1 ×1 + 2 ×2 + 3 ×3 + 4 ×4 + 5 ×5 + 6 ×6 + 7 ×7 1+ 2 + 3+ 4 + 5+ 6 + 7

1  4  9  16  25  36  49 140  5 = 28 28

24. Option (1) is correct. Explanation: S.P. of machine = 80% of ` 3840  3840  80  =`   = ` 3072  100 

25. Option (3) is correct. Explanation: Maximum length of each price = HCF of 1.5 m and 1.2 m = 0.3 m 12 15   1 12 3 12  4 12 x

∴ HCF of 1.5 and 1.2 metre = 0.3 metre. Hence, the greatest length of each pipe is 0.3m

67 29. Option (3) is correct. Explanation: Speed of boat in still water 1 (Downstream rate + Upstream rate) = 2 1 12 (8 + 4) kmph = kmph = 6 kmph = 2 2

30. Option (2) is correct. Explanation: Let time taken by B and C be x days So, time taken by A = 3x days Part of work done by A, B and C in 1 day 1 1 31 4 =    x 3x 3x 3x As per the question, 4 1 or 3x = 4 × 24 ∴ = 3x 24 4 × 24 = 32 days or x = 3 Thus, time taken by A = 3x = 3 × 32 = 96 days

31. Option (3) is correct Explanation: p, q, r are in G.P. q r = So, p q or or

q2 = pr q = pr

32. Option (1) is correct. Explanation:

A

26. Option (3) is correct Q

Explanation: 1  100 S.I .  100 100   12% p.a. Rate = 1 Principal  Time 1 12

27. Option (4) is correct. Explanation: As per question, The number 84 must not be a multiple of sum of the terms of ratio. For ratio 3 : 2. Sum of the terms of ratio = 3 + 2 = 5 which is not a factor of 84

28. Option (1) is correct. Explanation: As we know that, (m – n)2 = m2 + n2 – 2 mn 162 = 400 – 2 mn 2 mn = 400 – 256 2 mn = 144 mn = 72

B

R

C

P

According to the question, AQ = AR Q AB = AC So, BQ = RC Now, BQ = BP, CP = CR ∴ BP = PC

(Given)

33. Option (4) is correct. Explanation: Given that, ∆ABC is an isosceles triangle. AB = AC, XY || BC and ∠A = 30° A

30˚ X B

Y

C

OSWAAL CUET (UG ) 10 Mock Test Papers GENERAL TEST

68

180  30 2 150   75 2

Now, ∠ABC = ∠ACB =

Q ∴ ∴

38. Option (4) is correct. Explanation: Given that, Diagonals of a rhombus = 16 cm and 30 cm

XY || BC ∠AXY = ∠ABC = 75° ∠BXY = 180° – 75° = 105°

D

34. Option (4) is correct. Explanation: Time from 10 p.m., to 6 a.m. = 8 hours ∴ Required time (15 + 35)% of 8 hours

O

A

8  50  =   hours  100  = 4 hours

B

35. Option (1) is correct. Explanation: By using the alligation rule, we get Milk in Vessel A

4 litre 7

Milk in Vessel B

2 litre 5

1 2

1 2 − 2 5

4 1 − 7 2

8 −7 1 = 14 14 1 1 So, the required ratio = : 10 14 =

5 −4 1 = 10 10

=

= 14 : 10 = 7 : 5

36. Option (4) is correct. Explanation: Given: x = 3 − 2 2 

1 1 1 32 2    x 32 2 32 2 32 2



32 2  32 2 98 2

x+

1 x

= 3 −2 2 + 3 + 2 2 + 2 = 8 = 8 = 2 ×2 ×2 = 2 2

37. Option (3) is correct. Explanation: We know that, radius of the incircle (x) of an equilateral triangle side = 2 3 =

Now, AC OA ∴ BD ∴ OB ∠ AOB ∴ In ∆OAB

OA 2 + OB 2

 ( 8 )2  (15)2 =

64 + 225

=

289

= 17 cm ∴ Perimeter of rhombus = 4 × l = 4 × 17 = 68 cm

39. Option (1) is correct. Explanation: Distance covered by the train that started at 11 AM in half an hour Relative speed

1  55 km 2



= (140 – 110) km/hr



= 30 km/hr

Time Taken in meeting from 11:30 AM 55 11 = = hours. 30 6 Distance covered by the train that started at 11:30 a.m. 11   =  140 ×  km  6 =

2 3

2 3 = 1 cm.

= 16 cm = 8 cm = 30 cm = 15 cm = 90°

AB =

 110 

Now,  x  1   x  1  2   x x  ∴

C



770 km 3

2 ⇒ 256 km 3

MOCK TEST Paper-1

69 44. Option (2) is correct.

40. Option (4) is correct. Explanation: Rate = 10% per annum = 5% per half year Time = T years R     A  P1  2   100 

12

9



926.10  1   1 800  20 



9261  21   8000  20 

3

6

45. Option (4) is correct.

2T

41. Option (2) is correct.

Explanation: As,

17

We

+21

12

19 +7

40 +21

46. Option (1) is correct.

Explanation: According to the questions,

bought

45

Same as,

42. Option (3) is correct.

We

24 +7

Explanation: As Dentist comes under the category of Doctor, in the same way Chemistry comes under the category of Science.

chocolates

N

As per the question, at 12 a.m., the minute hand towards South. So, at 3 a.m. also the minute hand would at 12 and so it would be point at South.

2T

On comparing the powers on both sides, we get 1 2T = = 3 ⇒ T 3= years 1 years 2 2

like

W

E

2T

2T

 21   21        20   20 

I

S

2T

5   926.10  800  1   100  

3

Explanation:

9 

5   8 (i) 1

chocolates

like them

Explanation:

8  

5

As, 3(ii)

1  6 (iii)

2

12

15

3

11

B

L

O

C

K

+1

+1

+1

+1

+1

3

13

16

4

12

Similarly,

From equation (i), (ii) and (iii)

19

21

16

18

5

13

5

I = 9, bought = 3 and them = 6

S

U

P

R

E

M

E

∴ I bought them = 936

+1

+1

+1

+1

+1

+1

+1

20

22

17

19

6

14

6

43. Option (1) is correct. Explanation: Given, total students = 42 Swati’s rank from bottom = 19 So, Purshottam’s rank from bottom = 19 – 6 = 13 We know that, Total students = Position from top + Position from bottom – 1 ⇒ 42 = Position from top + 13 – 1 \ Position from top = 42 – 12 = 30 Hence, Purshottam’s rank from the top = 30th

47. Option (4) is correct. Explanation: Correct relation between the given classes Relatives Uncle



Rich

OSWAAL CUET (UG ) 10 Mock Test Papers GENERAL TEST

70

54. Option (3) is correct.

48. Option (4) is correct. Explanation: According to the statements, Diagonals

Decagons

Angles Polygons

Explanation:

Cubes X

Cones

I. () II. () III. () IV ()

herefore, Sujit is the brother-in-law of T Manideep.

Hence, conclusions I and III follow.

49. Option (2) is correct.

55. Option (4) is correct.

Explanation:

Explanation:

3 16 29 42 55

Given : 8 ÷ 2 + 4 = 10

+13 +13 +13 +13

8 + 4 ÷ 2 = 10

50. Option (4) is correct.

 +

Explanation: First arrange the position of A, B, C and D. Then, fix the position of the remaining friends.

E



÷

changed to

4

56. Option (3) is correct. Explanation:

D

F

2



changed to

E

50 m F

N

50 m

A

B

70 m

D

C

S H

51. Option (1) is correct.

53. Option (4) is correct. Explanation: Given, Mohini > Nita; Mohini ≥ Sarita = Malini and Mohini < Hema So, Hema > Mohini ≥ Sarita = Malini Hence, second tallest cannot be determined.

= 70 – 50 = 20 m

o, she is 20 m far at H from point D, in South S direction.

Explanation: Television signal to be telecast Similarly, Radio signal to be broadcast.

50 m G

∴ Required distance = DH = FG – ED

Explanation: According to the given statements, only conclusion II follows. Conclusion I does not follow because there is no information about the importance of retailers and distributors.

52. Option (1) is correct.

E

W

57. Option (2) is correct. Explanation:

Mother

Lady

Sister

Diwakar Son

Grandson The lady is Diwakar ‘s sister.

MOCK TEST Paper-1

71

58. Option (2) is correct.



59. Option (1) is correct. Explanation: 10 km

Explanation:

4

1

10 km 10 km

3

1

2

5

3 Hence, by joining all the parts we can see the correct figure.

E S

Home

ence, he is 10 km in North at school from his H home.

  

4

N W

5 2

School

60. Option (1) is correct. Explanation:

Y

C

L

+2

M

Q

Z

+2

+2

A

E

N

+2

O

S

+2

+1

B



SOLUTIONS OF

Mock Test Paper 1. Option (4) is correct. Explanation: Somnath Temple is a Hindu temple dedicated to Lord Shiva. It is located in Prabhas Patan, Gujarat, India. The temple is one of the twelve Jyotirlingas or holiest shrines of Shiva. The Somnath Temple was destroyed and rebuilt several times over the centuries, most recently in 1951. Kamakhya Temple is a Hindu temple dedicated to the goddess Shakti or divine energy. It is located in Guwahati, Assam, India. The temple is one of the five Shakti Peethas or the holiest shrines of Shakti. The Kamakhya Temple is known for its unique architecture, which features a lingam (phallic symbol) inside a cave. Ramnathaswamy Temple is a Hindu temple dedicated to Lord Rama. It is located in Rameswaram, Tamil Nadu, India. The temple is one of the five dhams, or holiest pilgrimage sites for Hindus. The Ramanathaswamy Temple is known for its imposing gopurams (gateway towers) and its 1212 pillars. Mahabodhi Temple is a Buddhist temple dedicated to the Buddha. It is located in Bodh Gaya, Bihar, India. The temple is the site where the Buddha attained enlightenment. The Mahabodhi Temple is one of the most important pilgrimage sites for Buddhists around the world.

2. Option (2) is correct. Explanation: The United Nations utilizes six official languages, namely Arabic, Chinese, English, French, Russian, and Spanish. Ensuring accurate interpretation and translation of these languages, both spoken and written is crucial to the organization's operations as it facilitates clear and concise communication on global issues. The General Assembly (Article 51 of the Rules of Procedure), the Economic and Social Council, and the Security Council all incorporate these languages during their meetings (Article 41 of its Rules of Procedure). Representatives from each country have the option to speak in any of the six official languages or any language with interpretation available in one of the six official languages. Official documents are also distributed in all six official languages,

2

and each of them holds equal authority. While English and French serve as the official working languages of the United Nations Secretariat.

3. Option (4) is correct. Explanation: The Battle of Chausa was fought in 1539 between the forces of Babur and the Delhi Sultanate. Babur emerged victorious, and this battle marked the beginning of the Mughal Empire in India. The Battle of Saraighat was fought in 1671 between the Mughal Empire and the Ahom Kingdom. The Ahoms were victorious, and this battle helped to prevent the Mughal Empire from expanding further into Assam. The Battle of Saragarhi was fought in 1897 between the British Indian Army and the Afghan tribesmen. The British Indian Army was victorious, but they suffered heavy losses. This battle is considered to be one of the most heroic battles in British Indian history. The First Battle of Panipat was fought in 1526 between the forces of Babur and the Delhi Sultanate. Babur emerged victorious, and this battle marked the beginning of the Mughal Empire in India.

4. Option (1) is correct. Explanation: Trade-Related Investment Measures (TRIMS) prohibit quantitative restrictions on imports by foreign investors, making statement 1 correct. However, the TRIMS agreement applies only to investment measures related to trade in goods, not services, as stated in Article 1 of the TRIMS agreement, making statement 2 incorrect. Regarding existing TRIMS that are inconsistent with the agreement, members are required to notify the WTO Council for Trade in Goods. TRIMS are designed to restrict the preferences given to domestic firms, allowing international firms to operate more easily in foreign markets. They do not directly regulate foreign investment; instead, foreign investment is governed by FEMA (Foreign Exchange Management Act) and DIPP (Department of Industrial Policy and Promotion) guidelines and regulations.

MOCK TEST Paper-2 5. Option (3) is correct. Explanation: The last Nawab of Awadh was Wajid Ali Shah. He was the eleventh and last King of Awadh and held his title for only 9 years from 1847 to 1856. His kingdom was annexed by the British and he was sent to exile in Kolkata. The Nawab is famously known to be a patron of the arts and revived Kathak dance to its present glory.

6. Option (4) is correct. Explanation: An individual with hypermetropia can see distant entities absolutely but cannot see nearby objects distinctly. Hypermetropia is even known as farsightedness. Such a person has to keep reading material much beyond 25 cm from the eye for satisfying reading. This deficiency can be rectified by utilizing a convex lens of reasonable power.

7. Option (4) is correct. Explanation: Mumbai Port, situated on the western coast of India, is the country's largest port in terms of size and shipping traffic. It benefits from a natural harbour with water depths of 10-12 meters, allowing smooth docking and passage for large cargo ships. Kochi, often referred to as the "Queen of the Arabian Sea," held significant importance as a spice trading centre on India's west coast since the 14th century. It maintained trade relations with Arab merchants even during the pre-Islamic era. The Kingdom of Cochin later became a protectorate of the Portuguese in 1503. Vishakhapatnam Harbour, located in Andhra Pradesh, serves as an example of a landlocked harbour. Chennai, situated on the eastern coast, holds the distinction of being the oldest port in the region.

8. Option (1) is correct. Explanation: A plotter is a computer hardware device much like a printer that is used for printing vector graphics. Therefore, it is an output device. MICR, OMR and Barcode Reader are examples of input devices (any hardware device that sends data to a computer allowing user to interact with & control the computers).

9. Option (2) is correct. Explanation: The largest invertebrate is Mollusc. With over 80,000 species Mollusc forms the second largest phylum of animal on Earth and range from tiny snails to giant squid

73 and octopuses. The number of fossil species is estimated between 60,000 and 1,00,000 additional species. It is the second largest phylum of invertebrate animals after the Arthropoda.

10. Option (1) is correct. Explanation: Unicellular organisms reproduce through asexual means of reproduction. Asexual mode of reproduction involves cell division in which a single parent cells divides into two or more daughter cells. Cell division usually occurs as part of a large cell cycle. Different asexual modes of reproduction includes binary fission, multiple fission, fragmentation, budding etc.

11. Option (2) is correct. Explanation: Jnanpith Award: The Jnanpith Award is one of the most prestigious literary awards in India. It is awarded for outstanding contributions to Indian literature. Instituted in 1961 by the Bharatiya Jnanpith organization, the award recognizes exceptional works in any of the 22 official languages of India. The recipient is presented with a cash prize, a citation, and a bronze replica of the goddess Saraswati, the Hindu goddess of knowledge and wisdom. Pritzker Architecture Prize: The Pritzker Architecture Prize is considered one of the highest honours in the field of architecture. Established in 1979 by the Pritzker family of Chicago, United States, the award aims to honour living architects who have made significant contributions to the world of architecture. The laureate receives a bronze medallion and a cash prize, and the prize ceremony takes place at various locations around the world. Grammy Award: The Grammy Award is bestowed by the Recording Academy in the United States. It is one of the most prestigious awards in the music industry, recognizing outstanding achievements in various musical genres. The Grammy Awards ceremony is an annual event where artists, producers, and other music professionals are honoured across a wide range of categories, including Best Album, Best Song, and Best New Artist. Abel Prize: The Abel Prize is an international award presented for exceptional achievements in the field of mathematics. Named after the Norwegian mathematician Niels Henrik Abel, the prize was established in 2001 by the Norwegian government. It is awarded annually by the Norwegian Academy of Science and Letters. The laureate receives a substantial cash award and a commemorative medal.

OSWAAL CUET (UG ) 10 Mock Test Papers GENERAL TEST

74 12. Option (2) is correct.

Explanation: In India the ‘Van Mahotsav’ Day is observed on 1st July. It is an annual treeplanting festival in India, celebrated in the first week of July. This movement was initiated in the year 1950 by India’s then Union Minister for Agriculture, Kulapati Dr. KM Munshi.

13. Option (2) is correct. Explanation: Kalpana Chawla was the first Indian American astronaut to fly into space in November 1997 with a crew of six aboard the Columbia space shuttle STS-87. Chawla was the first and second Indian woman to fly into space after astronaut Rakesh Sharma, who flew with the Soyuz T-11 in 1984. She died during her second space flight aboard the space shuttle Columbia in 2003.

14. Option (2) is correct. Explanation: Sarkaria commission was appointed by the central government in 1983 to examine issues related to Centre-state relations.

15. Option (2) is correct. Explanation: The Balwant Rai Mehta Committee, constituted on 16th January 1957 by the Government of India, had the primary objective of examining the functioning of the Community Development Programme and the National Extension Service. Chaired by Balwant Rai Mehta, the committee proposed various recommendations, including the establishment of a 3-tier Panchayati Raj system. According to their recommendations, the 3-tier system is comprised of the following levels: Gram Panchayat at the village level. Panchayat Samiti at the block level. Zila Parishad at the district level. These tiers were intended to bring about effective local governance and empower communities by decentralizing administrative responsibilities.

16. Option (3) is correct. Explanation: Suppose the soil is too acidic, and to modify the soil’s pH. To create it less acidic, both quick lime and slaked lime is counted to the soil, also learned as the liming of the soil. Lime is alkaline, and it balances the acidity of the soil and creates a neutral soil.

17. Option (4) is connect. Explanation: Diesel motors donate to environmental pollution generated by exhaust emissions. It is liable for several fitness troubles.

Four primary impurities are carbon monoxide (CO), hydrocarbons, particulate matter—PM, and nitrogen oxides (NOx).

18. Option (3) is correct. Explanation: Sports Terms associated with the sport Cricket Dead Ball, Bouncer, Wide ball, Bye, Carron Ball, Duck, No Ball. Volleyball Ace, Attacker, Joust, Pipe, Roof, Shank, Triple block. Basketball Dribble, Exceed, Free throw, Set shot, Chest pass, Lay-up, Slam dunk Dribble, Throw-in, Bicycle kick, Football Back heel, Dummy run, Penalty, Toe Poke, Wingers.

19. Option (3) is correct. Explanation: The vaccination for “small pox” was invented by Edward Jenner in 1796. Before the introduction of a vaccine, the mortality of the severe form of smallpox-variola major was very high. Edward Jenner acted upon his observation that milkmaids who caught the cowpox virus did not catch smallpox.

20. Option (2) is correct. Explanation: The city of Lucknow is famous for the Chikankari Embroidery style. It is very intricate and delicate thread work done on the muslin cloth originally. Nowadays the work is being used to decorate a wide variety of fabric.

21. Option (3) is correct. Explanation: S.I. =

Principal × Time × Rate 100

18 12  = ` 4000   12 100   = `720

22. Option (3) is correct. Explanation: We know that: (3)1 =3, (3)2 =9, (3)3 =27, (3)4 = 81 Last digit of 34 = 1 This implies 321 = (34)5 × 31 Last digit of 321 = 1 × 3 =3 So, units digit in the expansion of (3)21 = 3 Thus Remainder when 321 is dividing by 5 = 3

23. Option (1) is correct. Explanation: Single equivalent discount for 20% and 10% 20  10     20  10    28% 100  

MOCK TEST Paper-2

75

Single equivalent discount for 28% and 10% 28  10     28  10    35.2% 100   So, S.P. of piano =

15000  100  35.2  100

= ` 9,720

29. Option (1) is correct. Explanation: Multiple of 3 in between 1 to 20 = 3, 6, 9, 12, 15, 18 So, probability that the ticket drawn bears 6 multiple of 3 = 20

24. Option (3) is correct. Explanation: Suppose the numbers be 3x, 4x and 5x. ∴ 5x + 3x = 4x + 52. 52 = 13 or x = 13 or 4x = 52 or x = 4 So, the smallest number = 3x = 3 × 13 = 39

25. Option (3) is correct. Explanation: Given, distance are same  2xy  Average speed =   km/h xy 2  30  60  =   km/h  30  60  2  30  60 3600 =  90 90 = 40 km/h

26. Option (3) is correct. Explanation: No. of books in each stack of equal height HCF of 84, 90 and 120. Now, 84 = 2 × 2 × 3 × 7 90 = 2 × 3 × 3 × 5 120 = 2 × 2 × 2 × 3 × 5 ∴ HCF of 84, 90 and 120 = 2 × 3 = 6

27. Option (3) is correct. Explanation: Given: 2   2 p  2   7  13  17  4   p  1 7

⇒ 3p + 44 = 56 ⇒ 3p = 12 ⇒p=4 So, ascending order of numbers, 2, 3, 4, 7, 10, 13, 17 Hence, Median = 7 (Middle value of series)

28. Option (3) is correct. Explanation: A finish the work in 40 days.

=

3 10

30. Option (1) is correct. Explanation: 1 + 0.6 + 0.06 + 0.006 + 0.0006 + ... = 1.666 ... = 1.6 6 2 = 1 =1 9 3

31. Option (3) is correct. Explanation: Given that, E C

A

96°

D

AD = DC = BC, and ∠BCE = 96°, Now Suppose ∠ACD = a = ∠DAC ∴ ∠CDB = 2a = ∠CBD We know that, the angles of the base of an isosceles triangles are equal. ∴ ∠ACB = 180° – 96 = 84° or ∠ACD + ∠DCB = 84° or a + 180° – 4a = 84° or 180° – 3a = 84° or 3a = 180° – 84° = 96° or a =

96° = 32° 3

or ∠DBC = 2a = 2 × 32° = 64°

32. Option (3) is correct. Explanation: Given: (10.15)2 = 103.0225 ⇒ (1.015)2 = 1.030225

B finish the work in 60 days.

⇒ (101.5)2 = 10302.25

then time taken to do work together

∴ 1.030225 + 10302.25

xy = days xy

=

=

40  60 2400   24 days 40  60 100

B

(1.015)2 + (101.5)2

= 1.015 + 101.5 = 102.515

OSWAAL CUET (UG ) 10 Mock Test Papers GENERAL TEST

76 33. Option (2) is correct. Explanation: Weight calculated below: Weight Apeksha’s – = Height

to height ratio is 59 = 32.7 1.8

Apsara’s – 51 = 29.65 1.72

42 − 38 19 × 7

=

76 − 66 11 × 19

=

4 19 × 7

=

10 11 × 19

So, the required ratio

Aradhana’s – 62 = 39.49 1.57 Archana’s – 74 = 44.05 1.68 So, Apsara has the least weight to height ratio.

34. Option (2) is correct.

=

11 1100 ×100 cm = cm 5 5 = 220 cm 2πr = 220

=

= 44 : 70 = 22 : 35

37. Option (1) is correct. Explanation: More distance covered in one hour without stoppage = 72 – 60 = 12 km

38. Option (3) is correct. Explanation: R   A  P1    100 

where a = side of equilateral triangle a Circum-radius = 3 a a Now, required ratio = : 2 3 3 or required ratio = 1 : 2

36. Option (1) is correct. Explanation: By using the rule of alligation, we get Mixture I

Mixture II

Darjeeling tea

Darjeeling tea

6 2 − 19 7

6 19

2 7

3

 43   64000    40 

3

3



64000  43  43  43 64  43  43  43  40  40  40 444



5088448 64

= ` 79507 ∴ C.I. = ` (79507 – 64000) = ` 15,507

39. Option (2) is correct. Explanation: Given: P = 7 + 4 3 and PQ = 1 

1 1 74 3   P 7  4 3 (7  4 3 )(7  4 3 )

On rationalising the denominator, we get =

7 −4 3 = 7 −4 3 49 − 48

Now, Q = 

4 6 − 11 19

T

7.5  75     64000  1    64000  1  1000  100    

35. Option (1) is correct. Explanation: We know that, a In-radius = 2 3

12 × 60 72

1   60 = 10 minutes 6

or 2 × 22 × r = 220 7 220 × 7 or r= 2 × 22 1540 r= 44 = 35 cm

4 11

4 10 4 10 : = : 19 × 7 11 × 19 7 11

So required Time =

Explanation: Distance covered by wheel in one Revolution = Circumference of wheel 11000 11 metre = = 5000 5

Now,

=

1 = 7 −4 3 P

1 1 74 3   Q 7  4 3 (7  4 3 )(7  4 3 ) 74 3

MOCK TEST Paper-2 

1

P

2



1 Q

2

77

 ( 7  4 3 )2  ( 7  4 3 )2

43. Option (2) is correct. Explanation: For consonants = Position value of alphabet

 2[(7 )2  ( 4 3 )2 ]

For vowels =

= 2(49 + 48) = 2 × 97 = 194

A Reverse   (27 – 1 = 26) × 2 = 52 Order

40. Option (2) is correct.

Reverse  (27 – 5 = 22) × 2 = 44 E  Order

Explanation: Given: 20% of rice is lost in transportation. 4 64 So, the remaining rice  16   quintals 5 5 To gain 25%, 5632  125  Required S.P. = `   = ` 7040 100   So, the required rate = `  7040  per quintal  64     5 

Reverse  (27 – 9 = 18) × 2 = 36 I  Order Reverse  (27 – 15 = 12) × 2 = 24 O  Order Reverse  (27 – 21 = 6) × 2 = 12 U  Order

As, A  R  O  U  N  D

52  18  24  12  14  4

and F  I  X

= `  7040  5  per quintal  64   

6  36  24



= ` 550 per quintal

Similarly

41. Option (4) is correct.

P  L  A  S  T  I  C

16  12  52  19  20  36  3

Explanation: E

20 m

F final point

44. Option (2) is correct. B

50 m A

Explanation:

Home

T

N

M

G

Son Mother

120 m

90 m

R

Sister

Son K

It is clear from the graph, T is not the paternal grandfather of M. C

30 m

D

o, required distance AF = 120 – 90 = 30 m S Thus, she is 30 m in North from her house.

45. Option (1) is correct. Explanation: As,

42. Option (4) is correct. Explanation: Since we have 365 days in a year, so from going to 18th Feb 1998, we will come across 52 weeks and one day, therefore 18th Feb 1998 will be Wednesday. Similarly, 18th February 1999 will be a Thursday, because there is no leap year involved between the year 1997 and year 1999.

109

114 +5



139 +25

Same as, 313



318 +5

343 +25

OSWAAL CUET (UG ) 10 Mock Test Papers GENERAL TEST

78 46. Option (4) is correct.

50. Option (4) is correct.

Explanation:

Explanation: Fix Raju’s position in the extreme left and arrange according to the given conditions.

Reverse  21 9 22 13 24 19 F R E N C H  Order

Now, 21 + 9 + 22 + 13 + 24 + 19 = 108

R

⇒ 108 + 6 (total alphabets in the word) = 114

S

K

P

D



51. Option (4) is correct.

and

Explanation:

Reverse

  15 12 8 8 L O S S   Order

32 * 8 * 14 * 7 *16

Now, 15 + 12 + 8 + 8 = 43 ⇒ 43 + 4 = 47

– ÷ = ∴ 32 – × 14 ÷ 7 = 16 ⇒ 32 – × 2 = 16 ⇒ 32 – 16 = 16

Similarly, Reverse  24 12 6 9 8 22 C O U R S E  Order

Now, 24 + 12 + 6 + 9 + 8 + 22 = 81 ⇒

81 + 6 = 87

ote: Reverse positional value = 27 – Original N positional value.

52. Option (1) is correct. Explanation: 3 km

47. Option (1) is correct.

B P

Explanation: Correct relation between the given classes

18 km

final point P

Q F 6 km E Q 10 km

s

Reptiles ou

A Starting point

Po is

on

Snakes

Explanation: According to the statements, Stalls X

offices

Bank

Explanation: ‘Hum’ is the sound made by bees. Similarly, ‘Hiss’ is the sound made by snakes.

Hence, only conclusion III follows.

54. Option (4) is correct.

49. Option (3) is correct.

Explanation: Since 2 and 4 is common in both dice, therefore 3 is opposite to 1.

Explanation:

R

+1

–2

P

+1

R

–2

N

+1

S

–2

L

+1

T

S

D

53. Option (1) is correct.

I. () II. () III. () IV ()

Q

Q 9 km

E

W

\ Required distance = PQ = BA – ED = 18 – 10 = 8 km Hence, Q is now at F, 8 km in South from position of P.

48. Option (3) is correct.

P

N

final point

–2

J

L +1

M



55. Option (3) is correct. Explanation:

brother D B Daughter Son

+1

N +1

O +1

P

P

sister

L

N

T brother

rom the figure, it is clear that N is the brother F of T.

MOCK TEST Paper-2

79

56. Option (3) is correct.



Explanation: Both conclusions are incorrect

59. Option (2) is correct. Explanation: So,

as no information about activeness in given and also number of schools are not decreasing.

57. Option (3) is correct. Explanation:



Grandmother Only child

will complete the figure.

60. Option (2) is correct. daughter

Chetan

Explanation: E

Girl

50 km

brother

D 110 km

Chetan is the brother of that girl. B

58. Option (3) is correct. Explanation:

E

W S

50 km

C

40 km

4 3

N

3 1

1 4

A Starting point

2

2 Hence, by joining all the parts we can see the correct figure.

From figure,

EB = DC = 110 km

∴ Required distance AE = AB + BE = 40 + 110 = 150 km ence, bird is 150 km far at North from starting H point.



SOLUTIONS OF Question Paper 1. Option (4) is correct. Explanation: Ribosomes are small organelles that are found in the cytoplasm of cells. They are responsible for making proteins. Proteins are essential for all living things and they are involved in a wide range of functions, such as: • Building and repairing tissues • Producing enzymes • Transporting substances in and out of cells • Fighting infection Golgi apparatus is a large organelle that is found in the cytoplasm of cells. It is responsible for packaging and transporting proteins and other substances around the cell. The Golgi apparatus also plays a role in the production of lysosomes, which are organelles that break down waste products. Mitochondria are small organelles that are found in the cytoplasm of cells. They are responsible for producing energy for the cell. Mitochondria are often called the "powerhouses of the cell" because they produce most of the cell's energy. So, the statements that Ribosomes make protein, the Golgi apparatus is involved in transporting substances in and out of the cell, and Mitochondria produce energy for the cell are all correct.

2. Option (2) is correct. Explanation: Plastids are found in plant cells, not animal cells. They are responsible for photosynthesis, which is the process by which plants make their own food. There are three main types of plastids: chloroplasts, chromoplasts, and leucoplasts. Chloroplasts are the most common type of plastid and they are responsible for photosynthesis. Chromoplasts give plants their colourful pigments, such as the red colour of tomatoes and the orange colour of carrots. Leucoplasts are colourless plastids that store starch and other nutrients. Lysosomes are known as the "suicide bags" of the cell because they contain enzymes that can break down cellular components. Lysosomes are found in both plant and animal cells. They play a role in cell defence, autophagy, and apoptosis.

3

The nucleus is a double-membraned eukaryotic cell organelle that contains genetic material. The nucleus is found in both plant and animal cells. It is the control centre of the cell and it contains the DNA, which is the genetic material of the cell. The DNA stores the instructions for making proteins and other molecules that the cell needs to function. The cell membrane is the outermost covering of all cells. It is a selectively permeable membrane that allows certain molecules to enter and exit the cell. The cell membrane is found in both plant and animal cells. It is responsible for maintaining the cell's internal environment and protecting the cell from its surroundings.

3. Option (3) is correct. Explanation: National Bank for Agriculture and Rural Development (NABARD) is a development bank set up by the Government of India in 1982. It is headquartered in Mumbai, Maharashtra. NABARD provides financial assistance to banks, financial institutions, and cooperatives engaged in agricultural and rural development activities. National Housing Bank (NHB) is a government-owned financial institution that was established in 1988. It is headquartered in Delhi. NHB provides financial assistance to individuals and institutions for housing purposes. Insurance Regulatory and Development Authority (IRDAI) is a statutory body that was established in 2000. It is headquartered in Hyderabad, Telangana. IRDAI regulates and supervises the insurance industry in India. Small Industries Development Bank of India (SIDBI) is a financial institution that was established in 1990. It is headquartered in Lucknow, Uttar Pradesh. SIDBI provides financial assistance to small and medium enterprises (SMEs).

4. Option (3) is correct. Explanation: Wodeyars ruled Mysore under a subsidiary alliance with the British after the defeat of Tipu Sultan at the Battle of

MOCK TEST Paper-3 Seringpatnam. The Wodeyar dynasty ruled Mysore from 1399–1947. It is believed to be the only royal family that ruled for more than 500 years over the Kingdom. The Battle of Seringapatan took place between Apr–May 1799. Tipu Sultan was killed in the War.

5. Option (4) is correct. Explanation: Kuroshio Current is a warm current of the Pacific Ocean. It originates in the East China Sea and flows northeast along the coast of Japan. The Kuroshio Current is one of the strongest currents in the world and it plays a major role in the climate of East Asia. Humboldt Current is a cold current of the Pacific Ocean. It originates in the Peru-Chile Trench and flows northward along the coast of Chile and Peru. The Humboldt Current is responsible for the cold climate of the west coast of South America. Labrador Current is a cold current of the Atlantic Ocean. It originates in the Labrador Sea and flows southward along the coast of Newfoundland and Labrador. The Labrador Current is responsible for the cold climate of the east coast of Canada. Canaries Current is a cold current of the Atlantic Ocean. It originates in the Canary Islands and flows southward along the coast of Africa. The Canaries Current is responsible for the cool climate of the west coast of Africa.

6. Option (3) is correct. Explanation: Siraj-ud-Daulah became the Nawab of Bengal after the death of Alivardi Khan in 1756. He was the last self-governing Nawab of Bengal. Murshid Ali Khan was the first Nawab of Bengal. Mir Jafar served as the first Nawab of Bengal under British Influence.

7. Option (3) is correct. Explanation: Easternmost: Kibithu in Arunachal Pradesh is the easternmost point of India. It is located in the Lohit district of Arunachal Pradesh. Kibithu is a small village that is located on the border with China. Westernmost: Ghuar Mota in the Kutch region of Gujarat is the westernmost point of India. It is located in the Bhuj district of Gujarat. Ghuar Mota is a small village that is located on the border with Pakistan. Northernmost: Indira Col is the northernmost point of India. It is located in the Ladakh region of Jammu and Kashmir. Indira Col is a mountain pass that is located on the border with China. Southernmost (Mainland): Kanyakumari is the southernmost point of India's mainland. It

81 is located in the Kanyakumari district of Tamil Nadu. Kanyakumari is a town that is located on a small island at the tip of India. The statement that the Easternmost: Kibithu in Arunachal Pradesh, Westernmost: is Ghuar Mota in the Kutch region of Gujarat, Northernmost: Indira Col, and Southernmost (Mainland): Kanyakumari are all correct.

8. Option (3) is correct. Explanation: In Microsoft Excel, the MIN function networks the smallest value among the values passed an arguments. For example, the fastest time in a race, the smallest sales number, or the lowest temperature. Arguments can be provided, for example= MIN (15, 60) = MIN (B1, B2, B3) = MIN (A1: A10)

9. Option (2) is correct. Explanation: A virtual image is formed by a convex mirror. The image constructed is usually reduced because of the mirror curvature. A convex mirror is comprehended as a diverging mirror. It is a curved mirror with a reflective surface that bulges towards the light origin.

10. Option (4) is correct. Explanation: The Battle of Chamkaur was fought between the forces of the Mughal Empire under Aurangzeb and the forces of the Sikh Guru Gobind Singh in 1704. The Sikhs were defeated and Guru Gobind Singh was killed. The Battle of Swally was fought between the Portuguese and the British East India Company in 1612. The British were victorious and this victory helped them to establish a foothold in India. The Battle of Plassey was fought between the forces of the Mughal Empire under Nawab Siraj ud-Daulah and the forces of the British East India Company under Robert Clive in 1757. The British were victorious and this victory gave them control of Bengal. The Battle of Buxar was fought between the forces of the Mughal Empire under Shah Alam II, the Nawab of Awadh Shuja ud-Daula, and the Nawab of Bengal Mir Qasim on one side and the forces of the British East India Company under Hector Munro on the other side in 1764. The British were victorious and this victory gave them control of most of India.

OSWAAL CUET (UG ) 10 Mock Test Papers GENERAL TEST

82 11. Option (4) is correct.

Explanation: Carbon dioxide is the gas that donates most to the sensations of global warming. Carbon dioxide is developed as a byproduct of cellular respiration and burning of fossil fuels. It is also generated by human industries, which increases global warming.

12. Option (2) is correct. Explanation: The electron was found by J. J. Thomson. Thomson’s investigations with cathode-ray pipes revealed that all atoms possess small negatively charged subatomic particles on electrons.

13. Option (4) is correct. Explanation: Dr Thani Al Zeyoudi hails from United Arab Emirates (UAE). He is the Minister of State for Foreign Trade of the UAE and the Minister of State for Artificial Intelligence. He was chosen as the chair of the 13th Ministerial Conference of the World Trade Organization (WTO) in 2023. Dr Al Zeyoudi is a graduate of the University of Cambridge and the Massachusetts Institute of Technology. He holds a PhD in Computer Science from MIT. He is a leading expert in artificial intelligence and has published extensively on the topic. He is also a member of the World Economic Forum's Global Future Council on Artificial Intelligence. Dr Al Zeyoudi is a strong advocate for free trade and open markets. He believes that trade can help to boost economic growth and development. He is also a strong supporter of the WTO and believes that the organization can play a vital role in promoting free trade and global economic cooperation. Dr Al Zeyoudi's appointment as the chair of the 13th Ministerial Conference of the WTO is a significant achievement for the UAE. It is a recognition of the UAE's growing role in the global economy and its commitment to free trade.

14. Option (1) is correct. Explanation: Sir C.V. Raman was an Indian physicist who won the 1930 Nobel Prize in physics for his pioneering work in the field of light scattering. He found that some refracted light changes its wavelength when light passes through a transparent material.

15. Option (2) is correct. Explanation: J.J Thomson was an English physicist. He received the Nobel Prize in physics for the discovery and identification of electron. He won Nobel Prize in 1906. Electrons are the first subatomic particle to be discovered.

16. Option (4) is correct. Explanation: Rehuke Khim or Cowrie shawl is the most important cloth meant for the rich man of the Yimchunger tribe in Nagaland. In Naga society, cowrie is a symbol of marital status.

17. Option (4) is correct. Explanation: T he recipient of the Lokmanya Tilak Award 2023 is Prime Minister Narendra Modi. The Lokmanya Tilak Award is a prestigious award given by the Tilak Smarak Mandir Trust to individuals who have made significant contributions to the fields of politics, social work, and public service. The award is named after Lokmanya Tilak, a prominent leader of the Indian independence movement. Prime Minister Modi was awarded the Lokmanya Tilak Award for his outstanding contribution to the development of India. He was recognized for his leadership in the areas of economic growth, social welfare, and national security. The award was presented to Prime Minister Modi by the Governor of Maharashtra, Bhagat Singh Koshyari, at a ceremony in Mumbai on August 1, 2023. In his acceptance speech, Prime Minister Modi paid tribute to Lokmanya Tilak and said that he was inspired by his vision of a strong and prosperous India.

18. Option (3) is correct. Explanation: The Malimath Committee on Reforms of Criminal Justice System was constituted by the Ministry of Home Affairs under the Chairmanship of Justice V.S. Malimath, former Chief Justice of the Karnataka and Kerala High Courts. It has been constituted to recommend the admissibility of confessions made before a police officer as evidence in a court of law.

19. Option (4) is correct. Explanation: A ll of these articles are fundamental rights guaranteed to all citizens of India by the Constitution. Article 15 prohibits discrimination on the grounds of religion, race, caste, sex, or place of birth. This means that no one can be discriminated against on the basis of their religion, race, caste, sex, or place of birth. This article also prohibits the state from discriminating against any citizen on any of these grounds. Article 19 guarantees certain rights regarding freedom of speech, assembly, association,

MOCK TEST Paper-3

83

movement, residence, and profession. This means that all citizens have the right to freedom of speech, to assemble peacefully, to form associations, to move freely throughout the country, to reside in any part of the country, and to practice any profession they choose. Article 23 prohibits traffic in human beings and forced labour. This means that no one can be forced to work against their will. This article also prohibits the employment of children below the age of 14 years in any factory or mine. Article 25 guarantees freedom of conscience and free profession, practice, and propagation of religion. This means that all citizens have the right to freedom of conscience and the right to profess, practice, and propagate their religion. This article also prohibits the state from interfering with the free exercise of religion.

20. Option (4) is correct. Explanation: Bull’s eye is used in the game of shooting sport. In this game participants uses handguns at paper targets at fixed distances and time limits. Focus of this game on accuracy and precision. This game is mostly played in USA and Canada.

21. Option (2) is correct.

Explanation: Any number is divisible by 88 if it is divisible by 8 and 11. So, the given number is divisible by 8 if 8y2 is divisible by 8. When y = 3 then 832 ÷ 8 = 104 For divisibility by 11, Sum of digits at odd places – sum of digit at even places = 0 ⟹ (1 + 3 + x + 5 + 3) – (3 + 0 + 5 + 8 + 2) =0 ⟹ x + 12 – 18 = 0 ⟹x=6 ∴x+y=6+3=9

24. Option (3) is correct.

Explanation: Given numbers = 12, 15, 7, 8 and x +13 Where x = 2 So, last number = 15 Now mean = (sum of all numbers) / 5 = 57 / 5 = 11.4

25. Option (2) is correct. Explanation: Given: 20% of rice is lost in transportation. 4 64 So, the remaining rice  16   quintals 5 5 To gain 25%, 5632  125  Required S.P. = `   = ` 7040 100  

Explanation: We know that, LCM of fractions = LCM of numerators HCF of denominators ∴ LCM of

23. Option (2) is correct.

2 4 5 LCM of 2, 4, 5 , , = 3 9 6 HCF of 3, 9, 6

20 = 3

 7040  per quintal So, the required rate = `  64     5  = `  7040  5  per quintal  64    = ` 550 per quintal

22. Option (4) is correct.

26. Option (4) is correct.

Explanation: Downstream rate of boat (x) =

35 = 7 km/h 5

35 = 5 km/h Upstream rate of boat (y) = 7 1 speed of boat in still water = ( x + y ) 2 1 = (7 + 5) 2 1 =  12  6 km/h 2

Explanation: Given, a, 1, b are in A.P. So, 1 

ab 2

or a + b = 2

...(i)

and 1, a, b are in G.P. a2 = b ...(ii) Put this value of b in eq. (i) a + a2 = 2 or

a2

+a–2=0

OSWAAL CUET (UG ) 10 Mock Test Papers GENERAL TEST

84 or a2 + 2a – a – 2 = 0 or a(a + 2) – 1(a + 2) = 0 or (a – 1) (a + 2) = 0 From Eq. (ii), we get if a = –2 then b = 4 if a = 1 then b = 1 But given a ≠ b then b = 4.

So,

AB sin 60° = AC



3 120 = 2 AC



AC = (120 × 2 ) / 3 = 80 3 m

[sin 60° =

3 ] 2

∴ The length (in m) of the string is 80 3 m

29. Option (4) is correct.

27. Option (2) is correct. Explanation: Given that, DABC is an isosceles right angled triangle. A

E D

B

C

∠C = 90°

Explanation: Number of boys = 40 + 35 + 15 + 35 + 40 + 20 = 185 Number of girls = 10 + 25 + 10 + 35 + 30 + 20 = 130 ∴ Required ratio = 185:130 = 37:26

30. Option (1) is correct. Explanation: Given: x = 5 + 2 6

Now, AC2 + CB2 = AB2 or 2 BC2 = (AD + DB)2 or 2 BC2 = AD2 + DB2 + 2 AD DB  ...(i) Now, ∆CEB and ∆CED are right angles. then, CD2 = CE2 + ED2 and BC2 = CE2 + BE2 BC2 – CD2 = (CE2 + BE2) – (CE2 + ED2) = CE2 + BE2 – CE2 – ED2 ⇒ BC2 – CD2 = BE2 – ED2 = (BE + DE) (BE – DE) = (AE + DE) (BE – DE) = AD · BD ...(ii) From eq (i) and (ii) AD2 + DB2 = 2CD2



1 1 52 6   x 5  2 6 ( 5  2 6 )( 5  2 6 )

On rationalising the denominator, we get =

5 −2 6 = 5 −2 6 25 − 24 2

Now,  x  1   x  1  2   x x   52 6 52 6 2 2

1    x    12 x  ⇒

x

1 x

 12  2 3

31. Option (2) is correct.

28. Option (4) is correct. Explanation: Height of the kite = 120 m The angle is made by the string = 60° A

Explanation: Distance covered in one hour without stoppage = 80 – 72 = 8 km 8 × 60 min 80 = 6 minutes So, train stops for 6 minutes per hour

Required time = 120 m

60° C

B

s per the given information the figure is A drawn. The distance of kite from the ground (AB) = 120 m and ∠ACB = 60° The length of the string = AC

32. Option (1) is correct. Explanation: We know that, Period of stoppage per hour  Difference in speeds    60  minutes faster speed    80  64   60  minutes =  80   16     3  minutes  4 

MOCK TEST Paper-3in speeds  Difference

   60  minutes faster speed   80  64    60  minutes =  80 

85 36. Option (2) is correct. Explanation:

 16     3  minutes  4 

S.I. =

= 12 minutes

Principal =

33. Option (4) is correct. Explanation: Let, capacity of every container = x litre 3x litres In the I container, milk = 4 x Water = litres 4 5x In the 2nd container, milk = 7 litres 2x Water = 7 litres On mixing both, we get quantity of milk = =

3x 5x + 4 7

21x + 20 x 41x = litres 28 28

x 2x 7 x + 8x + = litres 4 7 28 15x litres = 28 41x 15x So, the required ratio = : 28 28 = 41x : 15x = 41 : 15 quantity of water =

34. Option (3) is correct. Explanation: Single equivalent xy   x  y  % 100   50  25     50  25  % 100  

discount

Principal × Time × Rate 100 S.I. 100 R  100   ` 50 Time  Rate 2R

37. Option (3) is correct. Explanation: According to question, B’s work = 1st, 3rd, 5th, 7th, 9th and 11th days = 6 days A work for 5

1 days 3

1 16 1   48 3 9 1 8 Remaining work = 1   9 9 None 8 ∴ B complete work in 6 days 9 6×9 days ∴ B complete 1 work in 8 Now, work done by A =

∴ Required time for B to finish 4 times of 6×9 ×4 work = 8 54 × 4 = 8 = 27 days

38. Option (1) is correct. Explanation: Let time taken by (B + C) in doing one work = x days So, time taken by A = x days 1 1 36 x (i)

= (75 – 12.5)% = 62.5% ∴ C.P. of article = (100 – 62.5)% of 2375

Now B‘s 1 day work =

 2375  37.5  =`   = ` 890.625 100   Actual Cost = ` 890.625 + 165 = ` 1055.625 To gain 62.5%, required S.P.

According to question, we have

From eqs. (i) and (ii), we get

 1055.625  162.5  =`   = ` 1715.39 100  

1 1 1 1    x 60 36 x

35. Option (4) is correct.

or

Explanation: Number of balls in bags x and y respectively 2a and 3a. ∴ 3a – 5 = 2a + 5 or a = 5 + 5 = 10 So, the total number of balls = 5a = 5 × 10 = 50 So, balls in each bag =

B’s 1 day work =

50 = 25 2

1 1 x 60 (ii)

2 1 1 35 8     x 60 36 180 180

or x 

180  2 360   45 8 8

∴ B’s 1 day work = =

1 1 36 45 54 1  180 180

OSWAAL CUET (UG ) 10 Mock Test Papers GENERAL TEST

86

Now time taken by B = 180 days 1 1 (A + C)’s 10 days work = 10     45 60  43 = 10    180  =

10  7 7  180 18

∴ Remaining work will be

= 11 ×

Explanation: The year 2001 was an ordinary year and in an ordinary year 1st day = Last day (remember) 1st January = 31st December As, given that, 1st January = Monday Hence, 31st December = Monday

43. Option (2) is correct. Explanation:

7 18  7 11 = 1   18 18 18 B’s work

42. Option (1) is correct.

180 = 110 days 18

39. Option (2) is correct. Explanation: As per question, we have Length of the rubber band = 3d + 2πr = (3 × 10 + 10π) cm = ( 30 + 10π) cm

40. Option (1) is correct. Explanation:   R  C.I  P  1  R  T  1 C.I  P  1  100 R   1 C.I  P  1  100   1  10 4   100    4000  1  10  44  1  4000  1  100 10   1  4000  1  100   1 4    11 100  4000  11  44  1  4000  10 11   1  4000  10   1 1011  11 11   11   4000  11  11  11  11  1 10 10 10 10  1   11  4000  11  11  11   4000  10  10  10  10  1 10  1010000 10  10   14641  4000  14641  10000  10000  4000  14641  10000  10000  4000   10000 4641    4000  4641  4000  10000 4641  4000  10000 10000 T T

= 4000 × 0.4641 = ` 1856.4

41. Option (4) is correct. Explanation:

will complete the given figure.

Reverse

 N O S A E R As, R E A S O N  Order N = 14 ⇒ 1 + 4 = 5, O = 4, S = 19 ⇒ 1 + 9 = 10 A = 1, E = 2, R = 18 ⇒ 1 + 8 = 9 I O U [∵ Codes for vowels A E ] 1 2 3 4 5 So, REASON = 5410129 Reverse  YHTLAEH and HEALTH  Order

= 25 ⇒ 2 + 5 = 7, H = 8 ⇒ 8 × 2 = 16 Y T = 20 ⇒ 2 + 0 = 2, L = 12 ⇒ 1 + 2 = 3 A = 1, E = 2, H = 8 ⇒ 8 × 2 = 16 So, HEALTHY = 716231216 Similarly, Reverse  EGLUVID DIVULGE  Order

= 2, G = 7, ⇒ 7 × 2 = 14, L = 12 ⇒ 1 + 2 = 3 E U = 5, V = 22, ⇒ 2 + 2 = 4, I = 3 D=4⇒4×2=8 Hence, DIVULGE = 21435438

44. Option (3) is correct. Explanation: T S

brother

Father P

brother

Mother R

Q

Therefore, it is clear that P is the grandson of T.

45. Option (2) is correct. Explanation:

MOCK TEST Paper-3

87

46. Option (4) is correct.

49. Option (1) is correct.

Explanation: As,

Explanation:

F

O

R

E

N

S

I

C

6

12

18

22

14

19

18

3

[∵ For vowels = Positional value in reverse order and for consonant = Positional value] a nd

D

O

R

M

4

12

18

13

i

Similarly, C

3

A

26

R

A

18

26

M

13

E

22

L

12

50. Option (1) is correct. Explanation: First fix the position of D, then, fix the position of other friends.

D B

47. Option (2) is correct. Explanation: Correct relationship between the given classes

E C

F G

Parents

Rich Persons

Farmers



51. Option (3) is correct. Explanation:

48. Option (2) is correct.

30 m

30 m

E

W S

20 m

Copies Envelopes

N

Starting point

End point 40 m

Explanation: According to the statements,



H A



Cards

Postcards

I. () II. () III. ()

Hence, conclusions I and III follow

I t is clear from the figure that from the starting point Lokesh is now in West direction.

OSWAAL CUET (UG ) 10 Mock Test Papers GENERAL TEST

88 52. Option (2) is correct.

55. Option (1) is correct.

Explanation:

Explanation: In the mirror image left becomes right and right becomes left.

Father

56. Option (4) is correct. Father

Explanation: Given :

Ritika

Son

÷ + = ∴ 54 ÷ 6 + 5 = 14 ⇒ 9 + 5 = 14

(Man)

Sister

53. Option (2) is correct.

54 * 6 * 5 * 14

57. Option (3) is correct. Explanation:

Explanation:

,

58. Option (3) is correct. Explanation:

50

54. Option (2) is correct. Explanation:

49 –1

Given,

47 –2

44 –3

40 –4

29

35 –5

–6

22 –7

59. Option (4) is correct. Explanation:

12 Aman

So,

Baman 18



o, Maximum boys in the row = 12 + 6 + 18 S = 36

will complete the figure.

60. Option (2) is correct. Explanation: Both conclusions are correct according to given statements.



SOLUTIONS OF Question Paper 1. Option (4) is correct. Explanation: Ahom Dynasty: The Ahom Dynasty ruled the Ahom Kingdom in the region of Assam, which is now a state in northeastern India. The dynasty lasted for several centuries, with its origins dating back to the 13th century. The Ahom rulers established a strong administration and a unique culture in Assam. They successfully resisted Mughal invasions and maintained their sovereignty for a significant period of time. Parmar Dynasty: The Parmar dynasty ruled in the Malwa region of central India. The dynasty is known for its rule during the medieval period, particularly in the 9th to 14th centuries. The Parmar rulers played a significant role in the political landscape of central India and contributed to the cultural and architectural development of the region. Gurjar Pratihara Dynasty: The Gurjar Pratihara dynasty ruled in the northwestern region of India, with its capital at Ujjain. They were prominent during the medieval period and played a major role in the history of the Indian subcontinent. The dynasty is known for its expansion, military prowess, and patronage of art and culture. Chandel Dynasty: The Chandel Dynasty ruled the region around Khajuraho, known for its famous group of temples with intricate sculptures. The dynasty existed during the medieval period and was centred in presentday Madhya Pradesh, India. The Khajuraho temples are renowned for their stunning architecture and artistic representations.

2. Option (3) is correct. Explanation: The United Nations Organisation (UNO) is headquartered in New York City, USA. The UNO is the main international organization for global cooperation. It was founded in 1945 after World War II to prevent future wars and to promote peace and security. The UNO has 193 member states. The United Nations Children's Emergency Fund (UNICEF) is also headquartered in New York City, USA. UNICEF is a specialized agency of the UNO that works to improve the lives of children around the world. It provides

4

humanitarian assistance to children in need, promotes child rights, and supports early childhood development. The United Nations Security Council (UNSC) is headquartered in New York City, USA. The UNSC is one of the six principal organs of the UNO. It is responsible for maintaining international peace and security. The UNSC has five permanent members (China, France, Russia, the United Kingdom, and the United States) and 10 rotating members. The United Nations Educational, Scientific and Cultural Organization (UNESCO) is headquartered in Paris, France. UNESCO is a specialized agency of the UNO that promotes education, science, culture, and communication. It works to build peace in these areas and to foster international cooperation.

3. Option (1) is correct. Explanation: The “Treaty of Salbai” was signed on 17 May 1782 by representatives of the Maratha Empire and the British East India Company after long negotiations to settle the outcome of the first Anglo-Maratha War. Salbai is situated in Gwalior District, Madhya Pradesh.

4. Option (3) is correct. Explanation: Lord Canning: As the GovernorGeneral of India from 1856 to 1862, Lord Canning held a prominent role. During his tenure, the pivotal Government of India Act, of 1858 was enacted. This legislation established that the position of Viceroy would be held by the same individual serving as the GovernorGeneral of India. In addition to his role as Governor-General, Lord Canning assumed the position of the inaugural Viceroy of India. Notably, he oversaw the withdrawal of the "Doctrine of Lapse" policy during his time. Lord Ripon: Serving as India's Viceroy from 1880 to 1884, Lord Ripon's term was marked by significant events. Noteworthy actions during this period include the repeal of the Vernacular Press Act in 1882, which aimed to provide greater freedom to the press. Additionally, a resolution introduced

OSWAAL CUET (UG ) 10 Mock Test Papers GENERAL TEST

90

in 1882 marked the initiation of local selfgovernance in India. The year 1882 also saw the establishment of the Hunter Commission, which focused on educational reforms. Lord Ripon's involvement with the compromised state of the Ilbert Bill is also of importance. Apart from his governance, Lord Ripon's notable contribution includes his renowned work "The Imperial Gazetteer of India," a project he embarked upon in 1869. Warren Hastings: Warren Hastings, the inaugural Governor-General of Bengal, played a significant role in reshaping administrative structures. During the period of the Dual System, the East India Company held Diwani rights, encompassing revenue collection, while administrative control rested with the Nizam or Indian chiefs. Hastings took action to dismantle the Dual System, originally established by Robert Clive. Notably, he reduced Nawab's annual allowance from Rs. 32 lakhs to Rs. 16 lakhs, bringing about a financial adjustment in the region. Lord Dufferin, who held the office of Viceroy of India from 1884 to 1888, was in power during the occurrence of the Third AngloBurmese War between 1885 and 1886. In a parallel instance, the Second Anglo-Burmese War transpired during the administration of Lord Dalhousie, who was in office from 1848 to 1856. The outcome of the Second Anglo-Burmese War was the British gaining control over Lower Burma. In contrast, the Third Anglo-Burmese War resulted in the British capturing the entirety of Burma and subsequently integrating it as a province of India. The transformation of Burma into a distinct crown colony took place in the year 1935.

5. Option (3) is correct. Explanation: Rajasthan is the largest Indian state in terms of area. It is located in the northwestern part of India and borders Pakistan to the west. Rajasthan is known for its desert landscapes, forts, and palaces. The capital of Rajasthan is Jaipur. Madhya Pradesh is the second-largest Indian state in terms of area. It is located in the central part of India and borders Rajasthan to the northwest, Uttar Pradesh to the north, Chhattisgarh to the east, Maharashtra to the south, and Gujarat to the southwest. Madhya Pradesh is known for its forests, wildlife, and temples. The capital of Madhya Pradesh is Bhopal.

Mahe is the smallest district in India by area. It is located in the Union Territory of Puducherry. Mahe is a coastal district and is surrounded by the state of Kerala. The capital of Mahe is Mahe. Kutch is the largest district in India by area. It is located in the state of Gujarat. Kutch is a desert district and is known for its salt flats. The capital of Kutch is Bhuj. Leh is the second largest district in India by area. It is located in the Union Territory of Ladakh. Leh is a mountainous district and is known for its Buddhist monasteries. The capital of Leh is Leh.

6. Option (1) is correct. Explanation: Rice: China is the world's largest producer of rice, followed by India and Indonesia. Corn: The United States is the world's largest producer of corn, followed by China and Brazil. Jute: India is the world's largest producer of jute, followed by Bangladesh and China. Wheat: China is the world's largest producer of wheat, followed by India and the United States.

7. Option (4) is correct. Explanation: Fiscal deficit: The fiscal deficit is the difference between the government's total expenditure and its total revenue, excluding borrowings. It is a measure of the government's borrowing requirement for the year. Revenue deficit: The revenue deficit is the difference between the government's total revenue and its total revenue expenditure. It is a measure of the government's inability to meet its revenue expenditure from its own revenue sources. Primary deficit: The primary deficit is the difference between the fiscal deficit and the interest payments. It is a measure of the government's borrowing requirement for the year, excluding interest payments.

8. Option (3) is correct. Explanation: M icrophone is an input device used to input sound that is then stored in a digital form. Whereas, a touch pad is a locator device for controlling input positioning on a computer display screen. It is used as an alternative to the mouse. Monitors, commonly called as VDU (also known a Visual Display Unit), are the main output device of a computer.

MOCK TEST Paper-4 9. Option (2) is correct. Explanation: Trochodendron is a genus of flowering plant with one living species fewer than 200 species of angiosperms have a primitive feature of vesselless wood with two exceptions - Trochodendron and tetracentron all vesselless angiosperms occurs in magnolidae.

10. Option (3) is correct. Explanation: A thump of either a single hind leg or both together shows the rabbit’s fear or displeasure over something. In any case, it shows the identification of some danger in the vicinity by the rabbit. The rabbit feels the danger and warns the rest of his warren.

11. Option (4) is correct. Explanation: The Bharat Ratna is the highest civilian award in India. It was instituted on January 2, 1954, by the President of India, Rajendra Prasad. The first recipients of the Bharat Ratna were C. Rajagopalachari, Sarvepalli Radhakrishnan, and CV Raman. They were awarded the award in 1954. Khan Abdul Ghaffar Khan and Nelson Mandela were the first two non-Indians to be awarded the Bharat Ratna. They were awarded the award in 1987 and 1990 respectively. Mother Teresa was a naturalized Indian citizen. She was awarded the Bharat Ratna in 1980.

12. Option (2) is correct. Explanation: The theme of World Cancer Day 2023 is "Close the Care Gap". This theme was chosen to highlight the inequities in cancer care that exist around the world. People in lowand middle-income countries are more likely to die from cancer than people in high-income countries. This is due to a number of factors, including lack of access to early detection and treatment, as well as poor quality of care. The campaign "Close the Care Gap" aims to raise awareness of these inequities and to call for action to address them. The campaign has four key messages: We can, I can: We all have a role to play in closing the care gap. We can raise awareness, donate to cancer charities, and support cancer research. Close the care gap: We need to work together to close the care gap. This means governments, healthcare providers, and individuals all working together to improve access to cancer care.

91 I Am and I Will: We can all make a difference. We can be advocates for cancer care, and we can take steps to reduce our own risk of cancer. Not Beyond Us: Closing the care gap is not beyond us. It is a challenge, but it is one that we can overcome if we work together.

13. Option (2) is correct. Explanation: The Laser (light amplification by stimulated emission of radiation) was invented by T.H. Maiman (Theodore Harold Maiman) in 1960 at Hughes Research Laboratories. Maiman’s functional laser used a solid-state flashlamppumped synthetic ruby crystal to produce red laser light, at 694 nanometers wavelength.

14. Option (1) is correct. Explanation: A coup is a sudden overthrow of a government illegally. A strike is a work stoppage caused by a huge number of employees refusing to work. Coup or an overthrow, is the illegal and overt seizure of a state by the military or other elites within the state apparatus.

15. Option (4) is correct. Explanation: According to Article 361 of the Constitution of India, a criminal proceeding cannot be instituted in a court against the Governor during his term of office. The President or Governor, while in the office is not personally answerable to any court for the exercise and execution of the powers.

16. Option (4) is correct. Explanation: Acetic acid is a significant component of vinegar. Its concentration specifies the strength of vinegar. It is an uncolored liquid with a solid eroding pungent aroma. As vinegar and an acidity regulator, acetic acid is widely utilized in the food industry.

17. Option (4) is correct. Explanation: Methane is the hydrocarbon having the lowest molecular mass with the chemical formula CH4. It is called Marsh gas because it is released in vast amounts from marshes where anaerobic bacteria decompose matter. Alessandro Volta discovered it in 1776. It is the main constituent of biogas and Compressed Natural Gas (CNG).

18. Option (2) is correct. Explanation: Aga Khan Cup is a hockey tournament held in Mumbai. It is one of the oldest hockey tournaments in India, having been started in 1896. The cup is named after the Aga Khan, the spiritual leader of the Ismaili Muslims.

OSWAAL CUET (UG ) 10 Mock Test Papers GENERAL TEST

92

Begum Hazrat Mahal Cup is a football tournament held in India. It is named after Begum Hazrat Mahal, the queen of Awadh who fought against the British in the Indian Rebellion of 1857. Ramanuja Trophy is a table tennis tournament held in India. It is named after Ramanuja, an 11th-century Hindu philosopher and theologian. Walker Cup is a golf tournament held between the United States and Great Britain and Ireland. It is the oldest international team golf competition in the world, having been started in 1922.

19. Option (3) is correct. Explanation: Sriharikota is a bare island off the coast of Andhra Pradesh in Southern India. It has India’s only satellite launch center at the Satish Dhawan Space Center (SHAR) and is used by the ISRO to launch satellites with multistage rockets such as polar satellite launchers and geostationary satellite launchers. Originally called the Sriharikota High Range and later the Sriharikota Launch Range. The center was renamed in 2002 following the death of former ISRO President Satish Dhawan.

20. Option (2) is correct. Explanation: Flamingo Festival is celebrated in Andhra Pradesh, near Pulicat Lake. It is a three-day festival that is organized during the winter season when thousands of migratory flamingos visit the region. The festival includes cultural events, boat rides, and bird watching. Pongal is a harvest festival celebrated in Tamil Nadu, Karnataka, and Andhra Pradesh. It is a four-day festival that is celebrated in January or February. The first day of Pongal is called Bhogi Pongal, when people clean their homes and burn old clothes and materials. The second day is called Surya Pongal, when people offer prayers to the sun god. The third day is called Mattu Pongal, when people worship cattle. The fourth day is called Kanum Pongal, when people visit their relatives and friends. Kambala Festival is celebrated in Karnataka, especially in the Dakshina Kannada district. It is a buffalo race that is held during the winter season. The race is held in a muddy field, and the buffaloes are decorated with colourful clothes and bells. Theyyam Festival is celebrated in Kerala. It is a ritualistic dance that is performed by Theyyam artists. The dancers wear elaborate costumes and masks, and they represent different gods and spirits. The Theyyam Festival is held in temples and public places.

21. Option (2) is correct. Explanation: 25 25 Given:  % of 1600  % of 800 4 2 

1600  25 800  25  400 200

= 100 + 100 = 200

22. Option (4) is correct. Explanation: Given: S.P. = `1000, Loss = 10% 100 ∴ Cost price of sewing machine  1080  90 = ` 1200 1200 × 110 So, S.P. for a profit of 10% = 100 = ` 1320

23. Option (4) is correct. Explanation: Let the length of flag = l In the triangle DBC, 10 tan 30° = BC 1 10 ⇒ = 3 BC ⇒

BC = 10 3

Now in DABC, 10 + l BC 10 3 = 10 + l l = 10 (1.732 − 1) = 7.32 m

tan 45° = ⇒ ⇒

24. Option (3) is correct. Explanation: Given numbers = 11, 15, 13, 120, 10, 8, 11, 7, 15, 11, 13, 7, 11 Ascending order of given numbers = 7, 7, 8, 10, 11, 11, 11, 11, 12, 13, 13, 15, 15 Mean of numbers = Sum of 13 Numbers = 144 Median of numbers = 11 (Mid number of given data) Mode of numbers = 11 (Most frequent number in given data) So, mean > mode = Median

25. Option (1) is correct. Explanation: Single equivalent discount 10  5     10  5   %  14.5% 100   ∴ Amount to be paid = (100 – 14.5)% of 110 =

110 × 85.5 = ` 94.05 = ` 94 100

MOCK TEST Paper-4

93

26. Option (1) is correct. Explanation: Given: S.I. = `60, Time = 5 years S.I. × 100 Principal = Time × Rate =

60  100 6000   ` 200 56 30

27. Option (4) is correct.

1 1  or 12   1 x  3 x  3   or x  3  x  3  1 ( x  3)( x  3) 12

or 24 × 12 = 36 × D2 or

D2 =

24  12  8 days 36

31. Option (3) is correct. Explanation: x 3 5 On squaring both sides, we get x  3  5  2 15

( 24 )  ( 24 )2  4  ( 9 ) 2 by quadratic formula 24  576  36 24  612  2 2

24  36  17 24  6 17  2 2 Q x cannot be negative =

∴x=

Explanation: We know that,

or x  8   2 15

or x2 – 9 = 24x or x2 – 24x – 9 = 0

=

30. Option (2) is correct. M1D1 = M2D2

Explanation: Let speed of motorboat in still water be x kmph Speed of stream = 3 km/h ∴ Downstream rate = (x + 3) kmph Upstream rate = (x – 3) kmph According to question, we have 12 12 60   x  3 x  3 60

or x =

31 1  or 3x = 4 × 9 3x 9 49 or x =  12 days 3 or

6( 4  7 )  3( 4  17 ) km/h 2

28. Option (4) is correct. Explanation: HCF of 391 and 323.

On squaring again both sides, we get x2 – 16x + 64 = 60 or x2 – 16x + 4 = 0 ∴ x2 – 16x + 6 = 2

32. Option (2) is correct. Explanation: 66049  257  257 = 257 So, the unit place in the square root of 66049 is 7

33. Option (2) is correct. Explanation: Given that, ∆ABC is an isosceles triangle. AB = BC Point D is the mid point of side BC and PD is internal bisector of ∠ADB. According to the question, A

323 391 1 323

68 323 4 272 51 68 1 51



17 51 3 51 x

∴ Number of classes = 17

29. Option (4) is correct Explanation: Let A complete work in x days ∴ time taken by B to complete the work = 3x days According to question, we have 1 1 1   x 3x 9

P B

Q

D

C

∠ADB = ∠ADC = 90° and ∠PDA = 45° Q PQ || BC ∴ ∠ADQ = 45° ∴ PDQ = 45° + 45° = 90°

34. Option (3) is correct. Explanation: Students who score 30 or more than 30 marks but less than 40 marks = 44 – 25 = 19. Students who score 20 or more but less than 30 marks = 61 – 44 = 17. ∴ Required answer = 19 + 17 = 36.

OSWAAL CUET (UG ) 10 Mock Test Papers GENERAL TEST

94 35. Option (3) is correct.

39. Option (3) is correct.

Explanation: (x + y)2 = 4xy or x2 + y2 + 2xy – 4xy = 0 or (x - y)2 = 0 or x = y or x : y = 1 : 1

36. Option (3) is correct. Explanation: Given that, Side of equilateral triangle = 12 cm Now, we know that, Height of equilateral triangle = =

3 × 12 2

3 × side 2

= 6 3 cm

37. Option (1) is correct. Explanation: Given: Average of L, M and N = 93 kg L + M + N = 3 × 93 = 279 kg

….(i)

L+ M = 89 2 ⇒ L + M = 89 × 2 = 178 kg

….(ii)

M+N = 96.5 2 ⇒ M + N = 96.5 × 2 = 193 kg

….(iii)

and

Explanation: Let the 2principal be ` P  R 2  C.I. P  1  R   1 C.I. P  1  100   1  100   2  5 2  328  P  1  5   1 328  P  1  100   1  100   2  21  2   328  P  21   1  328  P  20   1  20      441   328  P  441  1   328  P  400  1   400   441  400   328  P  441  400   328  P  400   400  41P  328  41P  328  400 400 328  400  P  328  400  P 41 41 131200  P  131200  P  41 41 P = ` 3200

40. Option (2) is correct. Explanation: In 100 kg of alloy the quantity of nickel 2 =  100  20 10 Suppose x kg of nickel be mixed. 20 + x 3 = 100 + x 11 or 220 + 11x = 300 + 3x or 11x – 3x = 300 – 220 80 = 10 kg or 8x = 80 or x = 8 So, =

Adding eqn (ii) and (iii), L + M + N + M = 371 ⇒ M = 371 – (L + M + N) ⇒ M = 371 – 279 ⇒ M = 92 kg

38. Option (1) is correct. Explanation: Let the distance between A and B = x km

41. Option (1) is correct. Explanation: Correct relation between the given classes, Literates

Then speed of train

x 4x   km / h 45 3 x 4x  60  km / h 45 3 4 x When speed is48reduced by 5 km/h then 60   4x 60 5 x 5 48 4   then 3 4x 3x  5 4 60 5  3  4 x  15 5 3x 4   4 x  15 5 ⇒ 16x – 60 = 15x ⇒ x = 60 km

Graduates

Hardworking



42. Option (4) is correct. Explanation: According to the statements, Glasses Bowls



Cups

Utensils Plates

I. () II. ()

Hence, neither conclusion I nor II follows.

MOCK TEST Paper-4

95

43. Option (1) is correct.

49. Option (4) is correct.

Explanation: In the mirror image left becomes right and right becomes left.

44. Option (2) is correct.

Explanation: According to the question, who

Explanation: As ‘Turmeric’ is ‘Spices’, same as ‘Dates’ are ‘Dry Fruits’.

you  

are

they

4  3  4  8 

you  

is

2 5

(i) (ii)

are dangerous 2 9 5 (iii) they rom equation (i), (ii), and (iii), dangerous = 9. F

45. Option (2) is correct. Explanation:

50. Option (1) is correct.

As, R A J Positional   18 1 10 Value

Explanation:

Now, (18 + 1 + 10 = 29) × 3

O

(total alphabets in the word) = 87

R +3

Positional

  7 9 20 1 and G I T A   Value

U +3

X +3

A +3

D +3

51. Option (4) is correct.

Now, (7 + 9 + 20 + 1 = 37) × 4 = 148 Similarly, Positional

  22 1 18 21 14 V A R U N   Value Now, (22 + 1 + 18 + 21 + 14 = 76) × 5 = 380

46. Option (2) is correct. Explanation: According to the information,

Explanation: Assertion: Leakages in household gas cylinders can be detected. It is true. Household gas cylinders have LPG in them but it is odorless. For leak detection in household cylinders, ethyl mercaptan is added to it as it has a smell of rotten cabbages which help in the detection of leaks. Reason: LPG has a strong smell. It is false as it is odorless.

52. Option (2) is correct. Explanation: Given :

o, maximum cars in the row = 6 + 1 + 9 + 1 + S 8 + 1 = 26

53. Option (4) is correct. Explanation:

47. Option (2) is correct.

4

Explanation: Devansh Dileep Father Son Sister Virat Preeti Manoj

∴ Maaya is the mother-in-law of Preeti.

48. Option (3) is correct. Explanation:

Maaya

Hence, required distance dotted line XY, i.e (17 + 7) = 24m, and Y is in east direction with respect to X.

54. Option (1) is correct. Explanation: T

R –2

P –2

N –2

L –2

J –2

H –2

OSWAAL CUET (UG ) 10 Mock Test Papers GENERAL TEST

96

55. Option (1) is correct. Explanation: So,

58. Option (4) is correct. Explanation:

2

8

32

×4

×4

128 ×4

512

×4

2048

×4

59. Option (3) is correct. will complete the figure.

Explanation: Staring point E 10 km A 3 km

56. Option (3) is correct. Explanation: Both the conclusions have information which does not follow from given statements. Hence, neither I nor II follows.

57. Option (3) is correct. Explanation: ‘Roar’ is the sound made by lion. Similarly, ‘Trumpet’ is the sound made by elephant.

D

6 km

N

B 3 km C

E

W S

Given, DC = BE = 6 km Required distance AE = AB – BE ∴ = 10 – 6 = 4 km Hence, he is at E, 4 km to the East from starting point A.

60. Option (1) is correct. Explanation: Only grandson of Anup’s mother means the son of Anup. Mother of Anup’s son means the wife of Anup. Only daughter of Anup’s wife is also the daughter of Anup.



SOLUTIONS OF Question Paper 1. Option (4) is correct. Explanation: Bimbisara (544 - 492 BC) - He founded the Haryanka dynasty. He's also known as Shrenika. He lived during the time of Lord Buddha. Bimbisara had three wives: Kosaladevi, Chellana, and Khema. When the ruler of Avanti, Pradyota, suffered from Jaundice, he sent the royal physician Jivaka to help. Ajatashatru (492 - 460 BC) - He seized the throne by killing his father. Ajatashatru had other names: Kunika and Ashokachanda. He defeated his maternal uncle, Prasanjit, and married Prasanjit's daughter, Vajira. The famous weapons Mahasilakantaka and Rathamusala were associated with him. He organized the first Buddhist council in Rajgriha after Buddha's death. Udayin (460 - 444 BC) - Udayin was the son of Ajatashatru. He designated Pataliputra as the capital of Magadha and built a fort at the confluence of the Ganges and Son rivers in Patna. Kalashoka (394 - 366 BC) - Sishunaga Dynasty Kalashoka was also known as Kakavarna. The second Buddhist council was held in Vaishali during his reign.

2. Option (2) is correct. Explanation: X-rays were discovered by Wilhelm Conrad Roentgen in 1895. Roentgen was a German physicist who was studying the properties of cathode rays (streams of electrons) when he noticed that a fluorescent screen nearby was glowing even though it was not connected to his electrical equipment. He realized that he had discovered a new kind of radiation that could pass through solid objects. Roentgen named his discovery "X-rays" because he did not know what they were. Electrons were discovered by JJ Thomson in 1897. Thomson was an English physicist who was studying cathode rays when he found that they were negatively charged particles. He called these particles "corpuscles," but they are now known as electrons. Thomson's discovery of the electron was a breakthrough in physics, and it helped to lay the foundation for our understanding of atomic structure.

5

Protons were discovered by Ernest Rutherford in 1917. Rutherford was a New Zealand-born physicist who was studying the scattering of alpha particles by gold foil. He found that a small fraction of the alpha particles were deflected through large angles, which he could only explain if the atom had a small, dense nucleus that was positively charged. Rutherford called the particles in the nucleus "protons."

3. Option (1) is correct. Explanation: Foehn: The Foehn is a warm, dry wind that occurs in the Alps. It is caused by the adiabatic heating of air as it descends from the mountains. The Foehn is known for its warm, dry weather and its ability to melt snow quickly. Sirocco: The Sirocco is a hot, dry wind that blows from the Sahara Desert to the Mediterranean Sea. It is known for its dusty, hot weather and its ability to cause sandstorms. Chinook: The Chinook is a warm, dry wind that occurs in the Rocky Mountains. It is caused by the adiabatic heating of air as it descends from the mountains. The Chinook is known for its warm, dry weather and its ability to melt snow quickly. Mistral: The Mistral is a cold, dry wind that blows from the north along the western coast of France. It is known for its strong winds and its ability to cause damage.

4. Option (1) is correct. Explanation: Delhi and Noida share the status of twin cities. Kolkata and Howrah are recognized as twin cities. Kolkata is situated on the western bank of the Hooghly River. Durgapur and Asansol are often referred to as twin cities. Hyderabad and Secundrabad bear the distinction of being twin cities. The capital of Telangana is Hyderabad. Cuttack and Bhubaneswar in Odisha are also designated as twin cities. Pune and Pimpri-Chinchwad in Maharashtra are commonly referred to as twin cities.

OSWAAL CUET (UG ) 10 Mock Test Papers GENERAL TEST

98

Hubli and Dharwad in Karnataka share the status of twin cities. Durg and Bhilai in Chattisgarh are also acknowledged as twin cities.

5. Option (1) is correct. Explanation: The “FOCAL LENGTH” improves when the human eye lens evolves light. When the eye muscles are comforted, the lens-thin and the focal size grow to make feasible the distant objects to see. When we look at objects nearer to the eye, the ciliary muscles contract and the lens becomes thick.

6. Option (4) is correct. Explanation: The 42nd Amendment to the Indian Constitution, enacted in 1976, established Administrative Tribunals in India. The original constitution did not include provisions regarding tribunals, but the 42nd Amendment introduced a new part, XIV-A, and Article 323A. Article 323A grants the Parliament the authority to create administrative tribunals. These tribunals handle disputes and complaints related to the recruitment and terms of employment for individuals in public service positions associated with the Union and the States. The 24th Amendment to the Indian Constitution, which followed, mandated that the president must provide assent to a constitutional amendment bill. Subsequently, the 44th Amendment removed the right to property from the list of fundamental rights. The First Amendment, passed in 1951 by the Provisional Parliament, occurred right after the Constitutional Assembly completed drafting the Constitution. This amendment modified articles 15, 19, 85, 87, 174, 176, 341, 342, 372, and 376. It allowed for the preservation of laws related to estate acquisition, introduced the Ninth Schedule to shield land reforms and included rules from judicial review, and inserted Articles 31A and 31B after Article 31.

7. Option (3) is correct. Explanation: “A record 13.5 crore people moved out of multidimensional poverty between 2015-16 and 2019-21 as per NITI Aayog’s Report ‘National Multidimensional Poverty Index: A Progress Review 2023’” is correct. The report found that the percentage of people living in multidimensional poverty in India declined from 24.85% in 2015-16 to 14.96% in 2019-21. This means that a total of 13.5 crore people moved out of multidimensional poverty during this period.

8. Option (4) is correct. Explanation: National Electronic Funds Transfer (NEFT) is a nationwide centralized payment system owned and operated by the Reserve Bank of India (RBI). It allows you to transfer funds from one bank account to another within India. IFSC or Indian Financial System Code is an 11 digits alpha-numeric code that uniquely identifies a bank branch participating in the NEFT system. It is used to facilitate electronic fund transfers between banks. MICR Code or Magnetic ink character recognition(MICR) is a 9 digits code printed on the cheque which enables identification of the cheques and identifies the bank and branch participating in an Electronic Cheque Clearing system. It is used to process cheques electronically. Swift Code (Society for Worldwide Interbank Financial Telecommunication) is an internationally recognized system for identifying banks and financial institutions globally. It is used to facilitate cross-border payments.

9. Option (3) is correct. Explanation: Read Only Memory (ROM) is read only, it cannot be changed, it is permanent and non-volatile, meaning it also holds its memory even when power is removed. It is a type of medium for storage that stores data permanently on personal computers (PCs) and other electronic devices.

10. Option (2) is correct. Explanation: Full-duplex data transmission is used to transmit data in both directions on a signal carrier at the same time. For example, on a local area network with a technology that has full-duplex transmission one workstation can be receiving data while another workstation is sending data on the line.

11. Option (3) is correct. Explanation: Cloves are the aromatic dried flower buds of a tree in the family Myrtaceae, Syzygium Aromaticum. They are native to the Maluku Islands in Indonesia and are commonly used as a spice. The flower buds are at first of a pale color and gradually become green, after which they develop into a bright red, when they develop into a bright red they are harvested by farmer.

MOCK TEST Paper-5 12. Option (2) is correct. Explanation: The People's Republic of China, situated in East Asia and bordering India and other countries, holds the title of the world's most populous nation and ranks as the fourthlargest in terms of land area. Its head of state is the President. China lies to the north of India, and Indian states that share borders with China include Ladakh, Himachal Pradesh, Sikkim, Arunachal Pradesh, and Uttarakhand. Afghanistan, located in Central Asia, is officially known as the Islamic Republic of Afghanistan. It spans a total land area of 652,230 square kilometres and is characterized by its predominantly Islamic population. The country's economy relies significantly on agriculture. In nearby countries like India, Afghanistan engages in industrial activities such as the production of cotton textiles, woollens, woven carpets, and handicrafts. Afghanistan shares a border with Pakistanoccupied Kashmir. Bhutan, situated to the north of India and officially recognized as The Kingdom of Bhutan, shares borders with the Indian states of Arunachal Pradesh, Sikkim, Assam, and West Bengal. It is a landlocked country with an area of 38,394 square kilometres and its capital city is Thimphu. Bhutan is governed by a constitutional monarchy and its economy is primarily driven by hydropower, agriculture, forestry, and tourism. Among India's neighbouring countries, Bhutan is the smallest. Bangladesh, officially named the People's Republic of Bangladesh, is positioned in South Asia and shares one of the world's longest borders with India. The capital city is Dhaka, and Indian states such as Assam, Mizoram, Tripura, Meghalaya, and West Bengal share their borders with Bangladesh. With a population of 161 million, Bangladesh ranks as the eighth-most populous country globally and is known for its high population density. Bangladesh was initially part of Pakistan, known as East Pakistan, but gained independence after the 1971 Liberation War.

13. Option (4) is correct. Explanation: United Nations Conference on Trade and Development (UNCTAD): The UNCTAD headquarters is located in the Palais des Nations in Geneva, Switzerland. The Palais des Nations is a complex of buildings that was initially built as the headquarters of the League of Nations. The UNCTAD was established in 1964 to promote international trade and development.

99 orld Health Organisation (WHO): The W WHO headquarters is located at Avenue Appia 20 in Geneva, Switzerland. The WHO was established in 1948 to promote health and well-being for all people. The WHO is the leading international authority on public health. International Labour Organisation (ILO): The ILO headquarters is located in International Labour Office in Route des Morillons 1 in Geneva, Switzerland. The ILO was established in 1919 to promote social justice and decent work for all. The ILO is the only tripartite United Nations agency, meaning that it brings together governments, employers, and workers to make decisions. Organisation for the Prohibition of Chemical Weapons (OPCW): The OPCW headquarters is located in The Hague, Netherlands. The OPCW was established in 1997 to prohibit the development, production, stockpiling, and use of chemical weapons. The OPCW is the implementing body of the Chemical Weapons Convention, which is an international treaty that was adopted in 1993.

14. Option (4) is correct. Explanation: The Arjuna Award is awarded by the Ministry of Youth Sports of the government of India in recognition of the excellence of national sports. The award was established in 1961 and is presented with a statue and a scroll of Arjuna for 500,000 rupees.

15. Option (3) is correct. Explanation: ‘Agha Khan Cup’ is associated with hockey tournament. It is a very famous tournament played in 1896. It is one of the most oldest hockey tournaments of India.

16. Option (3) is correct. Explanation: R adar (radio detection and ranging) is an object detection system that uses radio waves to determine the distance, height, speed or direction of an object. It can be used to identify aircraft, spacecraft, ships guided missiles and engines, vehicles, metrological structures and terrain.

17. Option (1) is correct. Explanation: Malic acid is seen in apples. It is a dicarboxylic acid that exists in grapes, wines, and rhubarb. It imparts a tart flavor to a part of the meals. Formic acid is seen in ants.

18. Option (4) is correct. Explanation: Ethyl acetate odor. This uncolored sweet aroma is utilized in decaffeinating tea and coffee. Acetate has a vigorous liquid that has distinctive glue also used as nail polish removers.

OSWAAL CUET (UG ) 10 Mock Test Papers GENERAL TEST

100 19. Option (2) is correct.

Explanation: ‘World Ozone Day’ is celebrated on 16 September every year since 1995. This day reminds the signing of the Montreal Protocol on substances that depletes the Ozone layer. The theme of World Ozone day 2021 was Montreal Protocol – keeping us, our food and vaccines cool.

24. Option (1) is correct. Explanation: The length of the shadow of tree is Height of tree is 30 m.

10 m. 3

A

20. Option (1) is correct. Explanation: Bhavai is a folk dance of Gujarat and Rajasthan also. The dancers perform on pieces of glass and sword in the dance. First Bhavai dancer was Mrs. Krishna Vyas Chhangani, who born in Jodhpur.

30

θ B

21. Option (3) is correct. Explanation: Given S.P. of 20 apples = `100 Gain = 20% C.P. of 20 apples,  120c.p.   100  100    1000 C.P. = 12 No. of apples that can be bought in `100 20 = 1000 × 100 12 = 24.

22. Option (3) is correct. Explanation: Let the roots of quadratic equation are a and b. Given that: Arithmetic mean of quadratic equation = 8 α+β ∴ =8 2 or α+β =16 Geometric mean of quadratic equation = 5 αβ = 5 αβ = 25 Using, quadratic equation ⇒ x2 – (a + b) x + ab = 0 ⇒ x2 – 16x + 25 = 0 So, option (3) is correct.

23. Option (1) is correct. Explanation: Suppose the numbers be x, 2x and 4x. As per question, x2 + (2x)2 + (4x)2 = 1029 or x2 + 4x2 + 16x2 = 1029 or 21x2 = 1029 1029 = 49 or x2 = 49 or x = 7 or x2 = 21 So, the required difference = 4x - x = 3x = 3 × 7 = 21

10

3

C

et us assume that AB be the height of the tree L and BC be the length of the shadow. Let the angle of elevation of the sun be θ. In the given triangle, AB tan θ = BC 30 10 3 3 ⇒ tan θ = 3



tan θ =

⇒ tan θ =

3

⇒ tan θ = tan 60° So, θ = 60° The angle (in degrees) of elevation of the sun is 60°.

25. Option (1) is correct. Explanation: According to the question, A’s 4 days work = B’s 5 days work A : B = 4 : 5 and B : C = 6 : 7 Now A : B : C = (4 × 6) : (5 × 6) : (5 × 7) = 24 : 30 : 35 Time taken by A = 7 days ∴ Time taken by C =

35 245 5 7   10 days 24 24 24

26. Option (2) is correct. Explanation: 67127y76x2 is divisible by 88. So, the given number is divisible by 8 and 11, As 632 is divisible by 8 so, x = 3 (6 + 1 + 7 + 7 + 3) – (7 + 2 + y + 6 + 2) = 0 ⇒ 24 – (17 + y) = 0 ⇒ 24 – 17 – y = 0 ⇒7–y=0 ⇒y=7 ∴ 7x – 2y = 7 × 3 – 2 × 7 = 21 – 14 = 7

MOCK TEST Paper-5

101

27. Option (4) is correct. Explanation: According to Rule Let the speed of the second train be x m/sec Speed of first train

29. Option (3) is correct. Explanation: Given that, AD ⊥ BC,

A

150 = = 10 m / s 15 Relative speed of trains = (x + 10) m/sec Total distance covered = (150 + 150) = 300 metre So, time taken = 300 x + 10 300 ⇒  12 x  10 ⇒ 12x + 120 = 300 ⇒ 12x = 300 – 120 = 180 180 x  15m / sec 12 15  18  km / hr 5

= 54 km/hr

28. Option (1) is correct. Explanation: LCM of 15, 18, 20 and 27. 2

15, 18, 20, 27

2

15, 9, 10, 27

3

15, 9, 5, 27

3

5, 3, 5, 9

3

5, 3, 5, 3

5

5, 1, 5, 1 1, 1, 1, 1

∴ LCM of 15, 18, 20 and 27 = 2 × 2 × 3 × 3 × 3 × 5 = 540 According to the question, ∴ x = 540k + 10, Where k is a position integer = 31 × 17k + 13k + 10 Here, 13k + 10 must be divisible by 31. For k = 4, 13 × 4 + 10 = 62, which is divisible by 31 ∴ x = 540 × 4 + 10 = 2170 Now, 4

2170

4

16

86

570

6

516

46

B

D

C

Suppose, AB = 2x units or BD = DC = x units ∴ AB : BD =2:1

30. Option (2) is correct. Explanation: Let the number of trees in the garden be x. Then, 60 Coconut trees = x 100 Mango trees = Apple trees =

25 60 15  x x 100 100 20 20 15 3x x  100 100 100

According to the question, Apple trees = 1500 3x = 1500 ⇒ 100 ⇒ ⇒

x = 500 × 100 x = 50,000.

31. Option (3) is correct. Explanation: GDP in the beginning of 2015

 

= 3 ×

104  trillion dollar = 3.12 100 

= 3.12 trillion dollar

32. Option (1) is correct. Explanation: x2 – 3x + 1 = 0 or x2 + 1 = 3x On dividing both sides by x, we get 1 x  3 x

1 1 x 2  x   2 x x 1  2 1   x  2 x   x x    2

∴ 47 × 47 = 2209

1 1   x   2 x   x x  

So, required number = 2209 – 2170 = 39

= 9 – 2 + 3 = 10

OSWAAL CUET (UG ) 10 Mock Test Papers GENERAL TEST

102 33. Option (2) is correct.

Now, we know that,

Explanation: In the original mixture, 1 Water = × 240 = 60 cc 4 3  Glycerine =   240   180 cc 4 

Radius of Incircle =

Suppose x cc of water be mixed So,

15 S

or S =

15 =5 3

∴ Perimeter = 2S = 2 × 5 = 10 cm

60 + x 2 = 180 3

or 180 + 3x = 360 or 3x = 360 – 180 = 180 or x =

or 3 =

180 = 60cc 3

38. Option (3) is correct. Explanation: Given:

34. Option (3) is correct.

=

Sum of 0, 1, 2, 3, ... 110

2 −1

( 2)

Also,

=

 ∴ Unit digit = 5

2

−1

2 +1

=

(

(

)

2 −1

)(

2 +1

1

n( n  1)   Q1  2  3  ...  n  2  

4+ 3 and 

35. Option (2) is correct. Explanation: According to the questions, ….(i)

X+Y

= 2 × 68 = 136 kg

….(ii)

Y+Z

= 2 × 78 =156 kg

…. (iii)

Adding eqn (ii) and (iii), X + Y + Z + Y = 292 ⇒ Y = 292 – 222 ⇒ Y = 70 kg

36. Option (2) is correct. Explanation: Length of wire = circumference of circle = 2πr 22 44 × 84 = × 84 =2× 7 7 = 528 cm ∴ Perimeter of square = 528 cm

= 132 cm

37. Option (4) is correct. Explanation: Given that Area of triangle = 15 sq. cm Incircle radius = 3 cm

1

= 9− 8

1 1 1 1 ..........   1 2 2 3 3 4 8 9

 2  1  3  2  4  3.......... 5

X + Y + Z = 3 × 74 = 222 kg

528 4

= 3− 2

= 4− 3

8+ 9

 4 6 5 7 6 8 7 9 8

 9 1  3 1  2

39. Option (2) is correct. Explanation: Single equivalent discount xy   x  y  % 100   30  20     30  20   %   50  6  %  44% 100  

40. Option (2) is correct. Explanation:  1  rrr11   1  rrr22  A P A 1 1 2    1  100  A P P  11   100  100 100   100 100  10   12  12  1  10 10000 10   11  12  10000    1  10000  11   100  100 100 100  100   100  110 112 112  10000  110  112 10000  110   10000    100 100 100 100 100 100 11 28 28  10000  11  28 10000  11   10000    10 25 10 10 25 25 3080000  3080000 3080000   250 250 250 = ` 12,320

)

2 −1

= 2 −1

2+ 3

1

So, length of side =

1

Rationalising the denominator, we get,

Explanation: 110(110 + 1) 2 = 6105

Area of triangle Semi perimeter

MOCK TEST Paper-5

103

41. Option (1) is correct.

47. Option (4) is correct.

Explanation: Only daughter of Nitin's motherin-law is Nitin's wife. Son of Sanchi's mother's father is Nitin. Therefore, Nitin is brother of Sanchi's mother.

42. Option (2) is correct. Explanation: Correct relation between the given classes Doctors

Salaried Persons

48. Option (2) is correct. Explanation: As,

11 ×15 ×19

Parents





165

209

180

228

Same as,

43. Option (1) is correct.

12

Explanation:



According to the statements, Chains

Explanation: (a) Amphibian ….(4) (b) Amorphous ….(3) (c) Amphidextrous ….(5) (d) Ambiguous ….(1) (e) Ambivalent ….(2) Hence, (d), (e), (b), (a), (c) is the correct answer.

Rings

Lockets

×15 ×19

49. Option (1) is correct. Explanation:

Players

Medals

A B C D



Doctors

I. () II. ()

Hence, only conclusion II follows.

44. Option (3) is correct.

Singers

50. Option (4) is correct.

Explanation:

Explanation: F A

D

B

C

will complete the given figure.

45. Option (3) is correct. Explanation: As ‘Jackal’ sound is called ‘Howl’, in the same way the ‘Rain’ sound is called ‘Patter’.

46. Option (2) is correct. Explanation:

Deepak

Fix the position of E and F first. Here, clearly from the figure, F and B are the neighbours of A.

51. Option (2) is correct.

Given

Sunil 20

E

Sunil 28 36 Deepak

ence, total boys in the row = position from H left + Position from right – 1 = 28 + 36 – 1 = 63

Explanation: S

7 km

R

2 km T

NW

N NE

W 4 km

U

SW

E S

SE

Clearly, he is at point U in South-West direction from R.

OSWAAL CUET (UG ) 10 Mock Test Papers GENERAL TEST

104 52. Option (4) is correct.

57. Option (2) is correct.

Explanation:

Explanation: Given,

1 2 3

Maruti 20

2 4



1

3 4

Hence, by joining all the parts we can see the correct figure.

53. Option (4) is correct.

Middle Position Hence, total cars in the row = 30 + 30 – 1 = 59

58. Option (3) is correct. Explanation:

Eshwar Father

Explanation: Conclusion I have the facts that can be proved right from statement I, but conclusion II does not relate to any of the statements. Hence, only conclusion I follows.

54. Option (3) is correct. Explanation:

Given :

525 Q 25 R 42 S 4 = 189 ÷ + × ∴ 525 ÷ 25 + 42 × 4 = 21 + 168 = 189

Honda 30

Arsh Father Shivam

Explanation:

B

M

+3

O

+2

E

O

+3

+2

Q

+2

H

Q

+3

+2

S

+2

K

S

+3

+2

U

+2

N

U

+2

W

60. Option (1) is correct. Explanation: Given, 1

2

3







So,

  〈 



4

5

6

7







Explanation: Teeth are used to cut. Similarly, Legs are used to walk.

Dhruv

59. Option (2) is correct.

Explanation: So,

56. Option (1) is correct.

brother

herefore, from the figure, it is clear that Bimla T is the daughter-in-law of Eshwar.

55. Option (1) is correct.

will complete the figure.

Bimla son

8

9

  ◊ = 9  1  4  8  6



SOLUTIONS OF Question Paper 1. Option (4) is correct. Explanation: The Nanda dynasty was founded by Mahapadma Nanda, who ruled from around 467 BC. The Nanda dynasty was the first imperial dynasty of ancient India and played a significant role in shaping the political landscape of the region during that time. Dhanananda is traditionally considered to be the last ruler of the Nanda dynasty. He was overthrown by the famous Mauryan emperor Chandragupta Maurya, marking the transition from the Nanda dynasty to the Maurya dynasty. Alexander the Great did invade the Indian subcontinent in 326 BC. His invasion reached as far as the Punjab region in northwestern India. It's believed that the ruler of the Nanda dynasty at that time was indeed Dhanananda, making this statement accurate.

2. Option (1) is correct. Explanation: Jallianwala Massacre: This tragic event took place on April 13, 1919, in Amritsar, Punjab, during the British colonial period. It occurred before the other events listed. Formation of the Swaraj Party: The Swaraj Party was formed in 1923. This party was a political group in India that aimed to attain self-governance (Swaraj) within the British Empire. Gandhi-Irwin Pact: Also known as the Delhi Pact, this agreement was signed on March 5, 1931, between Mahatma Gandhi and the then Viceroy of India, Lord Irwin, during the Indian independence movement. Lucknow Pact: The Lucknow Pact was an agreement between the Indian National Congress and the Muslim League, reached in 1916. It aimed to present a united Indian front to the British government on constitutional reforms while also addressing some Muslim demands.

3. Option (4) is correct. Explanation: Stratus: These clouds, as indicated by their name, form layers that cover extensive portions of the sky. They typically result from either heat loss or the mixing of air masses with varying temperatures.

6

Altostratus clouds, a type of stratus cloud, are mid-level clouds that have a grey or blue-grey appearance, often covering the entire sky. Nimbus: Nimbus clouds are characterized by their black or dark grey colouration. They develop at middle altitudes or very close to the Earth's surface. These clouds are highly dense and opaque, sometimes appearing so low that they appear to touch the ground. They lack a distinct shape, existing as thick masses of vapour. Cumulonimbus: Cumulonimbus clouds, commonly known as "thunderheads," are noticeable on warm summer days and have the potential to bring about strong winds, hail, and rain. They often occur along cold fronts, where cool air is pushed beneath warmer air. As evening approaches, cumulonimbus clouds typically diminish in size as the moisture in the air evaporates. Gradually, they transform into stratocumulus clouds, which seldom produce rain.

4. Option (2) is correct. Explanation: The Pakistan Resolution was a landmark document in the history of the Indian independence movement. It called for the creation of a separate Muslim-majority state in British India. The resolution was passed by the All India Muslim League at its 27th annual session held in Lahore on 22–24 March 1940. The resolution was moved by Mohammad Ali Jinnah, the leader of the All-India Muslim League. Jinnah argued that the Muslims of British India were a separate nation and that they deserved their own state. He said that the Muslims of India could not live in peace and harmony with the Hindus and that the only way to ensure their safety and security was to create a separate Muslim state. The Pakistan Resolution was met with mixed reactions. The Hindus were opposed to the resolution, and they argued that it would divide India and weaken it. The British were also opposed to the resolution, and they did not want to see India divided. However, the Muslims of India were overwhelmingly in favour of the resolution, and they saw it as their only hope for a secure and prosperous future.

OSWAAL CUET (UG ) 10 Mock Test Papers GENERAL TEST

106

The Pakistan Resolution ultimately led to the creation of the independent state of Pakistan in 1947. The partition of India was a bloody and chaotic process, but it ultimately fulfilled the dream of the Muslims of India for a separate Muslim state.

5. Option (3) is correct. Explanation: The city of Madurai is located in Tamil Nadu on the bank of river Vaigai. It has found mentions in the text of Kautilya and Megasthenes dating back to 3rd century B.C. The ancient city has served as the capital of the Kingdom of Pandyas.

6. Option (2) is correct. Explanation: The Danube river in Europe is the river that gives through the ultimate number of countries in its pilgrimage. It passes through several countries, Bulgaria, Croatia, Moldova, Ukraine, Slovakia. It begins in Southern Germany and steers into the Black Sea via Romania.

7. Option (1) is correct. Explanation: The Nand Baba Milk Mission is a flagship scheme of the Uttar Pradesh government launched in June 2023 with a budget of `1,000 crore. The mission aims to increase milk production in the state by 20% in the next five years and provide a fair price to milk producers. The mission will focus on the following areas: • Improving animal husbandry: The mission will provide financial assistance to farmers to improve their animal husbandry practices. This will include providing training to farmers on modern animal husbandry techniques, providing subsidies for the purchase of high-quality breeds of cattle, and providing veterinary care to cattle. • Setting up dairy processing units: The mission will set up dairy processing units in all districts of Uttar Pradesh. This will help to increase the value addition of milk and provide employment opportunities to the youth. • Creating a market for milk: The mission will work to create a market for milk in Uttar Pradesh. This will include promoting the sale of milk through dairy cooperative societies and providing financial assistance to milk producers to set up their own milk vending units.

The Nand Baba Milk Mission is a major initiative of the Uttar Pradesh government to boost the dairy sector in the state. The mission is expected to benefit millions of milk producers in Uttar Pradesh and help to improve their livelihoods.

8. Option (2) is correct. Explanation: T he Maharashtra government has recently introduced a new financial initiative designed to offer assistance to farmers within the state. The program, named the "Namo Shetkari Mahasanman Yojana," received approval during a Cabinet meeting presided over by Chief Minister Eknath Shinde. Under this scheme, farmers in Maharashtra will receive an annual payment of ` 6,000. This financial support is in addition to the existing ` 6,000 that farmers already receive annually in instalments from the central government's “Pradhan Mantri Kisan Samman Nidhi Yojana.” The objective of this initiative is to provide an extra boost to farmers' income and enhance their overall financial stability.

9. Option (2) is correct. Explanation: The first ever motion to remove a Supreme Court Justice was signed in the year 1991, by 108 members of the Parliament. It was against Justice V. Ramaswami, who was accused of wrong doing related to his tenure as Chief Justice of the Haryana and Punjab High Court.

10. Option (3) is correct. Explanation: The first stable product of photosynthesis is phosphoglyceric acid (a three-carbon compound). Almost immediately, two of these join to form a glucose molecule. Some herbaceous plant such as sugarcane, corn and sorghum, form as the first products of photosynthesis not three-carbon but fourcarbon compounds – oxalacetic, malic and aspartic acids.

11. Option (2) is correct. Explanation: Trypsin is a protease enzyme that breaks down proteins. It is most active in an alkaline environment, with an optimum pH of 7.8 to 8.0. Amylase is an enzyme that breaks down carbohydrates. It is most active in an alkaline environment, with an optimum pH of 7.0. Therefore, trypsin and amylase are functional in the alkaline environment. The other enzyme pairs, trypsin and pepsin, and chymotrypsin and pepsin are both functional in an acidic environment. Pepsin is a protease enzyme that is most active in

MOCK TEST Paper-6 a n acidic environment, with an optimum pH of 1.5 to 2.5.

12. Option (4) is correct. Explanation: As of August 12, 2023, 18 nations have officially attained recognition from the World Health Organization (WHO) that they have successfully eliminated trachoma as a public health concern. The 18th nation is Iraq. This achievement also marks Iraq as the 50th country to eliminate at least one Neglected Tropical Disease (NTD) on a global scale.

13. Option (2) is correct. Explanation: Aryabhatta, was the first satellite launched by Indian Space Research Organization on 19 April, 1975. Later, on June 7, 1979 an Earth observation satellite Bhaskara-I was launched for India. India launched its own satellite for the first time on July 18, 1980. It was the Rohini-1 Satellite Launch Vehicle (SLV) rocket from the Sriharikota island launch site.

14. Option (1) is correct. Explanation: Pakhawaj is not a vocal form of Hindustani Classical music. It is a two-headed drum which is barrel shaped, also known as mridang. Tarana, Draupad, Khyal and Dhamer are major vocal forms of Hindustani Classical music. Hindustani Classical music originated in 12th century C.E.

15. Option (4) is correct. Explanation: A moment of significance unfolded in the realm of archery for India as Aditi Gopichand Swami secured a remarkable achievement by earning the first individual gold medal for India at the World Archery Championships 2023. This remarkable victory was realized through a gripping competition in the final of the women's compound event in Berlin, Germany, where she prevailed against Andrea Becerra of Mexico. Further contributing to this historic occasion, the 2023 Berlin World Archery Championships witnessed the Indian women's compound team attaining gold medal glory in the final by surpassing Mexico. This triumphant team consisted of Jyothi Surekha Vennam, Aditi Gopichand Swami, and Parneet Kaur, thus establishing themselves as the first Indian women's compound team to claim the world title.

107 16. Option (1) is correct. Explanation: The capability of a visual mechanism or style of flick to split or differentiate slight or closely bordering images of the human eye with 20/20 eyesight is commonly assumed to be about ‘one arc minute’ or ‘60 arc seconds’.

17. Option (4) is correct. Explanation: E rnest Rutherford’s interest was to understand how the electrons are positioned within an atom. For this, Rutherford organized an investigation in which fast-moving alpha (a)-particles were constructed to drop on the gold foil.

18. Option (2) is correct. Explanation: Liquid hydrogen has been used as a fuel in rockets and space shuttles as it supplies a characteristic impulse almost 30%– 40% higher than further rocket fuels. Liquid oxygen and liquid hydrogen are used as the fuel in the high-efficiency main motors of the space shuttle.

19. Option (4) is correct. Explanation: 2 February – World Wetlands Day: This day is dedicated to raising awareness about the importance of wetlands for the environment and human well-being. Wetlands play a crucial role in water purification, flood control, carbon storage, and providing habitat for various species. 22 March – World Water Day: World Water Day focuses on the importance of freshwater and advocating for the sustainable management of water resources. It aims to address issues such as water scarcity, sanitation, and water-related challenges faced by communities worldwide. 7 April – World Health Day: World Health Day is observed to promote global health awareness and highlight a specific health theme. It is organized by the World Health Organization (WHO) and is meant to draw attention to important health issues. 22 April – World Earth Day: Earth Day is celebrated to raise awareness about environmental issues and promote actions to protect the planet. It emphasizes the need for sustainable practices and encourages people to take part in environmental conservation efforts.

20. Option (2) is correct. Explanation: Sir Frank Whittle and Dr. Hans von Ohain are considered as co-inventor of the Jet Engine. Hans von Ohain is recognized as the designer of the first operational turbojet engine. Frank Whittle was the first to register a patent for the turbojet engine in 1936. Hans von Ohain’s jet was the first to fly in 1939 and Frank Whittle’s in 1941.

OSWAAL CUET (UG ) 10 Mock Test Papers GENERAL TEST

108 21. Option (2) is correct.

Explanation: If x is subtracted from each quantity, the new mean is x less than the earlier mean. So, the required new mean = 18 – 2 = 16

22. Option (3) is correct. Explanation: Given that: Average score of 50 students = 79.5 \ Total score of 50 students = 79.5 ×50 = 3975 According to the question, 3975 − ( 94 + 69 ) + ( 49 + 89 ) 50 = 79

Correct means =

23. Option (3) is correct. Explanation: The given number is divisible by 72, if it is divisible by 8 and 9 So, the number is divisible by 8 if 5x6 is divisible by 8, When x = 7, Then 576 ÷ 8 = 72 The number is divisible by 9 if sum of its digits is divisible by 9. So, y + 56 is divisible by 9. For y = 7, y + 56 = 63 which is divisible by 9. ∴

xy  7  7  7

24. Option (4) is correct. Explanation: According to the question, Speed of current 1 (Downstream rate – Upstream rate) = 2 1 = (14 – 8) kmph = 3 kmph 2

25. Option (3) is correct. Explanation: Suppose the numbers be 2x and 3x respectively. As per question, 2x + 8 3 = 3x + 8 4 or 9x + 24 = 8x + 32 or 9x – 8x = 32 – 24 = 8 or x =8 So, the sum of numbers = 2x + 3x = 5x = 5 × 8 = 40

26. Option (1) is correct. Explanation: Journey on foot = x km Journey on cycle = (80 – x) km x 80  x So  7 8 16 2 x  80  x 7 or 16 or x + 80 = 16 × 7 = 112 or x = 112 – 80 x = 32 km

27. Option (2) is correct. Explanation: Let S be the sum of infinite terms of GP, having first term a and common ratio r. a =4 (1) So, S1 = 1−r According to the question, S2 =

a3 = 192 1 − r3

(2)

Putting value of a in equation (2) from (1). 64 (1 − r )3 = 192 1 − r3 2 r 2 + 5r + 2 = 0

⇒ ⇒ ⇒

r = −2 or

– 2 is not possible here. 1 So, r = − 2

1 2

28. Option (4) is correct. Explanation: The least number = LCM 4, 5, 8, 10 and 12. 2

4, 5, 8, 10, 12

2

2, 5, 4, 5, 6

2

1, 5, 2, 5, 3

3

1, 5, 1, 5, 3

5

1, 5, 1, 5, 1 1, 1, 1, 1, 1

∴ LCM of 4, 5, 8, 10 and 12 = 2 × 2 × 2 × 3 × 5 = 120 So, the least number = 120

29. Option (2) is correct. Explanation: Given, 9 x = 12 + 147 or 9 x  3  2  2  3  7  7 or 9 x = 2 3 + 7 3 = 9 3 or x = 3

30. Option (4) is correct. Explanation: Given that, the base and altitude of an isosceles triangle are 10 cm and 12 cm respectively. A

12 B

D 10

C

MOCK TEST Paper-6

109 32. Option (4) is correct.

Now,

Explanation: Given: 24010000 = 4900 and 4900 = 70

AC  AD 2  DC 2

12    5  2



2



 144  25 =

Explanation: Given that, Perimeter of an isosceles right triangle is





2  1 cm.

Now, ABC is an isosceles right triangle.

Explanation: The pattern is 3×2+1=6+1=7 7 × 2 + 2 = 14 + 2 = 16 16 × 2 + 3 = 32 + 3 = 35 35 × 2 + 4 = 70 + 4 = 74 ≠ 70 74 × 2 + 5 = 140 + 5 = 153

34. Option (1) is correct.

C

Explanation: Suppose 0.73 = a and 0.27 = b Given :

x

a

= A

Explanation: Let the marked price of the trip = ` x. and its cost price be ` 100.

or x2 = a2 + a2 = 2a2 And, Perimeter = 8





2 1

2 1

...(i)





 2  1 or a 2  2 a  8  2  1  or a  2  2   8  2  1 8  2  1 or a 

or x + 2a = 8

(From eq. (i))

2 2

  2  2  2  2  8 2 2  2  2  2  or a  or a 

8



2 1

2 2

22  2

or a 

8 2  42

a3 + b 3 ( a + b )( a 2 − ab + b 2 ) = 2 a − ab + b a 2 − ab + b 2 2

35. Option (1) is correct.

AC2 = AB2 + BC2 or x = a 2 

(0.73)3 + (0.27 )3 (0.73) + (0.27 )2 − (0.73) × (0.27 ) 2

= a + b = 0.73 + 0.27 = 1

B

a

Using Pythagoras theorem,

or x + a+ a = 8

24010000 = 70

33. Option (1) is correct.

= 169 13 cm.

31. Option (1) is correct.

8

4

2

S.P. = 141% of C.P. = ` 141 According to the question, 75x 75% of M.P. = ` 141   141 100 141 × 100 ⇒x= = ` 188 75 After a discount of 26% Required S.P. = (100 – 26)% of ` 188  188  74  =`    100  = ` 139.12 So, required profit % = 39.12

36. Option (1) is correct. Explanation: According to the question, we have Width of the road = r2 – r1 Given that the difference between circumference is 66 metre. B ∴ 2πr2 – 2πr1 = 66

[Q a2 – b2 = (a – b) (a + b)]

or a = 4 2 ∴ Length of hypotenuse = 4 2 × 2 = 8 cm.

r2 r1

A

OSWAAL CUET (UG ) 10 Mock Test Papers GENERAL TEST

110

41. Option (3) is correct.

or 2π (r2 – r1) = 66 or r2 – r1 =

Explanation:

66 66  7 462   2 2  22 44

×2 – 1

= 10.5 metres

1

37. Option (4) is correct.

×2 + 1

Explanation: Given that, Area of square = 24 sq. units We know that, Side of square = =

3

24 = 2 6 units



5

11

×2 + 1

×2 – 1

21

43

×2 – 1

85

×2 + 1

171

341

×2 + 1

42. Option (3) is correct. Explanation: Correct relation between the given classes,

Area



So, perimeter of square = 4  2 6 units = 8 6 units

×2 – 1

Entrepreneurs

Engineers

38. Option (1) is correct. Explanation: C.P. of I toy = ` x

Women

C.P. of 2nd toy = ` y 504  100 x  112 = ` 450 So,  504 ⇒ x  100 112 Again, y 

96 504  100  504  y  100 96



43. Option (1) is correct. Explanation: According to the statements,

= ` 525 Total C.P. = ` (450 + 525) = ` 975

Newspaper

The total S.P. = 2 × 504 = ` 1008 Gain = 1008 – 975 = ` 33

Photographs

33  100 44 5 ∴ Required, profit %   3 % 13 13 975

39. Option (2) is correct.



Explanation: Work done by both son in 1 hour = =

44. Option (3) is correct. 1 2

∴ Required time = 2 hours.

40. Option (1) is correct. Explanation: Let the required time be t years PRT   According to the question, SI  100    6000  4  5 8000  3  t ⇒  100 100 ⇒

120000 24000  t  100 100

⇒t=

120000 = 5 years 24000

I. () II. () III. ()

Hence, all of the conclusions follows.

1 1 21 1    3 6 6 2

∴ Work done by father in an hour =

Computers

Explanation:

M

h

+1

N

g

+1

O

+1

Q

–1

y

–1

f

+1

P

z

–1

–1

x –1

e

–1

w –1

d

–1

v

ence, Ludo is between the Carrom and H Monopoly games.

MOCK TEST Paper-6

111

45. Option (1) is correct.



Explanation:

51. Option (1) is correct. Explanation: So,

2

1

3

4

5

2 3 4

will complete the given figure.

5

Row: 5 + 4 + 3 + 2 + 1=15. Column: 5+ 4 + 3 + 2 + 1 = 15 = 15 × 15 = 225

52. Option (3) is correct.

46. Option (1) is correct. Explanation: As living place of ‘Bee’ is ‘Hive’, in the same way living place of ‘Hare’ is called ‘Burrow’.

47. Option (3) is correct.

Explanation: Dice is single so it is standard Dice. And we know the sum of the opposite side of standard Dice is 7. So, 3 is opposite to 4.

53. Option (1) is correct. Explanation:

R

Explanation: We know that, orange is a citrus fruit. But here orange has been called guava. So, guava will be a citrus food.

–2

P

48. Option (4) is correct.

–2

Explanation: According to the given information, Sunny/ Vicky < Vicky/Sunny < Hitesh < Bharat < Nitin Hence, Nitin has the maximum height among them.

N –2

L –2

J

49. Option (3) is correct. Explanation: As, Same as,

278

296

+9 313

322

O

Z

+2

–2

Q

X

+2

–2

S

V

+2

–2

U

T

+18

der  

  stay  here y er mer 

I  stay  there yer  fer 

jer (i)

ner (ii)

ner  

der

(iii)

jer (iv)

rom equation (i), (ii), (iii), and (iv), you = ter, F stay = yer. Hence, you stay late = ter yer ser.

5 ⇒

2

5

1

are   there ter 

are   late ser 

4

3

340

Explanation: According to the question,

we  

–2

Explanation:

50. Option (1) is correct.

we

+2

+18

+9

you 

B

54. Option (1) is correct. 269



M

1

4

3

2

ence, by joining all the parts we can see the H correct figure.

55. Option (3) is correct. 664 + 4 ÷ 34 × 28 = ? ↓  ↓   ↓ ÷ +



⇒ 664 ÷ 4 + 34 – 28 ⇒ 166 + 6 ⇒ 172

OSWAAL CUET (UG ) 10 Mock Test Papers GENERAL TEST

112 56. Option (3) is correct.

59. Option (3) is correct.

Explanation: Neither conclusion I nor conclusion II can be proved correct from the statement I and statement II. Hence, neither I nor II follows.

Explanation:

Celsius

is

a

unit

mother H

brother

R mother r

te

sis

T

of

Similarly, Kilometer is a unit of distance.

58. Option (2) is correct. Explanation: According to the given information, T Indus Tower > Pearl Tower > Sky Tower Hence, Pearl Tower is the second shortest.

Explanation:

C

6 km A

+5

Explanation:

56. Option (4) is correct.

Grand son

+5

59. Option (1) is correct.

will complete the figure.

+5

NW

N

×1

13 +1

26 ×2

28 +2

84 ×3

87 +3

NE

W SW

12

E S

SE

∴ Required distance AE = BC – DE = 9 – 3 = 6 km



SOLUTIONS OF Question Paper 1. Option (3) is correct. Explanation: Sindhu is the Vedic name for the Indus River. The Indus River is the longest river in Pakistan and the secondlongest river in India. It flows through the Himalayas, the Hindu Kush, and the Thar Desert. Vitasta is the Vedic name for the Jhelum River. The Jhelum River is a tributary of the Indus River. It flows through the Kashmir Valley and into Pakistan. Asikni is the Vedic name for the Chenab River. The Chenab River is a tributary of the Indus River. It flows through the Punjab region of Pakistan and India. Parushni is the Vedic name for the Ravi River. The Ravi River is a tributary of the Indus River. It flows through the Punjab region of India and Pakistan. Vipas is the Vedic name for the Beas River. The Beas River is a tributary of the Indus River. It flows through the Himachal Pradesh and Punjab regions of India. Sutudri is the Vedic name for the Sutlej River. The Sutlej River is a tributary of the Indus River. It flows through the Himachal Pradesh, Punjab, and Haryana regions of India.

2. Option (1) is correct. Explanation: SAARC was established in 1985 and has 8 member countries: Afghanistan, Bangladesh, Bhutan, India, Maldives, Nepal, Pakistan, and Sri Lanka. BIMSTEC was established in 1997 and has 7 member countries: Bangladesh, Bhutan, India, Myanmar, Nepal, Sri Lanka, and Thailand. Of the given options only Bangladesh, Bhutan, and India are members of both SAARC and BIMSTEC. Pakistan, Maldives, and Afghanistan are the only members of SAARC, while Thailand and Myanmar are the only members of BIMSTEC.

3. Option (1) is correct. Explanation: V. Narahari Rao served as the inaugural Comptroller and Auditor General (CAG) of India, holding the position from 1948 to 1954. An Indian civil servant, he was

8

associated with the Indian Audit and Accounts Service during the post-independence era. In recognition of his contributions to the civil service, he was honoured with the Padma Bhushan, the third-highest civilian award in India, in 1954. Sukumar Sen held the distinction of being the first Chief Election Commissioner of India, holding office from 21 March 1950 to 19 December 1958. Under his leadership, the Election Commission impeccably orchestrated India's first two general elections, which took place in 1951–52 and 1957. The Attorney General of India assumes the pivotal role of the government's primary legal counsel and representative in the Supreme Court of India. M. C. Setalvad was the pioneering Attorney General of India, while the present incumbent is R. Venkataramani. The inception of the initial Finance Commission took place through a Vice Presidential Order dated 22.11.1951, with Shri K.C. Neogy as its inaugural chairman. This commission was formed on 6th April 1952. Subsequently, fifteen Finance Commissions have been established at five-year intervals. The President appoints the Finance Commission under Article 280 of the Constitution, primarily to provide recommendations on the equitable allocation of tax revenues between the Union and the States, as well as among the States themselves.

4. Option (4) is correct. Explanation: In a splendid event, the victors of the 69th National Film Awards were unveiled. Allu Arjun received recognition as the Best Actor for his role in the movie "Pushpa," whereas Alia Bhatt and Kriti Sanon were honoured with the Best Actress Award for their performances in "Gangubai Kathiawadi" and "Mimi" respectively. "The Nambi Effect" secured the esteemed title of Best Feature Film, while "The Kashmir Files" clinched the Nargis Dutt Award for Best Film on National Integration. • list of winners at the 69th National Film Awards: • Best Feature Film: Rocketry • Best Director: Nikhil Mahajan, Godavari

OSWAAL CUET (UG ) 10 Mock Test Papers GENERAL TEST

122

• Best Popular Film Providing Wholesome Entertainment: RRR • Nargis Dutt Award for Best Feature Film on National Integration: The Kashmir Files • Best Actor: Allu Arjun, Pushpa • Best Actress: Alia Bhatt, Gangubai Kathiawadi and Kriti Sanon, Mimi • Best Supporting Actor: Pankaj Tripathi, Mimi • Best Supporting Actress: Pallavi Joshi, The Kashmir Files • Best Child Artist: Bhavin Rabari, Chhello Show

5. Option (4) is correct. Explanation: Lakhpati Didi Scheme: The primary objective of this initiative by the government is to empower women in rural areas, aiming to generate two crores of financially self-sufficient "lakhpati didis" or prosperous sisters. This scheme is aligned with the broader mission of reducing poverty and enhancing economic empowerment. Under the Lakhpati Didi Scheme, women residing in villages will undergo skill development training to enable them to earn an annual income exceeding Rs 1 lakh. The scheme involves the provision of drones to women's Self-Help Groups (SHGs) for agricultural tasks. This endeavour intends to blend technology with the agricultural sector's transformation, concurrently bolstering the status of women in rural societies. Approximately 15,000 SHGs composed of women will receive comprehensive training in drone operation and maintenance. This training not only opens doors to fresh income-generating avenues but also equips women with state-of-the-art proficiencies. Drones hold immense potential to revolutionize agriculture through precision farming, real-time crop surveillance, and effective pest management. Additionally, the program encompasses training women in a range of skills such as crafting LED bulbs and plumbing, among others.

6. Option (1) is correct. Explanation: The thermal growth causes the Eiffel Tower to be taller during summers. Thermal elaboration is when material changes measurements, it experiences temperature differences. Materials, the tower made of iron experiences thermal expansion.

7. Option (1) is correct. Explanation: India holds the position of the world's largest millet producer. In the year 2020, two types of millet from India, namely Pearl Millet (Bajra) and Sorghum (Jowar), jointly constituted approximately 19 per cent of the global millet production. Among these, Pearl Millet's share in the world production was 40.51 per cent, while Sorghum accounted for 8.09 per cent in the same year. Noteworthy millet-producing states in India encompass Rajasthan, Karnataka, Maharashtra, Uttar Pradesh, Haryana, Gujarat, Madhya Pradesh, Tamil Nadu, Andhra Pradesh, and Uttarakhand. Collectively, these ten states were responsible for nearly 98 per cent of millet production within India during the 2020-21 period. More specifically, the millet production landscape was dominated by six states: Rajasthan, Karnataka, Maharashtra, Uttar Pradesh, Haryana, and Gujarat, contributing to over 83 per cent of the total millet yield. Rajasthan, in particular, stood out by contributing 28.61 per cent of India's entire millet production.

8. Option (1) is correct. Explanation: The largest fish-exporting region of the world is the Northeast Atlantic region. It accounts for about 30% of the global fish export market. The region includes countries such as Norway, Iceland, and the Faroe Islands. These countries have a long history of fishing and aquaculture, and they have developed highly efficient fishing and processing industries. The Northeast Atlantic region is also home to some of the most productive fishing grounds in the world. The region's cold, nutrient-rich waters support a wide variety of fish species, including cod, haddock, and salmon. The other regions you mentioned are also important fish exporters, but they do not export as much fish as the Northeast Atlantic region. The Northwest Pacific region is the second largest fish exporting region, followed by the Southeast Asian region.

9. Option (4) is correct. Explanation: Hatti tribe: The Hatti tribe is found in the state of Himachal Pradesh, primarily in the Chamba district. They are known for their distinct cultural practices and are part of the larger Gaddi community in the region. Narikoravan tribe: The Narikoravan tribe resides in the state of Tamil Nadu, primarily in urban areas like Chennai. Historically, they

MOCK TEST Paper-8 were involved in various professions like street performance, acrobatics, and fortunetelling. Over time, they have faced social and economic challenges. Toda tribe: The Toda tribe is indeed from Tamil Nadu, primarily residing in the Nilgiri Hills. They are known for their unique culture and distinctive barrel-shaped huts. The Toda people traditionally practised buffalo pastoralism and have a rich mythology and religious system. Binjhia tribe: The Binjhia tribe is predominantly found in the state of Jharkhand. They are one of the indigenous communities of the region, with their own cultural traditions, languages, and way of life. The Binjhia tribe was already listed as ST in Jharkhand and Odisha but not in Chhattisgarh. Now, this tribe is added to the list of STs in Chhattisgarh.

10. Option (2) is correct. Explanation: Fiber optics (or optical fiber), refers to the medium and the technology associated with a high-speed data transmission. It contains tiny glass or plastic filaments that carry light beams. The light signals do not travel at the speed of light because of the denser glass layers, instead travelling about 30% slower than the speed of light.

11. Option (2) is correct. Explanation: The male part of a flower is stamen. It is composed of the filament and the anther. It bears the pollen grain. The female part of a flower is called the pistil and it is composed of ovary, the stigma, and the style.

12. Option (4) is correct Explanation: World Hindi Day is celebrated on 10 January every year to promote the Hindi language. It was first celebrated in 2006 under the instructions of then-Prime Minister Manmohan Singh. The date of 10 January was chosen as Hindi was spoken for the first time in the United Nations General Assembly on 10 January 1949. National Youth Day is celebrated on 12 January every year to mark the birth anniversary of Swami Vivekananda. Swami Vivekananda was a Hindu monk and philosopher who played a major role in the introduction of Indian philosophy to the West. He is also considered the father of modern India. National Girl Child Day is celebrated on 24 January every year to raise awareness about the girl child and to promote her rights. The day was first celebrated in 2011 by the Ministry of Women and Child Development.

123 Martyrs Day or Shaheed Diwas is celebrated on 30 January every year to commemorate the death anniversary of Mahatma Gandhi. Mahatma Gandhi was the leader of the Indian independence movement and is considered the father of modern India. He was assassinated on 30 January 1948.

13. Option (2) is correct. Explanation: Mihir sen was a famous long distance swimmer. He was first Indian to swim across English Channel from Dover to Calais in 1958. He was the only man who swim the oceans of five continents in one year.

14. Option (1) is correct. Explanation: Louis Pasteur was a French microbiologist who is considered the father of microbiology. He made many important discoveries in the field of microbiology, including the discovery of fermentation. Edward Jenner was an English physician who is credited with developing the first smallpox vaccine. The smallpox vaccine is a suspension of cowpox virus that is injected into the body to protect against smallpox. James Watson and Francis Crick were American molecular biologists who discovered the structure of DNA. DNA is the genetic material that contains the instructions for building and maintaining an organism. Alexander Fleming was a Scottish bacteriologist who discovered penicillin. Penicillin is an antibiotic that is used to treat bacterial infections.

15. Option (3) is correct. Explanation: B al Gangadhar Tilak was a freedom fighter and social reformer who is considered the "Father of Indian Unrest". He was a strong advocate of Swaraj, or self-rule, for India. Dr Rajendra Prasad was the first President of India. He was a respected lawyer and politician who was known for his honesty and integrity. He was given the nickname "Deshbandhu", which means "friend of the nation". Dadabhai Naoroji was an economist and politician who is considered the "Grand old man of India". He was a strong advocate for Indian independence and was the first Indian to be elected to the British Parliament. Chittaranjan Das was a freedom fighter and lawyer who was known for his eloquence and his commitment to social justice. He was given the nickname "Desh Ratna", which means "jewel of the nation".

OSWAAL CUET (UG ) 10 Mock Test Papers GENERAL TEST

124 16. Option (2) is correct.

Explanation: Marfati songs are traditional folk songs of Bangladesh. Purbeli songs are folk songs of Nepal. Cartman’s songs, Baul songs, Jari gaan, Kabi gaan and Pala gaan are some other traditional Bangladeshi songs.

21. Option (2) is correct. Explanation: By using the rule of alligation, we get Variety I

Variety II (` 7)

(` 12) (` 8)

17. Option (3) is correct. Explanation: Savitribai Phule belongs to Maharashtra. She was born on January 3, 1831, and was married to Jyotirao Phule. Her husband decided to educate her in order to elevate the status of women. She was a social reformer and worked a lot for women empowerment. She died in 1897 of Plague while helping the people affected with Bubonic Plague epidemic. Her writings – Kavya Phule; Go, Get Education; Bavan Kashi Subodh Ratnakar.

8-7=1

So, the required ratio = 1 : 4

22. Option (4) is correct. Explanation: After arranging the data in ascending order. 3, 5, 6, 7, 9, 9, 10 Here the number of observations is odd.  n + 1 So, Median =   2   7 + 1 =  2 

18. Option (3) is correct. Explanation: The Archaeological Survey of India (ASI) made a remarkable discovery of a 1,300-year-old stupa right within a mining site in the Jajpur district of Odisha. This location served as the source of Khondalite stones used in the beautification project of the Shree Jagannath Temple in Puri during the 12th century. Preliminary assessments suggest that the stupa could be around 4.5 meters in height, possibly dating back to the 7th or 8th century. This significant find was made at Parabhadi, situated in close proximity to Lalitagiri, a prominent Buddhist complex renowned for its numerous stupas and monasteries. Lalitgiri holds a special place among the three sites (Lalitagiri, Ratnagiri, and Udayagiri) due to its belief as the most sacred. It is here that a massive stupa was excavated, housing a relic of Buddha inside a stone casket.

19. Option (3) is correct. Explanation: The substance used as a replacement for chlorine in bleaching is Hydrogen peroxide. It is an excellent substitute for bleach because of the oxidation of colouring matter by nascent oxygen. It is used for bleaching of wool, silk and cotton.

20. Option (1) is correct. Explanation: I n the pure state, Tungsten has the most elevated melting point, i.e., 3422°C or 6192°F. It also has the most increased tensile force and most low vapor pressure. Helium is the chemical component with the lowest melting point, while the most increased one is Carbon.

12 - 8 = 4

th

term

th

term

= 4 th term =7

23. Option (2) is correct. Explanation: Single equivalent discount for 10% and 12% 12  10     12  10   %  20.8% 100   Single equivalent discount for 20.8% and 5% 20.8  5     20.8  5   %  24.76% 100  

24. Option (3) is correct. Explanation: 4x573y is divisible by 72. ∴ 4x573y is divisible by 8 and 9. 73y is divisible by 8. ∴y=6 4x573y is divisible by 9. Then 4 + x + 5 + 7 + 3 + 6 = 27 ⇒ 25 + x = 27 ⇒ x = 27 – 25 = 2 ∴x+y=2+6=8

25. Option (2) is correct. Explanation: R   A  P1  100  

T

R   4000  2000  1   100   R    2 1  100  

2

2

MOCK TEST Paper-8  1

125 ...(i)

R  2 100

R   Now, 8000  2000  1    100   4  ( 2 )T 4

 ( 2)  ( 2)

T

[ From eq (i )] T

⇒ T = 4 years

26. Option (1) is correct. Explanation: Total possible outcome when a pair of dice thrown = 36 According to the question, possible Outcomes = (1, 2), (1, 4), (2, 1), (2, 3), (2, 5), (3, 2), (3, 4), (4, 1), (4, 3), (5, 2), (5, 6), (6, 1), (6, 5), (1, 6) So required probability = 14/ 36 = 7/18

27. Option (2) is correct. Explanation: Given, 1

x3 3  100 5

On squaring both sides, we get 1−

3

9 x = 100 25

or

9 x3 =1− 100 25

or

x 3 25 −9 16 = = 100 25 25

or x 3 

16  100  64 25

or x = ( 4 ) 3

3

or x = 4

28. Option (1) is correct. Explanation: Total production of cars in 2018 = 270 thousands = 38 + 40 + 54 + 68 + 70 = 270 thousands Production of type C cars = 54 thousands Now, 270 thousand ≡ 360˚ 360 ∴ 54 ≡ × 54 = 72˚ 270

29. Option (4) is correct. Explanation: Let the number be x. According to the question, 7% of x = 84 7  x  84 100 8400 x  1200 7

30. Option (1) is correct. Explanation: Q3=3×1 5=5×1 7=7×1 9=3×3 ∴ LCM of 3, 5, 7 and 9 = 3 × 5 × 7 × 3 = 315 We know that, largest 4-digit number = 9999

315

9999 31



945



549



315



234

So, the greatest 4-digit number divisible by 315 = 9999 – 234 = 9765 Q The difference between each divisor and remainder is 2. ∴ Required number = 9765 – 2 = 9763

31. Option (1) is correct. Explanation: Given, first term a = –20 last term l = 28 and n = 17 n ∴ Required sum = ( a + l ) 2 17 = ( 20  28 ) 2 17  8 =  68 2

32. Option (2) is correct. Explanation: Distance covered 5   30  25km 6 Time Taken = 3 hours 20 minutes 1 10  3 hours  hours 3 3 Distance So, speed  Time  25   km / h 10     3   25  3    km / h  10  = 7.5 km/hr

OSWAAL CUET (UG ) 10 Mock Test Papers GENERAL TEST

126 33. Option (4) is correct.

Explanation: Given that, A circular wire of length = 168 cm Rectangle sides ratio = 5 : 7 Now, Breadth of rectangle = 5x cm Length of rectangle = 7x cm Now, perimeter = 2 (l + b) 168 = 2 (7x + 5x) 168 or 12x = = 84 2 84 or x = =7 12 Diagonal of rectangle =  7 x 2   5x 2 =

49 x 2 + 25x 2

=

74 x 2  74  7 2  3626 cm

Now, ∠BCD = ∠DCA = 30° ∠DCE = 180° ∴ ∠ACE = 180° – 30° = 150° AC = CE 30° = 15° ∴ ∠CAE = ∠CEA = 2

39. Option (3) is correct. Explanation: 1 1 2 days’ work of (B + C)’s = 2     20 30  32 = 2    60 

Now remaining work

34. Option (4) is correct. Explanation: For A, C.P. of horse = 4800 ×

100 = ` 6000 80

6000 × 115 = ` 6900 100 B’s profit = ` (6900 – 4800) = ` 2100

For B, S.P. =

∴ A’s complete work

40. Option (2) is correct. Explanation: Given: N =

35. Option (4) is correct. 

Explanation: Here, x = 5%, R = 240, y = 8% ∴ C.P. =

R 240  100   100 = ` 8000 yx 85

36. Option (3) is correct. Explanation: Here, a = 7, b = 16, c = 43 and d = 79 bc − ad So, the required number x = ( a + d ) −(b + c ) 16 × 43 − 7 × 79 = (7 + 79 ) −(16 + 43) 688 − 553 35 = = =5 86 −79 7

7− 3 7+ 3

( 7  3 )( 7  3 ) ( 7  3 )( 7  3 )



7  3  2 7  3 10  2 21  73 4



2( 5  21 ) 5  21  4 2

Now,

1 2 2 5  21    N 5  21 5  21 5  21 

 N

5  21 2

1 5  21 5  21 10    5 N 2 2 2

41. Option (1) is correct.

37. Option (1) is correct.

Explanation:

Explanation: In an equilateral ∆ABC, ∠A = ∠B = ∠C = 60° AB = BC = CA ∴ AX = BY = CZ

+2

+2

+2

+2

+2

a,

cd,

fgh,

jklm,

opqrs,

uvwxyz

1

2

3

4

5

6

42. Option (1) is correct.

38. Option (4) is correct. Explanation: Given that, ∆ABC is an equilateral triangle AC = CE

Explanation: Correct relation between the given classes Literates Graduates

A

D B

5 1  60 6 1 5 = 1  6 6 5 =  18  15 days 6

= 2

C

E



Teachers

MOCK TEST Paper-8

127

43. Option (3) is correct.

Similarly, D  E  C  O  D  E

Explanation: According to the statements,

–2 +1 –2 +1 –2 +1

Tree X

Pomegranates

Plum

I. () II. () III. () Hence, all the conclusions follow.

50. Option (2) is correct. Explanation: According to the given information, Q